Ευκλειδης Β 60

84
β r g( β ) f g'(x)f(g(x))dx = J _ f(x)dx α g( α ) J g(x) , . α f(t)d t = f(g(x))·g'( x) J β β J β f '(x)g(x)dx= [t(x)g(x)J - f(x)g'(x)dx α α α ατική Εταιρεία

description

Ευκλειδης Β 60

Transcript of Ευκλειδης Β 60

Page 1: Ευκλειδης Β 60

β rg(β) f g'(x)f(g(x))dx= J_ f(x)dx α g(α)

Jg(x) , . α f(t)dt = f(g(x))·g'(x)

Jβ β Jβ f '(x)g(x)dx= [t(x)g(x) J - f(x)g'(x)dx α α α

ατική Εταιρεία

Page 2: Ευκλειδης Β 60

ιδεuιι ά Βιβλία yια μαβητέ�

και εκπαιδευτικού� MAPIA ΕΥΠΑθΙΟΥ • ΕΛΕΥθΕΡΙΟΣ ΠΡΟΤΟΠΑΠΑΣ

ΜΕθΟΔΟλΟΓ IA

σ' ινισΙοu Λuκcrou

Μεθοδολογία Άλγεβρας Α' Ενιαίου Λυκείου

Μ. Ευσταθίου, Ε. Πρωτοπαπάς

ΕΛΕΥΘΕΡΙΟΙ: ΠΡΟΤΟΠΑΠΑΣ

ΜΕΘΟΔΟΛΟΓΙΑ

/)' ενιαίου Λυκείου

Μεθοδολογία Άλγεβρας Β' Ενιαίου Λυκείου

Γ ενικής Παιδείας Ε. Πρωτοπαπάς

ΕΚΔΟΣΕΙΣ ΠΑΤΑΚΗ www.patakis.qr

Γεωμετρία Α' Ενιαίου Λυκείου Γ. Βιδάλης, Β. Γ κιμίσης

Αρχές οικονομικής θεωρίας Γ Ενιαίου Λυκείου

Π. Μηλίτσης

Γεωμετρία Β' Ενιαίου Λυκείου

Γενικής Παιδείας Ε. Πρωτοπαπάς

μαθηματικά ι;· τόμος

ΘΕfΙΚΗΣ ΚΑΙ TEXNOΛOflkHΣ: t(AfEYθ'fNDii

Μαθηματικά Γ Ενιαίου Λυκείου

Θετικής και τεχνολογικής κατεύθυνσης Α· τόμος: Ε. Πρωτοπαπάς

Β' τόμος: Ε. Πρωτοπαπάς, Σ. Γκούμας

σ ε: ό ι4 α. τ α. b ι b n ι o n w n ε:ia. ΒΙΒΛΙΟΠΩΛΕΙΟ ΠΑΤΑΚΗ: ΑΚΑΔΗΜΙΑΣ 65, 106 78 ΑΘΗΝΑ, ΤΗΛ. 210.38.11.850

KENTPIKH ΔΙΑΘΕΣΗ: ΕΜΜ. ΜΠΕΝΑΚΗ 16, 106 78 ΑΘΗΝΑ, ΤΗΛ. 210.38.31.078

ΥΠΟΚ/ΜΑ: Ν. ΜΟΝΑΠΗΡΙΟΥ 122, 563 34 ΘΕΣΣΑΛΟΝΙΚΗ, ΤΗΛ. 2310.70.63.54

Web site: http://www.patakis.gr - e-mail: [email protected], [email protected]

Page 3: Ευκλειδης Β 60

ΕΛΛΗΝΙΚΗ ΜΑθΗΜΑΤΙΚΗ ETAIPEIA Τεύχος 60 • Απρίλιος • Μάϊος • Ιούνιος 2006 • Ευρώ: 3,50

e-mail: [email protected] www.hms.gr

ΜΑΘΗΜΑ 11ΚΟ ΠΕΡΙΟΔJΚΟ ΠΑ το Λ ΥΚΕΙΟ

ΠΕΡΙΕΧΟΜΕΝΑ ./ Ιστορικές Μαθηματικές Αναφορές • yωνία της Ιστορίας των Μαθηματικών ... . 2 ./ Στοιχεία Μαθηματικής Λοyικής .................................. ...... 6 ./ Homo· Mathematicus ........................................................ 11 ./ Μαθηματικοί Διαγωνισμοί - Μαθηματικές Ολυμπιάδες ............................ 15

Μαθηματικά Α' Τάξης ./ Άλyεβρα

./ Γεωμετρία

..................................................... 17 ................................................ / ...... . . . . ..... 25

I

Μαθηματικά Β' Τάξης ./ Άλyεβρα

./ Γεωμετρία

./ Κατεύθυνση

.... . • .. . 31 .34

38

Μαθηματικά Γ' Τάξης ./ Μαθηματικά Κατεύθυνσης ...... . . 49

. 57 ./ Μαθηματικά Γενικής Παιδεία

./ Το Βήμα του Ευκλείδη . ..... . ............................... ...... 67 ./ Ο Ε υκλείδης προτείνει ... Ευκλείδη και ... Διόφαντο .............. 71 ./ Τα Μαθηματικά μας Διασκεδάζουν . . . . . . . . . . . . . . . . ............. 76

••••••••••••••••••••••••••••••••••••••••••••••••••••••••••••••••••••••••••••••

ΕΚΔΟΣΗ ΤΗΣ ΜΑΘΗΜΑΤΙΚΗΣ ΗΑΙΡΕΙΑΣ

ΠΑΝΕΠΙΠΗΜΙΟΥ 34- 106 79 ΑΘΗΝΑ Τηλ.: 21 Ο 3617784 " 3616532 Fax:2103641025 Εκδότης: Εξαρχ6κος Θεόδωρος Διευθυντής: Τυρλής Ιω6ννης

Κωδικός ΕΛ.ΤΑ.: 2055 ISSN: 1105- 7998

Επιμέλεια 'Εκδοσης: Κυριακόπουλος Αντώνης Ευσταθίου Βαγγέλης

Εκτελεστική Γραμματεία

Πρόεδρος: Κυριακόπουλος Αντώνης

Αντιπρόεδροι: Α": Eυcnαtfoυ Βαγγcλης

Β": Ταcrcrόπουλος Γιώργος

Γραμματέας: Χριcnόπουλος Παναγιώτης

Μέλη: Αργυρ6κης Δ.

Δρούτcrας Π.

Λουρfδας Σ. Ταπeινός Ν.

Σuνταντική επ ιτροπή Αθανασόπουλος Γεώργιος Ανδρουλακάκης Νίκος Αντωνόπουλος Νίκος Αργυράκης Δημήτριος Ατσαλόγλου Χρήστος Βακαλόπουλος Κώστας Βλάχου Αγγελική Γράψας Κων/νος Δρο'\ίτσας Παναγιώτης Ευσταθίου Βαγγέλης

Ζώτος Βαγγέλης Καλίκας Σταμάτης Καρακατσάνης Βασίλης Καρκάνης Βασίλης Κατσοvλης Γιώργος Κερασαρίδης Γιάννης Κηπουρός Χρήστος Κόvτζιας Νίκος

·Κυριακόπουλος Αντώνης Κυριακόπουλος Θανάσης Κυριακοποvλου­Κυβερνήτου Χρυστ .

Λαζαρίδης Χρήστος Λουρίδας Σωτήρης Μαλαφέκας Θανάσης Μεταξάς Νικόλαος Μυλωνάς Δημήτρης Μώκος Χρήστος

ΣαίτηΕvα Σταθόπουλος Γεώργιος Σταϊκος Κώστας Στάϊκος παναγιώτης Στρατής Γιάννης Ταπεινός Νικόλαος Τασσόπουλος Γιώργος Τριάvτος Γεώργιος Τσαγκάρης Ανδρέας Τσικαλουδάκης Γιώργος Τσιοvμας Θανάσης Τυρλής Ιωάννης; Χαραλαμπίδης Γιώργος Χαραλαμποποολου Λίνα Χαραλάμπους Θάνος Χριστόπουλος Παναγιώτης

••••••••••••••••••••••••••••••••••••••••••••••••••••••••••••••••••••••••••••••

KENWO fPAΦIΚfiN ΤΕΧΝΩΝ: ΔΙΗΝΕΚΕΣ Στοιχειοθεσία - Σελιδοποίηση: Ακαδημίας 43, Αθιivα τηλ.: 210 3606760,

fax.: 210 3606826 e-mail: [email protected]

Εκτύnωαη: ΙΝ7ΈΡΠΡΕΣ Α.Ε. τηλ.: 2 10 8 I 60330 Υπcιίθuνος τunογpαφeίοu: Β. Σωτηριάδης

• Τα διαφημιζόμενα βιβλία δε σημαίνει 6τι πρστείνοντ:αι απ6 την Ε.Μ.Ε. • Οι συνεργάτες, τα άρθρα, οι προτεινόμενες ασκήσεις, οι λοοεις ασκήσεων κτλ. πρέπει

να σtεΛ.νοντ:αι έγκαιρα, σαι γραφεία της Ε.Μ.Ε. με την ένδειξη "Για τον Ευκλείδη β"''. Τα χειeόγραφα δεν επωtρέφοντ:αι. Τιμή Τεύχους ευρώ 3,50 Ετήσια σιινδQομή (10;00 + 4,00 ΤαχυδQομικά: ευQώ 14,00)

Ετήσια συνδρομή για Σχολεία ευρώ 10,00

Το αντίτιμο για τα τε'ύχη ποιι παραγγεΛ.νονw.ι στεΛ.νεται με απλή επιταγή σε διαταγή Ε.Μ.Ε. Ταχ. Γραφείο Αθήνα 54 Τ.Θ. 30044 ή πληρώνεται σtα γραφεία της Ε.Μ.Ε.

Page 4: Ευκλειδης Β 60

Επιμέλεια: Χρήστος Κηπουρός

Το άρθρο που ακολουθεί μας έστειλε ο καθηγητής του Πανεπιστημίου Ιωαννίνων και, πρόεδρος της ΕΜΕ, κατά το παρελθόν, κ. Θόδωρος Μπόλης. Το περtεχόμενό του, που ανήκει στη θεωρία των αριθμών, αναφέρεται σε μια ειδική κα­τηγορία πρώτων αριθμών που ονομάζονται αριθμοί του Mersenne. AJJ..f:ι. δεν περιορίζεται μόνο σ' αυτούς. Μας δίνει πλη­θώρα ηλεκτρονικών διευθύνσεων εmστημονικών ομάδων που ασχολούνται μ' αυτούς τους αριθμούς καθώς, επίσης, και ενδιαφέρουσες ιστορικές πληροφορίες για τους Μαθημαnκούς που οδηγούμενοι από «θεία περιέργεια», μελέτησαν σε βάθος αυτούς τους αριθμούς αλλά και άλλους πρώτους. Επειδή η στήλη αυτή επιθυμεί και τη συμμετοχή των μαθητών του Λυκείου, που διαθέτουν Η/Υ, θα δώσουμε μερικά πληροφοριακά στοιχεία για να εισαγάγουμε mo απλά τους ανα­

γνώστες μας στο αντικείμενο του άρθρου. Οι πρώτοι αριθμοί, γνωστοί από την αρχαιότητα, έχουν μελετηθεί από τον Ευκλείδη,

τον Ερατοσθένη, τον Διόφαντο και άλλους .. Ο εντοπισμός τους έγινε συστηματικά από τον Ερατοσθένη με το περίφημο «κόσκινό του». Έκτοτε πολλοί μαθηματικοί προσπάθη­σαν να δώσουν μαθηματικό τύπο που να παρέχει τους πρώτους αριθμούς. Ωστόσο, όλες οι προσπάθειες απέτυχαν. Ο Fermat, πρώτος ισχυρίστηκε ότι οι αριθμοί της μορφής:

Ν= 22" + 1 είναι πρώτοι, για n=0,1,2,3,4. Ο Fermat και ο Toπicelli νόμιζαν ότι και οι δύο επόμενοι αριθμοί είναι πρώτοι.

Pieπe de Feπnat 1601- 1 665

Ο Euler, όμως, απέδειξε ότι είναι σύνθετοι. Έτσι οι Μαθηματικοί αναγκάστηκαν να μελετήσουν διάφορα υποσύνολα των πρώτων αριθμών που πληρούν κάποια ιδιότητα. Π.χ. οι αριθμοί του Mersenne (βλ.πιο κάτω), οι δίδυμοι πρώτοι αριθμοί(δηλ. οι πρώτοι που διαφέρουν κατά 2 μονάδες, βλ. Ευκλείδης Β' στο άρθρο Ερατοσθένης).

Και τώρα ακολουθεί το άρθρο του καθηγητή Θ. Μπόλη

Η ΓΩΝΙΑ ΤΗΣ ΙΣΤΟΡΙΑΣ ΤΩΝ ΜΑΘΗΜΑΤΙΚΩΝ

ΚΑΙ ΠΡΩΤΟΣ ΚΑΙ ΠΟΛ Υ ΜΕΓ ΑΛΟΣ

Στις 25 Δεκεμβρίου 2005 ανακοινώθηκε στο διαδύ­

κτιο, με ηλεκτρονικές διευθύνσεις:

(i) (http://www.mersenne.org/30402457.htm) και

(ii)(http://mathword. wolfram.co/news/2005-12-25/

mersenne-43) ότι οι συμμετέχοντες στο Πρόγραμμα

Great Intemet Mersenne Pήme Search (GIMPS,

www.mersenne.org) Curtis Cooper και Steνen Boone

του Central Missouή State Uniνersity, ανακάλυψαν (με

τη χρήση του τεστ Lucas- Lehmer) στις 15 Δεκεμβρίου

2005 ότι ο αριθμός 230.4°2.457 1 =

31541647561884608093 . . . 11134297411652943871

που έχει 9.152.052 δεκαδικά ψηφία, είναι ο 43ος γνω­

στός πρώτος του Mersenne και ο μεγαλύτερος γνω­

στός μέχρι σήμερα πρώτος αριθμός.

Οι αριθμοί του Mersenne είναι οι αριθμοί της μορφής:

Από τον Θόδωρο Μπόλη, Γιάννινα

(1)

όπου ο p είναι πρώτος αριθμός και φυσικά στη βάση

2, (δηλ.γραμμένος στο δυαδικό σύστημα αρίθμησης)

Mersenne Lucas

Το 1644, ο Γάλλος ιερομόναχος Marin Mersenne

(1588 - 1648) που ήταν ξακουστός επιστήμονας στην

ΕΥΚΛΕΙΔΗΣ Β' λθ' τ.4/2

Page 5: Ευκλειδης Β 60

------------- Ιστορικες Μαθηματικές Αναφορές -------------

εποχή του, ανακοίνωσε στο βιβλίο του Cogitata physica mathematica, ότι για p = 2, 3, 5, 7, 13, 17, 19, 31, 67,

127 και 257 ο αριθμός Μρ είναι πρώτος και για τους

υπόλοιπους 44 πρώτους p που είναι μικρότεροι του 257,

ο Μρ είναι σύνθετος. Τελικά αποδείχτηκε ότι αυτή η

ανακοίνωση περιείχε πέντε λάθη: Οι Μ67 και Μ257 είναι

σύνθετοι ενώ οι Μ61, Ms9 και Μ ιΟΊ είναι πρώτοι.

Ο Merseηηe ήταν φίλος του Καρτέσιου (Reηe

Descartes, 1596-1650) και του Fermat (Pieπe de Fer­

mat, 1601-1665), υποστηρικτής του Γαλιλαίου (Gali­

leo Galilei, 1564-1642) και του νεαρού τότε Ολλανδού

Φυσικού και Μαθηματικού Huygens (Christiaaη Huy­

geηs, 1629-1695).

Όπως αναφέρθηκε και παραπάνω, το πιο αποτελε­

σματικό μέσο που χρησιμοποιείται για να αποδείξει κα­

νείς ότι ένας αριθμός του Merseηηe είναι πρώτος, είναι

το τεστ των Lucas- Lehmer. Ο πρώτος, δηλ. ο Γάλλος

Μαθηματικός Franςois Edouard Anatole Lucas ( 1842

- 1891), ήταν ονομαστός για τη συμβολή του στα Δια­

σκεδαστικά Μαθηματικά (εισήγαγε το παζλ «Πύργος

του Χανόϊ») και προ παντός στο χώρο της Θεωρίας των

Αριθμών {ακολουθία Fibonacci (Leoηardo Pisaηo Fi­

boηacci, 1170 - 1250), ακολουθία Lucas, διαιρετότητα

διωνυμικών συντελεστών και το περίφημο τεστ του). Ο

Lucas, δυστυχώς, πέθανε σε ηλικία 49 ετών από επιπό­

λαιο τραύμα στο πρόσωπο που του προκάλεσε θανατη­

φόρο μόλυνση. Ο δεύτερος, Derrick Henry Lehmer

(1905 - 1991)

D. H. Lehmer D.N. Lehmer

Ήταν Αμερικανός Μαθηματικός και καθηγητής της

Θεωρίας Αριθμών στο Πανεπιστήμιο της Καλιφόρνιας

στο Berkeley. Ήταν γιος του Derrick Norman Lehmer

(1867 - 1938). που ήταν, επίσης, διάσημος Μαθηματι-

κός και καθηγητής στο ίδιο Πανεπιστήμιο.

Η σύζυγός του Emma Markovna Trotskaia Leh­

mer (1906 - ) είναι Ρωσικής καταγωγής (γεννήθηκε στη

Σαμάρα) και επίσης είναι διάσημη Μαθηματικός και

(τέως βέβαια ... ) συνεργάτις του συζύγου της (ο οποίος

ήταν απόγονος Γερμανών αποίκων της Πενσυλβανίας).

Εκτός της Θεωρίας Αριθμών, ασχολήθηκε και με τους

Ηλεκτρονικούς Υπολογιστές (ισχυρότατο εργαλείο για

τη Θεωρία Αριθμών). Αξιοσημείωτο γεγονός ήταν η

άρνησή του να υπογράψει τον «Όρκο Αφοσίωσης» που

επέβαλε η Μακαρθική aντικομουνιστική υστερία με

συνέπεια να χάσει τη θέση τού καθηγητή στο Berkeley.

Επανήλθε στη θέση του όταν τα δικαστήρια έκριναν ότι

η υποχρέωση της υπογραφής του «Όρκου» ήταν αντι­

συνταγματική.

Το τεστ Lucas - Lehmer (σύντομα LL) έχει ως ε­

ξής: Ορίζουμε την ακολουθία

(Sn) : Sn + ι = (S0)2- 2, Sz = 4, η = 2, 3, ...

Ο αριθμός του Mersenηe Μρ = 2Ρ - 1 για p > 2 είναι

πρώτος τότε και μόνο τότε αν ο αριθμός Sρ διαιρεί τον

Μρ. Βέβαια αρκεί ο υπολογισμός του Sn mod Μρ για η = 2, 3, ... , p.

Το Πρόγραμμα GIMPS παρέχει δωρεάν λογισμικό

για συμμετοχή στο κυνήγι των πρώτων του Merseηηe.

Νεαροί μαθητές έχουν ανακαλύψει τέτοιους πρώτους

στο παρελθόν.

Ο πίνακας που ακολουθεί δίνει την ιστορία της α­

νακάλυψης των πρώτων του Merseηηe (πρβλ. Εήc W.

Weissteiη, http://mathword. wolfram.com/ηews/2005-

12-25/merseηηe-43). Το σύμβολο ? σημαίνει ο αύξων

αριθμός είναι προσωρινός, π.χ. δεν έχουν ελεγχθεί ε­

παρκώς (έχει γίνει μόνο ένα LL) όλοι οι πρώτοι εκθέτες

μεταξύ του 6972593 που είναι ο εκθέτης για τον 38°

πρώτο του Merseηηe και του 13466917 που είναι ο εκ­

θέτης για τον επόμενο γνωστό πρώτο του Merseηηe

(τον 39?). Τα δύο ?? σημαίνουν ότι υπάρχουν εκθέτες

στο μεταξύ που δεν έχουν ελεγχθεί καθόλου ακόμα

Αριθμός ψη- Χρόνος Ανακά-α/α Εκθέτης ρ φ ίων λυψης Ποιος ανακάλυψε

1 2 1 Αρχαιότητα 2 3 1 Αρχαιότητα 3 5 2 Αρχαιότητα 4 7 3 Αρχαιότητα

ΕΥΚΛΕΙΔΗΣ Β' λθ ' τ.4/3

Page 6: Ευκλειδης Β 60

------------- Ιστορικες Μαθηματικές Αναφορές -------------

5 13 4

6 17 6

7 19 6

8 31 10

9 61 19

10 89 27

11 107 33

12 127 39

13 521 157

14 607 183

15 1279 386

16 2203 664

17 2281 687

18 3217 969

19 4253 1281

20 4423 1332

21 9689 2917

22 9941 2993

23 11213 3376

24 19937 6002

25 21701 6533

26 23209 6987

27 44497 13395

28 86243 25962

29 110503 33365

30 132049 39751

31 216091 65050

32 756839 227832

33 859433 258716

34 1257787 378632

35 1398269 420921

36 2976221 895832

37 3021377 909526

38? 6972593 2098960

39?? 13466917 4053946

40?? 20996011 6320430

41?? 24036583 7235733

42?? 25964951 7816230

43?? 30432457 9152052

Ένας θετικός ακέραιος λέγεται τέλειος αν ισούται με το άθροισμα των θετικών και μικρότερων του εαυ­τού του διαιρετών. Ο Ευκλείδης ο Αλεξανδρεύς (325; π.Χ.- 265; π.Χ.) απέδειξε άτην τελευταία πρόταση του ΙΧ βιβλίου των «Στοιχείων» ότι οι αριθμοί:

Τρ= 2Ρ-1Μρ (2) όπου Μρ πρώτος, είναι τέλειοι. Σε ένα αδημοσίευτο

χειρόγραφο, ο πολυγραφότατος Ελβετός Μαθηματικός Leonhard Euler (1707 - 1783) απέδειξε ότι ισχύει και το αντίστροφο. Κάθε άρτιος τέλειος αριθμός πρέπει να έχει την ανωτέρω μορφή (2). Έτσι, μέχρι στιγμής γνωρίζου-

1461 Regius, Cataldi

1588 Cataldi

1588 Cataldi

1750 Euler

1883 ΠepaywHH, Seelhof

1911 Powers

1913 Powers

1876 Lucas

30/1/1952 Robinson

30/1/1952 Robinson

25/611952 Robinson

7/10/1952 Robinson

9/10/1952 Robinson

8/9/1957 Riesel

3/11/1961 Hurwitz

3/11/1961 Hurwitz

11/5/1963 Gillies

16/5/1963 Gillies

2/6/1963 Gillies

4/3/1971 Tuckerman

30/10/1978 Noll & Nickel

9/2/1979 Noll

8/4/1979 Nelson & Slowinski

25/9/1982 Slowinski

28/1/1988 Colquitt & Welsh

20/9/1983 Slowinski

6/9/1985 Slowinski

19/2/1992 Slowinski & Gage

10/1/1994 Slowinski & Gage

3/9/1996 Slowinski & Gage

12/11/1996 Joel Armengaud/GIMPS

24/8/1997 Gordon Spence/GIMPS

27/1/1998 Roland Clarkson/GIMPS

1/6/1999 Nayan Hajratwala/GIMPS

14/11/2001 Michael Cameron/GIMPS

17/11/2003 Michael Shafer/GIMPS

15/5/2004 Josh Findley/GIMPS

18/2/2005 Mai'tin Nowak/GIMPS

15/12/2005 C. Cooper & S. Boone/GIMPS

με 43 άρτιους τέλειους αριθμούς: τ 2 = 6, τ 3 = 28, ... ' Τ30402457· Πόσα δεκαδικά ψηφία έχει ο τελευταίος και καινούρ-yιος τέλειος αριθμός;

Δεν ξέρουμε αν υπάρχουν άπειροι άρτιοι τέλειοι αριθμοί (ισοδύναμα άπειροι πρώτοι του Mersenne). Επίσης, είναι άγνωστο αν υπάρχει περιττός τέλειος αριθμός. Ξέρουμε όμως ότι, αν υπάρχει, θα πρέπει να είναι υπερβολικά μεyάλος: Σήμερα είναι γνωστό ότι θα πρέπει να έχει τουλάχιστον 300 ψηφία, να έχει του­λάχιστον 75 πρώτους παράγοντες και οι τρεις μεγαλύ­τεροι πρώτοι παράγοντες θα πρέπει να είναι τουλάχι-

ΕΥΚΛΕΙΔΗΣ Β' λθ' τ.4/4

Page 7: Ευκλειδης Β 60

------------- Ιστορικες Μαθηματικές Αναφορές -------------

στον 10000019, 10007 και101 (βλ. Eric W. Weisstein et al., 'Όdd Perfect Numbers",) http://mathworld.wo1fram.com/OddPerfectNumber.html).

Στην ιστοσελίδα http://pήmes.utm.edu του καθηγη­τή Chήs Ca1dwell, Uniνersity of Tennessee at Martin,

υπάρχει πληθώρα πληροφοριών για τους πρώτους αριθ­μούς καθώς τα πιο πρόσφατα <<ρεκόρ των πρώτων», π. χ. οι Ι Ο μεγαλύτεροι γνωστοί πρώτοι σήμερα δίνονται από τον ακόλουθο πίνακα:

[ΞΞ]I πρώτος 11 ψηφία 10�1 Αναφορά I

[�] 230402457-11191520521012005 IIMersenne 43??1 0

1 225964951-111 78162301012005 IIMersenne 42?? 01 224036583-111 7235733 1012

004 IIMersenne 41 ??I 01 01

220996011-11163204301012003 IIMersenne 40?? 213466917-1 1 1 4053940 1012001 IIMersenne 39?

I .Q 1127653·29167433+11127596771012005 IIGordon I

lz 1128433·27830457+111 23572071012004 ΙΙΑτ Prime Rib I

ω ι 26972593-1Ιι 2098960 IO�ιMersenne 38 I ι 2 11 5359"25054502+1Ιι 1521561Ιοι2003 ιιsandquist

I ι 1 0 11 4847"23321063+1Ιι 999744Ιοι2005 ΙιHassler I

η μάς χρειάζονται οι μεγάλοι πρώτοι αριθμοί; Ο μεγάλος Άγγλος Μαθηματικός Godfrey Harold

Hardy (1877 - 1947) είχε προφητέψει ότι ο μόνος κλά­

δος των μαθηματικών που δεν θα έχει ποτέ καμμία χρη­

σιμότητα είναι η Θεωρία Αριθμών. Έπεσε πολύ έξω.

Σήμερα οι μεγάλοι πρώτοι αριθμοί χρησιμοποιούνται

πάρα πολύ ουσιουδώς στην Κρυπτογραφία. Για αυτό θα

ασχοληθούμε σε άλλο σημείωμα.

Ο καλύτερος ιστιότοπος για την Ιστορία των Μα­

θηματικών είναι ο ιστιότοπος του Πανεπιστημίου του

St. Andrews της Σκωτίας :

http://www-histoιy.mcs.st-and.ac.uk/-histoιy . Από

εκεί προέρχονται όλες οι φωτογραφίες και πολλές από

τις πληροφορίες του παρόντος.

Γιάννινα, 5 Φεβρουαρίου 2006.

Ο, Μικρ:ός, Ε.υικλε\δηiς. Ο μικρός Ευκλείδης είναι το απαραίτητο βήμα των μαθητών και των δασκάλων του Δ ημοτικού Σ χολείου

στα μαθηματικά. Είναι ένα εργαλείο για την προσέγγ ιση ··-- _ •e των Μ αθηματικών με νέες ιδέες και ευρηματι κότητα με

Μ_7- � " " στόχο να αγαπήσουν οι μικροί μαθητές τα Μ αθηματικά. �-:\.�� 'Εχει σταυρόλεξα, παιχνίδια, ωr).f;;, ιστορlΟύλες, ωr)..fJ. μα-, . ...::; ________ .. --- '" ·-�ι

δ � θήματα και εν διαφέρουσες ασκήσεις, που καλλιεργούν και οξύ -ι• 'Ι! νουν τη σκέψη, εικόνες και σχέδια και πα ραμύθια και ποιήμα-

' tι 1, '' τα, που με καλαισθησία και χαρά οδηγούν τον υπέροχο κό­

σμο των Μαθηματικών. Ο Μικρός Ευκλείδης Σ ελίδες 32 ISSN: 1790-2029 Σ χήμα 21Χ29 Τιμή τεύχους 3 €

Ατομική ετήσια συνδt]ομή 12 € (4 τεύχη)- Ομαδικές ετήσιες συνδt]ομές σχολείων 8 €

ΕΥΚΛΕΙΔΗΣ Β ' λθ ' τ.4/5

Page 8: Ευκλειδης Β 60

"Τα Μαθηματικά θεμελιώνονται, κατανοούνται και αναπτύσσονται με τη βοήθεια της Μαθηματικής Λογικής"

του Αντώνη Κυριακόπουλου

(Σ υνέχεια από το προηγούμενο τεύχος)

3. ΜΕΘΟΔΟΙ ΑΠΟΔΕΙΞΗΣ ΚΑΙ ΕΥΡΕΣΗΣ ΣΤΑ ΜΑΘΗΜΑΤΙΚΆ

Σ τα προβλήματα των Μ αθηματικών συνήθως ή ζητάμε να αποδείξουμε μία πρόταση ή ζητάμε να βρούμε ένα ή περισσότερα μαθηματικά αντικείμε­να (αριθμούς, διανύσματα, συναρτήσεις κτλ.).

Α. ΑΠΟΔΕΙΞΗ ΠΡΟΤ ΑΣΗΣ 3.1. Η ΕΝΝΟΙΑ ΤΗΣ ΑΠΟΔΕΙΞΗΣ ΣΤΑ

ΜΑΘΗΜΑΤΙΚΆ

Σ ε μία Μ αθηματική Θ εωρία όταν λέμε ότι θα αποδείξουμε μία πρόταση q, εννοούμε ότι: από τα αξιώματα της θεωρίας και με τη βοήθεια των νό­μων και των κανόνων της Μαθηματικής Λογικής θα συμπεράνουμε την αλήθεια της q. Με άλλα λό­για ενν οούμε ότι: αν Ρι, pz, ... Pv είναι τα αξιώ μα­τα της θεωρίας, θα συμπεράνουμε την αλήθεια της συνεπαγωγής:

(Ρι και pz και ... και Pv) => q. Κ αι επειδή το πρώτο μέλος είναι μία πρόταση

αληθής, με βάση τον κανόνα αποσπάσεως, συ μπε­ραίνουμε την αλήθεια της q. Υπενθυμίζουμε τον κανόνα αποσπάσεως:

Κανόνας Προσοχή!

αποσπάσεως p =>q α

p =>q α q α

Ρ α p ; q α

Συνήθως, στην απόδειξη μιας πρότασης δε χρειάζεται να χρησιμοποιήσουμε όλα τα αξιώματα

της θεωρίας. Επίσης, για τη συντόμευση των απο­δείξεων, χρησιμοποιούμε σ' αυτές και άλλες προ­τάσεις (εκτός από τα αξιώματα), που έχουν απο­δειχθεί προηγουμένως (προηγούμενα θεωρήματα).

Για να αποδείξουμε μία πρόταση ω εργαζόμα­στε συνήθως με έναν από τους παρακάτω τρόπους. 3.2. ΜΕΘΟΔΟΣ ΤΩΝ ΣΥΝΕΠΑΓΩΓΩΝ

Κ ατασκευάζουμε μία (πεπερασμένη) διαδοχή αληθών προτάσεων της μορφής:

p => q, q => r, . .. , t => u, u =>ω, ή συντομότερα:

p => q => r => ... => t => u =>ω (1) όπου p είναι μία αληθής πρόταση της θεωρίας (α­ξίωμα ή θεώρημα ή υπόθεση). Τότε, η πρόταση p => ω είναι αληθής (κανόνας υποθετικού συλλογι­σμού) και επειδή η p είναι αληθής και η ω είναι αληθής (κανόνας αποσπάσεως). • Με άλλα λόγια:

I Ξεκινάμε από μία αληθή πρόταση p και με συνεπαγωγές ( =>) φθάνουμε

στην ω.

Παράδειγμα. Για ένα αριθμό α ε JR , ισχύουν:

-3:::;; α < 2 . Να δείξετε ότι:

l2α + 3 l <7.

Λύση. Έχουμε: -3::Ξ α< 2=>-6::Ξ 2α<4=>-3s; 2α+ 3<7 =>

- 7 < 2α + 3 < 7 => l2α + 3 1 < 7 .

3.3. ΜΕΘΟΔΟΣ ΤΟΥ "ΑΡΚΕΙ"

Πολλές φ ορές η έναρξη της απόδειξης μιας πρότασης ω με συνεπαγωγές, δεν είναι εύκολη. Τότε, συνήθως επιχειρούμε την κατασκευή της

ΕΥΚΛΕΙΔΗΣ Β' λθ ' τ.4/6

Page 9: Ευκλειδης Β 60

Στοιχεία Μαθηματικής Λογικής

διαδοχή ς των συνεπαγωγών αρχίζοντας από το τέλος, δηλαδή από την αποδεικτέα πρόταση ω. Τότε λέμε:

Για να δείξουμε την ω, αρκεί να δείξουμε την u (εννοούμε u � ω). αρκεί να δείξουμε την t (εννοούμεt � u).

κ.ο.κ. Προχωρούμε κατ' αυτόν τον τρόπο, έως ότ ου

φ θάσουμε σε μία αληθή πρόταση ρ της θεωρίας. Τότε έχουμε:

ω αρκεί u, αρκεί t, ... , αρκεί ρ, δηλαδή :

ω� u � t � ... �ρ . (2) Η διαδοχή (2) είναι η διαδοχή (1) της προηγού­

μενης περίπτωσης, γραμμένη από το τέλος. • Με άλλα λόγια:

I Ξεκινάμε από την αποδεικτέα πρόταση ω και με αρκεί ( �) φθάνουμε σε μία

αληθή πρόταση ρ. Παράδειγμα. Για τους αριθμούς α, β, γ e R.

ισχύει: α + β + γ = 1. Να δείξετε

ότι: α2 + β2 + 2 (αβ + γ) =γ2 + 1.

Λύση. Για να δείξουμε τη ζητούμενη ισότητα, αρ­κεί να δείξουμε ότι:

α2 + β2 + 2αβ+ 2γ =γ2 + 1, αρκεί α2 + β2 + 2αβ = 1 + γ2 - 2γ,

αρκεί (α + β) 2 = ( 1- γ) 2 , αρκεί α + β = 1- γ , αρκεί α+ β + γ = 1 , ισχύει.

3.4. ΜΕΘΟΔΟΣ ΤΗΣ ΕΙΣ ΑΤΟΠΟ ΑΠΑ­

ΓΩΓΗΣ.

I Ξεκινάμε από την ω (όχι ω) και με συνε­

παγωγές ( �) φθάνουμε σ' ένα άτοπο.

Έτσι, η ω είναι ψευδής και άρα η ω είναι αληθής. Σ την πράξη λέμε: <<'Έστω ότι η ω δεν ισχύει».

Τότε θα ισχύ ει η ω , και με συνεπαγωγές φ θάνου­με σε άτοπο.

Β λέπουμε πόσο σημαντικό είναι να ξέρουμε να σχηματίζουμε την άρνη ση μιας πρότασης. 3.5. ΠΑΡΑΤΗΡΗΣΕΙΣ

α) Σ την απόδειξη της ω με συνεπαγωγές, όπως και στην απόδειξη με τη μέθοδο του " αρκεί", δεν χρη-

σιμοποιήθηκε καμία από τις aντίστροφ ες συνεπ α­γωγές, δηλαδή από τις συνεπαγωγές:

ω=> u => t => ... => r => q => p. (3) Έ τσι λοιπόν δεν μας ενδιαφ έρει αν οι συνεπα­

γωγές αυτές ισχύουν ή όχι, αφ ού δεν έχουν καμία σχέση με την κατασκευαζόμενη απόδειξη της ω.

• Κ αι αν μία ή περισσότερες από αυτές τις a­ντίστροφ ες συνεπαγωγές ισχύουν, τότε δεν θα το σημειώνουμε στη διαδοχή (2) ή (1) ;

Όχι, γιατί σκοπός μας δεν είναι να αναφ έρουμε όλα όσα ισχύουν στα Μ αθηματικά, αλλά από αυτά που ισχύουν, εκείνα που χρειάζονται για να κά­νουμε την απόδειξη της ω (αλλιώς θα έπρεπε να αναφ έρουμε και το θεώρημα του Πυθαγόρα ! !, για­τί η σχέση του με την κατασκευαζόμενη απόδειξη της ω δε διαφ έρει από εκείνη των aντίστροφων αυτών συνεπαγωγών). β) Αν, ξεκινώντας από την αποδεικτέα πρόταση ω και προχωρώντας με " αρκεί" φ θάσουμε σε μία ψ ευδή πρόταση ρ, τότε δεν μπορούμε να συμπερά­νουμε τίποτα για την αλήθεια της ω.

• Κ αι τι θα κάνουμε τότε; Θ α επαναλάβουμε την προσπάθειά μας με άλ­

λες προτάσεις, οι οποίες ενδεχομένως να μας οδη­γήσουν (με αρκεί) σε μία αληθή πρόταση ρ. γ) Ό ταν θέλουμε να αποδείξουμε μία πρόταση ω

είναι λάθος να λέμε: «'Έστω ότι ω ισχύει» (τότε δεν έχουμε να

αποδείξουμε τίποτα!) και μετά (με

συνεπαγωγές) να φθάνουμε σε μία αληθή

πρόταση ρ (διαδοχή (3)) και στη συνέχεια

να λέμε: «άρα, η ω ισχύει», γιατί η ω

υποθέσαμε ότι ισχύει ( φαύλος κύκλος).

Ακριβέστερα, τότε, θα έχουμε αποδείξει την αλήθεια των προτάσεων: ω � ρ και ρ, από τις οποίες συμπεραίνουμε την αλήθεια της ω, που, όπως είδαμε παραπάνω, είναι λάθος. Έ τσι λ οιπόν, τότε, όχι μόνο δεν έχουμε αποδείξει την αλήθεια της ω, αλλά οι συνεπαγωγές αυτές δεν έχουν κα­

μία σχέση με την απόδειξη της ω.

δ) Αν, ξεκινώντας από την αποδεικτέα πρόταση ω και προχωρώντας με (αληθείς) ισοδυναμίες φ θά­σουμε σε μία αληθή πρόταση ρ, δηλαδή :

ΕΥΚΛΕΙΔΗΣ Β ' λθ ' τ.417

Page 10: Ευκλειδης Β 60

Στοιχεία Μαθηματικής Λογικής

ω<::::> u<::::>t<::::> ... <::::> r<=> q<=> p (4)

τότε δεν θα έχουμε απ οδείξει την ω; Ν αι, αλλά, αν ένας έγραφ ε ενδιάμεσα το θεώρημα του Πυθαγό­ρα, ασφ αλώς θα το διαγράφ ατε, όχι γιατί δεν ισχύ­ει, αλλά γιατί είναι άσχετο με την απ όδειξη της ω. Για τον ίδιο λόγο θα π ρέπ ει να διαγράψ ετε και ό­λες τις συνεπ αγωγές π ου οδηγούν απ ό την ω στην p, δηλαδή τις συνεπ αγωγές:

ω:::::> u :::::> t :::::> ... :::::> r :::::> q :::::> p (5)

αφ ού, αν και είναι αληθείς, είναι άσχετες με την απ όδειξη της ω π ου κατασκευάζετε. Η απ όδειξη της ω είναι η διαδοχή :

ω<=u<=t<= ... <=r<= q<=p (6) όπ ου p αληθής.

Το χειρότερο είναι, όταν, για να απ οδείξει ένας την ω, π ροσπ αθεί να κατασκευάσει τη διαδοχή των ισοδυναμιών (4) και συ μβεί οι συνεπ αγωγές (6) (που αποτελούν την απόδειξη της ω) να ισχύουν και μία τουλάχιστον απ ό τις συνεπ αγωγές (5) (που

είναι άσχετες με την απόδειξη της ω) δεν ισχύ ει. Τότε, χωρίς λόγο, δεν θα κάνει την απόδειξη της

ω. Για π αράδειγμα, βλέπ ε π αραδείγματα πα ρα­γράφ ων 2.2. και 2.3. 3.6. ΑΠΟΔΕΙΞΗ ΣΥΝΕΠΑΓΩΓΗΣ

Έστω, ότι έχουμ ε να απ οδείξουμ ε μια συνεπ α­γωγή :

«Αν p, τότε q>> , συ μβολικά: p :::::> q.

Σ την π ερίπ τω­ση αυτή δεν έχου­με να απ οδείξουμε την αλήθεια της p, ούτε της q, αλλά την αλήθεια της π ρότασης: p :::::> q .

Ρ α α Υ Υ

q p=> q α α ν Υ α α Υ α

Απ ό τον ορισμ ό της συνεπ αγωγής (διπ λανός π ί­νακας), έχουμε:

• Α ν η p είναι ψευ δής, τότε η συνεπ αγωγή p :::::> q είναι αληθής.

• Α ν η p είναι αληθής, τότε η συνεπ αγωγή p :::::> q είναι αληθής, μόνο όταν και η q είναι αληθ ής (π ρώτη γραμ μή στον π αραπ άνω π ί­νακα).

Έτσι, αν η p είναι αλη θής, α ρκε ί να δε ίξουμε ότι και η q είναι α ληθής.

Στην π ράξη υπο θέτουμε ότι η p ε ίναι αλη θής και δείχνουμε την αλήθε ια της q. Την πε ρίπ τωση που η p είναι ψευδής δεν την αναφ έρουμε κα θόλου, όχι για τί δεν χρ ειάζεται, αλλά γιατί υπο τίθεται ότι όλοι γνωρί­ζουν ότι τότε η p :::::> q είνα ι αλη θής.

3.7. ΜΕθΟΔΟΣ ΤΗΣ ΑΝΤΙΣΤΡΟΦΟΑΝτΙ­

θΕΤΟΥ ΠΡΟΤΑΣΕΩΣ

Ο ι συνεπα γωγές p:::::>q κα ι q:::::> p

ονο μάζονται αντιστροφοαντίθετες. Ισχύει: (ρ:::::> q) <::::> (q :::::> p)

( αντιστροφοαν τίθετο ς νόμο ς) Έτσι, για να δε ίξουμε μία σ υνε πα γωγ ή p :::::> q ,

αρκεί να δείξουμε την q :::::> p , η οπο ία με ρικέ ς φ ορές είναι ευκολότε ρη από εκε ίνη τη ς p :::::> q . 3.8. ΑΠΟΔΕΙΞΗ ΙΣΟΔ ΥΝΑΜΙΑΣ: p <::::> q 1 ος τρόπος. Δ είχνουμε ότι: p :::::> q κα ι q :::::> p . 2ος τρόπος. Κ ατασκευάζουμε μία δι αδο χή ( αλη-

θών) ισοδυνα μιών της μο ρφής: p<::::>r<::::>t<::::> ... <::::>u<::::>q

ή της μο ρφή: q<::::> u<::::> ... <::::>r<::::> p.

3°ς τρόπος. Δε ίχνουμε ότι p <::::> r και q <::::> r .

3.9. ΑΠΟΔΕΙΞΗ ΠΡΟΤΆΣΕΩΝ τΗΣ ΜΟΡΦΗΣ: <<ΓΙΑ ΚΑθΕ χ Ε Ω, p( χ) »

Έστω, ότι έχουμε να α ποδε ίξουμε μία πρότα ση τη ς π αρα πάνω μο ρφής, συμβολ ικά:

« 'dx ε Ω, ρ( χ)» όπ ου p(x) είναι ένα ς προ τα σια κός τύπο ς ο ρισμένο ς επ ί ενός συνόλου Ω. Τότε:

I Θεωροίιμε ένα (τυχαίο, όχι συγκεκριμέ­νο) στοιχΕίο ξ του Ω και δείχνουμε ότι η πριΊταση ρ(ξ) r;ίναι αληθής.

Συνήθως, αντί για το γράμμα ξ χρησ ιμοποιούμε πά-λι το γράμμα χ και αρχίζουμε την απόδειξη λέγοντας :

• I «Έστω ένα χ Ε Ω . Τότε κ τλ. Φ θάνουμε στην αλήθε ια τη ς p(x). Ειδικά όταν έχουμε να απ οδείξου με μ ία

π ρόταση της μορφής: ΕΥΚΛΕΙΔΗΣ Β' λθ ' τ.4/8

Page 11: Ευκλειδης Β 60

Στοιχεία Μαθηματικής Λογικής

«Γ ια κάθε φυσικό α ριθμό ν� ν0 ( ν0 Ε Ν), ρ( ν)», μπο ρούμε να εφα ρμόσουμε κα ι τη γνωστή μέθο δο της τέλεια ς επαγωγής.

- Είνα ι φανερό ότι για να α πο δείξουμε ότι μία πρότα ση της πα ραπάνω μο ρφής: « Vx Ε Ω, ρ (χ)» δεν ισχύει (δεν είνα ι αλ ηθής), α ρκεί να βρούμε μια τιμή χ =ξ Ε Ω για την οπο ία η πρότα­ση p(ξ) δεν ισχύ ει (δεν είνα ι α ληθής). Μ ια τέτο ια τιμή του χ λέγετα ι κα ι ένα aντιπαράδειγμα για την πρότα ση « '\/χ Ε Ω, ρ( χ)».

- Στο σημείο αυτό θα ήθελα να επισημάνω ένα πα ραλογισμό που τον ακούμε σuχνά από την τηλεόρα­ση, το ρα διόφωνο κα ι &xJ_ μόνο. Σε σ υζητήσεις, μεταξύ σο βα ρών κατά τ' άλλα ανθρώπων, θα έχετ ε ακο ύσει κάποιος να ισχυρίζετα ι ότι ισχύει κάτι γεν ικό, δηλαδή ότι ισχύει μία πρόταση τη ς μο ρφής: « Vx Ε Ω, ρ (χ)». Ένα ς όλλ.ος να βρίσκ ει ένα πα ράδειγμα που δεν ισχύει κα ι ο πρώτο ς να απαντά: «αυτό είναι μια εξαίρεση

που επιβεβαιώνει τον κανόνα!!!». Μεγαλύτερο ς πα­ραλογισμός δεν υπάρχει, αφού μία εξαίρ εση (ένα aντι­πα ράδειγμα), όπως είδαμε πα ραπάνω, είνα ι α ρκετή για να αναφέσει (κα ι &xJ_ να επιβεβαιώσει) ένα υποψήφιο κανόνα (ένα γενικό ισχυρισμό).

Β. ΕΥΡΕΣΗ ΜΑΘΗΜΑΤΙΚΟΥ ΑΝΤΙΚΕΙΜΕΝΟΥ

Όταν σ' ένα πρόβλημα ζητάμε να βρούμε ένα μα θημα τικό αντικείμενο, εργα ζόμα στε συνήθως μ ε έναν α πό τους πα ρα κάτω δύο τρόπους. 3. 10. ΜΕΘΟΔΟΣ ΤΩΝ ΙΣΟΔ ΥΝΑΜΙΩΝ

Βρίσκουμε τις αναγκαίες και ικανές συν­

θήκες που οφείλει να πληροί το ζητούμενο

αντικείμενο και μετά με ισοδυναμίες βρί­

σκουμε το αντικείμενο αυτό (μπορεί να

βρούμε περισσότερα από ένα).

Παράδειγμα. Να βρείτε τους αριθμούς χ Ε IR , για τους οποίους ισχύει: .J2x + 5 = χ + 1.

Λύση. Έχουμε:

.J2x + 5 =χ+ 1 <=> χ+ 1 �Ο <=> {2χ+ 5�0

2χ + 5 = (χ+ 1 )2

{χ + 1 �ο {χ � -1 <=> 2 <=> 2χ + 5 = (χ + 1) 2χ + 5 = χ 2 + 2χ + 1

<=> {χ�-1<=> {χ� -1 <::>χ= 2. χ2=4 (χ= 2 ή χ= - 2)

3.11. ΜΕΘΟΔΟΣ: <<ΕΥθΥ-ΑΝτΙΣΊ'ΡΟΦΟ».

α) Υποθέτουμε ότι ένα αντικείμενο (α­

ριθμός, διάνυσμα, συνάρτηση κτλ.)

πληροί τις δοσμένες συνθήκες και με

συνεπαγωγές το βρίσκουμε (μπορεί

να βρούμε περισσότερα από ένα).

Τονίζουμε ότι κατ' αυτ όν τ ον τ ρόπ ο από πουθενά

δεν εξασφαλίζεται ότι το αντικείμενο που βρήκα­

με πληροί τις δοσμένες συνθήκες. Γι' αυτό είναι απαραίτητο να εξετ άσουμ ε και τ ο αντ ίστ ροφ ο.

I β) Α ντιστρόφως. Εξετάζουμε αν το α­

ντικείμενο που βρήκαμε πληροί τις

δοσμένες συνθήκες.

Α ν το αντικείμενο π ου βρήκαμ ε π λη ροί τι ς δο­σμένες συνθήκες, τ ότ ε αυτ ό είναι τ ο ζητ ούμ ενο. Α ν όχι, τότε τέτοιο αντι κείμ ενο δεν υπ άρχει. Εννο­είται ότι αν βρούμ ε π ερι σσότ ερα απ ό ένα αντι κεί­μενα, τότε θα εξετ άσουμ ε τ ο καθένα χωρι στ ά, αν π λη ροί τις δοσμ ένες συνθήκες και θα κρατήσουμ ε εκείνα π ου τις π λη ρούν (αν υπ άρχουν). Παράδειγμα. Να βρείτε τους αριθμούς λ ε IR ,

για τους οποίους ισχύει:

Jίm ΧΖ + λχ - 2λ = 5 • (7)

Χ--+1 Χ - 1

Λύση. α) Έστω ότι γι α ένα αρι θμ ό λ Ε IR η (7) ισχύ ει. Τότε, απ ό αυτ ή και επ ει δή lim(x -1) =Ο,

χ--+1

έπ ετ αι ότι: 1. χ 2 + λχ - 2λ ( 1) - 5 ο ιm · χ - - · � Χ--+1 Χ -1

lim(x2 + λχ - 2λ) =0� 1 + λ - 2λ = 0� λ= 1. χ --+1

β) Α ντιστρόφως. Έστω ότι λ = 1. Τότ ε: 1. χ2+ λχ - 2λ 1. χ2+ χ - 2 ιm ιm----χ --+1 χ -1 χ--+1 χ -1

1. (χ -1) (χ + 2) ι· ( 2) 3 5 = ιm = ιm χ + = =F- • Χ--+1 Χ -1 Χ--+1

Άρα, τέτο ιο ς αρι θμ ός λ δεν υπ άρχει.

ΕΥΚΛΕΙΔΗΣ Β' λθ' τ.419

Page 12: Ευκλειδης Β 60

Στοιχεία Μαθηματικής Λογικής

3.12 ΛΑΘΗ ΠΟΥ ΠΙΘΑΝΟΝ ΝΑ ΚΑΝΟΥΜΕ

ΟΤΑΝ ΔΕΝ ΓΡΑΦΟΥΜΕ ΤΟΥΣ ΠΟ­

ΣΟΔΕΙΚΤΕΣ

Πολλές φορές, σε μία λύση, δεν γράφουμε το υς ποσοδείκτ ες, όχι γιατί δεν χρειά ζονται, αλλά γιατί έτσι νομίζουμε ότι απλοποιούμε τα θέματα. Σ την ουσία όμως δυσκολεύουμε π ερισσότερο τα π ράγματα, γιατί τότε π ρέπει να τους υπονοούμε. Τώρα, αν δεν τους γράφουμε και ο ύτε το υς υπονοού με (εκεί π ου π ρέπ ει), τότε όχι μόνο αρχίζουν οι ασάφειε ς, αλλά είναι σίγου­ρο ότι θα κάνουμε και λάθη. Για παράδειγμα:

Έστω ότι ζητάμε να βρούμε πόσες συναρτήσεις

υπάρχουν f : JR --+ JR , για τις οποίες ισχύει:

f2 (x) = χ2 + 1, για κάθε χ ε JR. • Ένας ίσως να έλεγε: Έχουμε: f2(x) = χ2 + 1 <::> )f2(x) = .Jx2 + 1 <::>

<=>lf(x)l=� <::::>

[ f (χ) = .J χ 2 + 1 ή f (χ) = -.J χ 2 + 1 J Άρα, υπ άρχουν δύο τέτοιες συναρτήσεις, οι εξή ς:

f(x) = .Jx2 + 1 και f(x) = -.Jx2 + 1. Το θέμα δεν είναι να τον π είσουμε ότι έχει κά­

νει λάθος και ότι υπ άρχουν και άλλες (άπ ειρες) τέτοιες συναρτήσεις, όπως π. χ. η εξής:

f(x) = {.Jx2 + 1 , αν χ� Ο -.Jx2+1 , αν χ<Ο.

Το θέμα είναι να δο ύμε γιατί κατέληξε σ' αυτό το λάθος. Και κατέληξε σ' αυτό το λάθος γιατί δεν έγ ραψε και ούτε υπονοούσε τους ποσοδείκτες. Δηλαδή αντιμε­τώπισε τις συναρτή σεις σαν πραγματικούς αριθμούς, που δεν είναι Το σωστό είναι να εργαστεί ως εξή ς:

[ Vx Ε JR ,f2 (x) = χ2 + 1 J <::::>

[ Vx Ε JR ,�f2 (x) = .Jx2 + 1 J <::::>

[ Vx Ε JR ,If(x)l = .Jx2 + 1 J <::::>

[ Vx Ε JR ,( f(x) = .Jx2 + 1 ή f(x) = -.Jx2 + 1)]. Σ' αυτό το σημείο θα έπρεπε να ξέρει, απ ό τη Μα­

θηματική Λογ ική (νόμοι π οσοδεικτών) ότι η τελευταία π ρόταση δεν είναι ισ οδύναμη με την π ρόταση : [( Vx E�f(x) =.Jx2 + 1) ή ( Vx E�f(x) =�) J.

Έτσι, δεν θα έφ θανε στο π αραπ άνω λανθασμέ­νο συμπ έρασμα.

Η απ άντηση βέβαια είναι ότι υπ άρχουν άπ ειρες τέτοιες συναρτήσεις. 3.13. ΕΠΙΛΟΓΟΣ

Τελειώνοντας θα ήθελα να π ω ότι δεν ισχυρί­ζομαι ότι, αν ένας ξέρει όλα αυτά απ ό τη Μ αθημα­τική Λ ογική, θα λύνει όλα τα π ροβλήματα. Ισχυρί­ζομαι όμως, ότι, τότε, θα γνωρίζει σίγουρους τρό­π ους για να εργαστεί, δεν θα οδηγείται σε εσφ αλ­μένες λύσεις και το σπ ουδαιότερο θα μπορεί να

ελέγχει αν μία λύση είναι σωστή ή όχι.

ΒΙΒΛΙΟΓΡΑΦΙΑ

1. Bittinger, Μ.: «Logic and proof».

2. Bochenskί, J.: «Α precis of Mathematical Logic».

3. Cήstian, R.: «lntroduction to Logic and Sets».

4. Copi, 1.: «Sympolic Logic».

5. Dopp, J.: «Notions de Logique formelle».

6. Exner - Rosskopf: «Logic in Elementary

Mathematics».

7. Kleene, S.: «Logique Mathematique».

8. Lightstone, Α.: «The Axiomatic Method - An

Introduction to Mathematical Logic».

9. Lorenzen, Ρ.: «Einfϋhrung ίο die operative Logik

und Mathematik)).

10.Lyndon, R.: «Notes on Logic)).

11. Mendelson, Ε.: «lntroduction to Mathematical

Logic)).

12. Novikov, Ρ.: <<lntrαluction a la logique mathematique)).

13. Reichenbach, Η.: <<Elements of Symbolic Logic)).

14. Robinson, G.: «An Intrαluction to Mathematical

Logic)).

15.Rosenbloom, Ρ.: «The elements of Mathematical

logic)).

16. StoU, R.: «Sets, Logic and Axiomatic Theoήes)).

17.Suppes, Ρ.: «lntroduction to logic)).

18. Tarski, Α.: «lntroduction to Logic and to the

methodology of deductive sciences)).

19. Thomas, Ν.: «Modem Logic)).

20.G. Polya: «Πώ ς να το λύ σ@). 21. Περιοδικό: «ΤΗΕ AMERICAN MATHEMAτi­

CAL MONTHLY)).

22.Α. Κυριακόπουλου: «ΜΑΘΗΜΑΠΚΗ ΛΟΓΙΚΗ)). 23.Α. Κυριακόπουλου: «Πώς θα το αποδείξω; ...

Πώ ς θα το βρω;)), άρ θρ ο στο περ ιοδικό «ΕΥ­ΚΛΕΙΔΗΣ Β)) (1999, τεύχος 33, σελ. 11).

ΕΥΚΛΕΙΔΗΣ Β' λθ' τ.4/10

Page 13: Ευκλειδης Β 60

HIJMIJ MATHEMATICVS Η Homo Mathematicus είναι μια στήλη στο περιοδικό μας, με σκοπό την ανταλλαγή απόψεων και την ανάπτυξη προβληματι­σμού πάνω στα εξής θέματα: 1 ) Τι είναι τα Μαθηματικά, 2) Πρέπει ή όχι να διδάσκονται, 3) Ποιοι είναι οι κλάδοι των Μαθημα­τικών και ποιο το αντικείμενο του καθενός, 4) Ποιες είναι οι εφαρμογές τους, 5) Ποιες επιστήμες ή κλάδοι επιστημών απαιτούν καλή γνώση των Μαθηματικών για να μπορέσει κάποιος να τους σπουδάσει. Για τους συνεργάτες της στήλης: παράκληση! τα κείμενα της στήλης αυτής, ως προς το πεpιεχόμενό τους και ως προς το επί­πεδό τους, θα πρέπει να είναι συμβιβαστά με τα ενδιαφέροντα και το επίπεδο κατανόησης από μέρους των παιδιών.

Προς τους φίλους της στήλης αγαπητοί φίλοι, τον τελευταίο καιρό έφ τασαν στους υπ εύθυνους της στήλης π ολλές εργασίες, η κάθε

μια απ ό τις οπ οίες έχει τα ιδιαίτερα χαρακτηριστικά της. Κ ατά τη γνώμη της στήλης δεν υπ άρχουν " κα­λές" ή " κακές" εργασίες, αλλά εργασίες με διαφ ορετικά ενδιαφ έροντα. Ό μως, λόγω του π λήθους, του ό­γκου τους και των διαθέσιμων σελίδων της στήλης, δεν είναι δυνατό να δημοσιευθούν όλες μαζί και ολο­κληρωμένες. Για το λόγο αυτό απ οφ ασίσαμε να δημοσιεύσουμε τις π ολυσέλιδες εργασίες ολόκληρες μεν, σε συνέχειες δε. Σ ας ευχαριστούμε για την κατανόησή σας. Γιάννης Κερασαρίδης

Ια. "αυτό το ξέρατε,· " Μήπως γνωρίζετε σε ποιον ανήκει η απάντηση στην πα­ρακάτω στιχομυθία;

« Δημοσιογράφος: Είπατε πως δουλεύεται με μαθη­ματικούς τύπους και αντίστοιχα μηχανήματα. Αυτά θα δίνουν μια πρώτη ύλη υποθέτω, πάνω στην οποία επεμ­βαίνετε εσείς, ή όχι; Ή αυτό το αποτέλεσμα μας δίνει το μηχάνημα είναι το τελικό; Δεν είναι το τελικό; Επεμβαί­νετε κι εσείς; Πως επεμβαίνετε; Απάντηση: Αυτό που σας είπα εγώ, είναι ορισμένες κα­τευθυντήριες γραμμές και νευρώσεις στην συνθετική σκέψη, που καταλήγουνε σε αποτελέσματα, χρησιμο-

11. ''οι συνεργάτες της στήλης γράφουν-ερωτούν''

ποιήσιμα εντελώς ή όχι εντελώς. Υπάρχει μια σειρά έρ­γων που βασίζεται σ' αυτούς τους τύπους, τους οποίους βρήκα, προσπαθώντας να κάνω σύνθεση και απαντώ­ντας σ' ορισμένα ερωτήματα. Η δουλειά μου είναι να βρω αυτούς τους τύπους, ν' ανακαλύψω, έστω, τύπους ( αλλά εγώ δεν ανακάλυψα, τους βρήκα στο δρόμο μου), να τους συναρμολογήσω, να κατευθύνω τα παραμετρικά

μεγέθη και τα συναρμολογιστικά, ώστε το αποτέλεσμα να είναι ενδιαφέρον . ... » [η απάντηση στο τέλος της στήλης]

1 ο θέμα: Εφαρμογές από τη «θεωρία του μαθηματικού μπιλιάρδου» Προλεγόμενα. Συνεχίζουμε με το δεύτερο μέρος της ενδιαφ έρουσας αυτής εργασίας. Σ το επ όμενο τεύχος θα δημοσιευθεί το τρίτο και τελευταίο μέρος.

«Τα Μαθηματικά και το "αεικίνητο"» (2° μέρος), από το Θάνο Λύπα Απόδειξη της οπτικής ιδιότητας

(σχήμα Ια) (σχήμα Ιβ) Απόδειξη της οπτικής ιδιότητας

Έστω 1 η εφ απ τόμενη της έλλειψης στο σημείο F1N +NF2> 2α=F1M+MF2 Μ (σχήμα 1) και έστω α και β οι γωνίες π ου σχη- Ό μως, απ ό κατασκευή, F1N= F1 'N και ματίζει η 1 με τα ευθύγραμμα FιΜ και F2M (Fι, Fz FιM=F1 'Μ, και κατά συνέπ εια έχουμε F1 'Ν+ NF2 οι εστίες). Θ εωρούμε το συμμετρικό σημείο F1 ' της > F1 'Μ+ MF2 ή εστίας Fι ως π ρος την 1, φ έρουμε το ευθύγραμμο F1 'M+MF2<F1 'F2 τμήμα F 1 'F2, και σημειώνουμε με Ν την τομή αυ- η οπ οία έρχεται σε αντίφ αση με την τριγωνική τού του τμήματος και τη ς 1. Έστω ότι Μ;#Ν. Τότε ανισότητα για το τρίγωνο F1 'F2M. Άρα τα σημεία; το Ν βρίσκεται έξω απ ό την έλλειψη (π ράγματι, Ν και Μ π ρέπ ει να συμπ ίπ τουν, και επ ομένως η ολόκληρη η 1, εκτός απ ό το σημείο Μ, βρίσκεται F2M F1 ' είναι ευθεία, δηλ., α= β. έξω απ ό την έλλειψη, διότι η έλλειψη είναι κυ ρτή καμπύλη), επ ομένως,

ΕΥΚΛΕΙΔΗΣ Β' λθ ' τ.4/11

Page 14: Ευκλειδης Β 60

ΗΟΜΟ ΜΑΤΗΕΜΑ τiCUS

Και τώρα το πείραμα! Τοποθετήστε δύο σώματα με ίση θε ρμοκρασία

στα σημε ία F 1,F2 (σχήμα 2) . Παρατηρήστε ότι ο­ποιαδήποτε ακτίνα ( φωτός ή θε ρμότητας) που ξε κι­νά από το F1 πρέπε ι να φ τάσε ι στο F2. Κ αι, φυσικά, οποιαδήποτε ακτίνα διαδίδε ται ακολουθώντας την ίδια τροχιά από το F2 θα φτάσε ι ξανά στο F 1 • Πα­ρατηρήστε, όμως, μια ακτίνα από το F2 προσκρούε ι στο κατακόρυφο τμήμα της κατασκευής μας. Α ν δεν υπήρχε αυτό το κατακόρυφο φράγμα κι αν δεν ε ίχαμε σβήσε ι το μισό μέρος της ε ξωτε ρικής έλλε ι­ψης, η ακτίνα αυτή θα ανακλάτο στο σβησμένο τμήμα και θα έφ τανε στο F 1 •

Α σχήμα 2

Επομένως, το σώμα στο F2 θα θε ρμαίνε ται ενώ το σώμα στο F1 θα ψύχε ται. Μπορούμε, ακόμη, και να υπολογίσουμε τις τε λικές θε ρμοκρασίε ς που α­ποκτούν τα σώματα με τά τη σταθε ροποίηση της δι­αδικασίας.

Υποθέτουμε ότι αρχική θε ρμοκρασία τους ε ίναι Τ 0 Κ (βαθμοί Kelvin, όπως ονομάζονται προς τιμήν του Άγγλου φυσικού William Thomson που ανα­φ έραμε προηγουμένως, και στον οποίο δόθηκε ο τίτλος του λόρδου Kelvin λόγω των ε ξαιρε τικών ε πιστημονικών ε πιτευγμάτων του). Είναι γνωστό ότι ο ρυθμός απώλε ιας ενέργε ιας δι ακτινοβολίας ε ίναι ανάλογος με την τέταρτη δύναμη της θε ρμο­κρασίας (σε βαθμούς Kelνin) του ακτινοβολούντος σώματος.

Επομένως αν θε ρμοκρασία στο σημε ίο F1 ε ίναι Τ1 και στο F2 ε ίναι Τ2 , τότε η ακτινοβολούμενη ι­σχύς στα Fι και F2 μπορε ί να γραφ τε ί kT1 4 και kT24, αντίστοιχα. Όλε ς οι ακτίνε ς που ξε κινούν από το F 1 φ τάνουν έπε ιτα από μια ανάκλαση στο F2 , αλλά μόνο ένα μέρο ς από τις ακτίνε ς που ξε κινούν από το F2 φτάνουν στο F ι. Πόσε ς ε ίναι αυτές; Θεωρή­στε μια μικρή σφαίρα με κέντρο στο F2. Ο ι ακτίνε ς που επιστρέφουν στο F2 aποκόπτουν ένα δακτύλιο της σφ αίρας (σχήμα 3) . Αν το ε μβαδόν του ε ίναι Α 1

Τώρα, όμως, θα ανακλαστε ί σε κάποιο σημε ίο R του κατακόρυφου φράγματος, και θ' ακολουθή­σε ι την πορε ία αν διαδιδόταν από το σημε ίο F1 προς το R (σχήμα 2) . Επομένως θα επιστρέψε ι στο F2 ! Κ αι ε ίναι φανε ρό ότι αν, ξε κινώντας από το F2, ακολουθήσε ι την ίδια πορε ία αλλά με την αντίθετη φορά, θα επιστρέψε ι και πάλι στο F2. Επομένως υπάρχε ι ένα σημαντικό πλήθος ακτίνων οι οποίε ς παγιδεύονται σε <<βρόγχους φωτός». Παρατηρώντας την εγκάρσια τομή της «μηχανής» μας στο σχ. 2, βλέπουμε ότι ε ίναι όλε ς οι ακτίνε ς που βρίσκονται μέσα στη γωνία AF2B και στη συμμε τρική της A 'F2B '.

σχήμα 3 και το συνολικό ε μβαδό της σφ αίρας ε ίναι Α, τότε το ποσό της ακτινοβολίας που ε πιστρέφε ι στο F2 ισούται με A1k Τ24/Α και το μέρος τη ς που φτάνε ι στο Fι ε ίναι (Α- A1) k Τ24/Α.

Σε κατάσταση ισορροπίας, η ενέργε ια που α­κτινοβολε ίται σε ορισμένο χρονικό διάστημα από το σημε ίο F 1 ισούται με την ενέργε ια που φτάνε ι σ' αυτό στο ίδιο χρονικό διάστημα. Δ ηλ.

(Α- Αι) k Τ24/Α= kT1 4 Από την άλλη πλευρά, βάσε ι του νόμου διατήρη­

σης της ενέργε ιας, έχουμε ότι η απώλε ια ενέργε ιας στο F 1 ισούται με την αύξηση της ενέργε ιας στο F2 [δηλ. C(τι-Το) =C(τ0-Τ2), όπου C ε ίναι η θε ρμο­χωρητικότητα των δύο σωμάτων - υποθέτουμε ότι τα δύο σώματα ε ίναι απολύτως όμοια]. Η τε λευταία ε ξίσωση μας . δίνε ι τ I +τ 2=2Τ Ο· Χρησιμοποιώντας την προηγούμενη σχέση και θέτοντας

b= [(Α-Αι) /Α]1 14 (και παρατηρώντας ότι b<l) παίρνουμε τε λικά Τι =[(2Τοb) /(1 +b) ]<To, Τ2=[(2Το) /(1 +b) ]>To

Καταρρίψαμε, λοιπόν, το Δεύτερο Νόμο της Θερμοδυναμικής; Δυστυχώς (ή ευτυχώς) όχι. Έ­χουμε κάνε ι ένα λάθος. Που, όμως; (η συνέχεια στο επόμενο)

2° θέμα: Οι μέλισσες σπουδάζουν Μαθηματικά και Αρχιτεκτονική Προλεγόμενα. Πρόκε ιται για μια ενδιαφέρουσα ε ργασία, διατυπωμένη με γλαφυρή γλώσσα. Η Φύ­

ση, δηλ. η υπαρκτή πραγματικότητα, ε ίναι ο δάσκαλός μας και, ταυτόχρονα, το ασφ αλές κριτήριο για την αλήθε ια ή όχι των όποιων ιδε ών μας. Απολαύστε την.

«Το κυνήγι της βέλτιστης λύσης», του Παναγιώτη Τριπικέλη (Κόρινθος) Εισαγωγή . Ένα από τα πολλά χαρακτηριστικά : λυση ενός μαθηματικού ( και όχι μόνο) προβλήματος

που μεγαλώνουν το ενδιαφέρον σε σχέση με την επί- ! είναι το γεγονός ότι μεταξύ των πολλών πιθανών

ΕΥΚΛΕΙΔΗΣ Β ' λθ ' τ.4/12

Page 15: Ευκλειδης Β 60

ΗΟΜΟ MATHEMAτiCUS

γνωρίζει τίποτα από Ευκλείδεια γεωμετρία. Από έν­σrικτο έχει δώσει τη βέλτιστη λύση σrο πρόβλημα: «να βρεθεί η ελάχισrη διαδρομή που ενώνει δυο ση­μεία σrο επίπεδο».

λύσεων προτιμάται η καλύτερη δυνατή ( όπως λέμε η βέλτιστη) λύση. Οι μαθηματικοί εισήγαγαν την έν­νοια της βέλτισrης λύσης υπακούοντας σε μια πρα­κτική η οποία εφαρμόζεται σrο φυσικό περιβάλλον. Ειδικότερα οι ζωντανοί οργανισμοί την αναζητούν από ένσrικτο. Ένα μωρό (για παράδειγμα) θα μπου­σουλίσει προς μια κατεύθυνση πάνω σrην ευθεία

Χαρακτηριστικότε ρο παράδε ιγμα ίσως αποτε­λε ί ο τρόπος με τον οποίο οι μέλισσε ς κατασκευά­ζουν τα κε λιά της κυψ έλης τους. Σ το τέλος του άρ­

που το ενώνει με το προορισμό του, χωρίς φυσικά να θρου θα διαπιστωθε ί ότι τα πε ρίφημα ε ξάγωνα που την αποτε λούν δεν κατασκευάστηκαν τυχαία.

Ι. Το πρόβλημα στις δυο διαστάσεις Δισδιάσrατο πολυγωνικό πλέγμα (σχήμα 4) είναι ένα σύνολο πολύγωνων σrο επίπεδο έτσι ώσrε κανένα

από αυτά δεν έχει ελεύθερη πλευρά εκτός από εκείνα τα πολύγωνα που ορίζουν το εξωτερικό σύνορο του. Ωσrόσο έχει παρατηρηθεί ότι αν ζητηθεί από κάποιον να σχεδιάσει ένα πλέγμα το πιο πιθανό είναι να κα­ταλήξει σrο παρακάτω σχήμα 5 :

Προφανώς το παραπάνω πλέγμα είναι μια ειδική περίπτωση του ορισμού , αφού αποτελείται από ίσα μεταξύ τους ορθογώνια και κατασκευάζεται πολύ πιο εύκολα από το αρχικό σχήμα όπου τα πολύγωνα είναι διαφορετικά μεταξύ τους.

Δ / ι v..L v v

I I I I I σχήμα 4 - πλέγμα σχήμα 5 - ορθογώνια παρ/μα, σχήμα 6 - κυψέλες

Αυτό σημαίνει ότι για να κατασκευάσει κάποιος ένα τέτοιο πλέγμα θα έπρεπε να έχει οδηγίες κατα­σκευής για κάθε ένα από τα πολύγωνα που το αποτε­λούν , ενώ στη περίπτωση του δεύτερου ο κατασκευ­ασrής χρειάζεται να ξέρει μόνο τις «προδιαγραφές» ενός από τα ορθογώνια και στη συνέχεια κατασκευ­άζει το πλέγμα επαναλαμβάνοντας το αρχικό ορθο­γώνιο.

Ας υποθέσουμε τώρα ότι ένα πλέγμα έχε ι κα­τασκευαστε ί για να αντέχε ι διάφ ορε ς δυνάμε ις που ενε ργούν πάνω του (πχ. το πλέγμα που αποτε λε ί μια πε ρίφ ραξη).

Σ τη πε ρίπτωση αυτή , ένας μηχανικός μπορε ί να ε πιβε βαιώσε ι , ότι το πλέγμα θα αντέχε ι πε ρισ­σότε ρο αν τα πολύγωνα (τα οποία θα ονομάζουμε «κε λιά») ε ίναι ίσα με ταξύ τους κανονικά πολύγω­να. Αυτό ε πιτρέπε ι στις δυνάμε ις που ασκούνται πάνω στην επιφ άνε ια του πλέγματος να κατανέμο­νται πιο ομοιόμορφ α με αποτέλε σμα το πλέγμα , συνολικά , να ε πιβαρύνε ται λιγότε ρο και ε πομένως να αντέχε ι πε ρισσότε ρο.

τίθε ται τώρα το ε ρώτημα: «ποσά τέτοια διαφ ο­ρε τικά πλέγματα υπάρχουν , για να διαλέξε ι ο κα­τασκευαστής». Ο αναγνώστης μπορε ί να αποδε ίξε ι

Αν ένα πλέγμα σrο επίπεδο αποτελείται από κελιά που είναι κανονικά πολύγωνα τότε αυτά θα είναι τετρά­γωνα η ισόπλευρα τρίγωνα η κανονικά εξάγωνα

Η απόδε ιξη βρίσκε ται στο 11 κεφ αλαίο του σχολικού βιβλίου της Γε ωμε τρίας όπου ο αναγνώ­στης μπορε ί να ανατρέξε ι.(Εννοε ίται βέβαια ότι η αναζήτηση πρέπε ι να πε ριλαμβάνε ι και τις ασκή­σε ις του κεφ αλαίου ! !)

Σ αν τε λευταίο ζητούμενο τίθε ται το ε μβαδόν του κάθε κε λιού. Γ ια κάποιο λόγο ο κατασκευα­

οποίο κατασκευάζε ι το πλέγμα ε ίναι ακριβό και γι αυτό εκ τός από αντοχή ζητάε ι με συγκε κριμένη και σταθε ρή ποσότητα υλικού το κάθε κε λί του πλέγ­ματος να έχε ι το μεγαλύτε ρο δυνατό ε μβαδόν.

Σ τις δυο διαστάσε ις αυτό σημαίνε ι, ασφ αλώς, ότι τα κε λιά έχουν την ίδια πε ρίμε τρο Η απάντηση βασίζε ται στις παρακάτω :

στής του πλέγματος ισχυ ρίζε ται ότι το υλικό με το Προτάσεις

Α) Ένα κανονικό ε ξάγωνο , ένα ισόπλευρο τρί- ι 2 ω Ε=- Ρ σφ -γωνο και ένα τε τράγωνο μπορούν να ε ίναι ισοπε ρι­με τρικά

Β) Αν Ρ η πε ρίμε τρος ενός κανονικού ν-γωνου , Ε το ε μβαδόν του , ω η κεντρική του γωνία τότε

4ν 2

Γ) Α ν ένα κανονικό ε ξάγωνο , ένα τε τράγωνο και ένα ισόπλευρο τρίγωνο έχουν την ίδια πε ρίμε τρο τότε μεγαλύτε ρο ε μβαδόν έχε ι το ε ξάγωνο

ΕΥΚΛΕΙΔΗΣ Β ' λθ ' τ.4/13

Page 16: Ευκλειδης Β 60

ΗΟΜΟ MATHEMAτiCUS

Εργονομία - Οικονομία Οι μέλισσες κατασκευάζουν τα κελιά τις κυψέ- • Η κατασκευή να μπορεί να γίνεται ταυτόχρονα

λης χρησιμοποιώντας κερί. Γι' αυτές αυτό το υλικό από πολλές μέλισσες ώστε να λιγοστεύει ο χρόνος είναι ακριβό καθώς για κάθε γραμμάριο κεριού α- κατασκευής. παιτείται κατανάλωση 7 γραμμάριων μελιού. Επι- • Με δεδομένο ότι για λόγους οικονομίας του πλέον είναι γνωστό ότι κάθε κελί είναι χώρος απο- κεριού το κάθε κελί κατασκευάζεται με την ίδια θήκευσης επομένως όταν κατασκευάζουν τα κελιά ποσότητα υλικού πρέπει να εξασφαλιστεί η μεγα-πρέπει να σιγουρευτούν για τα παρακάτω: λύτερη δυνατή χωρητικότητα

• Το κάθε κελί και επομένως συνολικά η κυψέλη Έχοντας λάβει υπόψη όλα τα παραπάνω ο ανα-αν έχουν τη μεγαλύτερη δυνατή αντοχή σης κατα- γνώστης διαπιστώνει ότι η κυψέλη πρέπει να είναι πονήσεις όπως στο (σχήμα κυψέλες) .

3° θέμα: Μια άγνωστη πλευρά του μεγάλου Γρηγόρη Ξενόπουλου Προλεγόμενα. Ο φίλος και συνεργάτης της στήλης μας, Γιάννης Σταμέλος, πάντα μας «ξαφνιάζει)) ευχά ριστα. Στο σημείωμά του αυτό αναδεικνύει μια άγνωστη, στους πολλούς Έλληνες, πλευρά του μεγάλου θεατρικού συγγραφέα, μυθιστοριογράφου και διευθυντή του περιοδικού «Διάπλαση Παίδων)) Γρηγόρη Ξενόπουλου. Αυτή του η πλευρά αφορά τη μαθηματική του ιδιότητα.

«Τα μαθηματικά στο έργο του Ξενόπουλου))' του Γιάννη Σταμέλου, (Λασίθι)

Την προηγούμενη σχολική χρονιά (2004-2005) Ο Ξενόπουλος έγραψε πολλά μυθιστορήματα, με τους μαθητές της τρίτης τάξης του 2ou Γυμνασί- διηγήματα, κριτικές και θεατρικά έργα. Είναι επί­ου Αγίου Νικολάου Λασιθίου είχαμε τη χαρά να σης ταυτισμένος με την περίφημη «Διάπλαση των παρακολουθήσουμε μια εξαιρετική παράσταση Παίδων)), η οποία γαλούχησε γενιές και γενιές. στην Αθήνα, στο θέατρο «Ακάδημορ), με πρωτα- Αρκετά έργα του Ξενόπουλου περιέχουν αυτο­γωνιστή τον Σπύρο Παπαδόπουλο. Το έργο ήταν βιογραφικά στοιχεία, όπως το «Δεν είμαι εγώ)). Οι του Γρηγορίου Ξενόπουλου και είχε τίτλο: «Δεν εί- συνεδριάσεις της Μαθηματικής Εταιρείας, της ο­μαι εγώ)) (το βιβλίο κυκλοφορεί από τις εκδόσεις ποίας υπήρξε μέλος (στο έργο αναφέρεται ως Επι­«Αδελφοί Βλάσση)) ). Ο πρωταγωνιστικός ρόλος, στημονική Εταιρεία) ο τύπος του Σωσύ, η χρήση τον οποίο ερμήνευε ο Παπαδόπουλος, είναι γραμ- συμβολισμών και εξισώσεων, η διατύπωση κανό­μένος για έναν καθηγητή μαθηματικών, τον Πέτρο νων, είναι μερικά στοιχεία που χρησιμοποιεί στο Παπαπέτρου. (Πέτρος Παπαχρήστου είναι ο κε- έργο του αυτό ο Ξενόπουλος . .Ά ναφορά επίσης γί­ντρικός ήρωας, επίσης μαθηματικός, στο διάσημο νεται και στον Νεύτωνα. πλέον έργο του Απόστολου Δοξιάδη «0 θείος Πέ- Το έργο «Δεν είμαι εγώ)) είναι φάρσα στην ο-τρος και η εικασία του Γκόλντμπαχ)) ) . ποία σατιρίζεται η Λογική και οι άνθρωποι που

Οι μαθητές εύκολα αναγνώρισαν στην ερμη- στηρίζονται μόνο σ' αυτήν. Η Λογική, σημειώνει νεία χαρακτηριστικά των καθηγητών τους και μά- σε σχετικό σημείωμά του, θα πρέπει να συνοδεύε­λιστα το σχολίασαν και στον Παπαδόπουλο, όταν ται κι από τη Φαντασία. «0 σωστός, ο φρόνιμος, ο πήγαν στο καμαρίνι του για να τον συγχαρούν. φυσιολογικός να πούμε άνθρωπος συνταιριάζει και

Ο Ξενόπουλος (1867-1951) σπούδασε μαθημα- τα δύο)) . τικά στο Πανεπιστήμιο Αθηνών και όπως ο ίδιος Ο καθηγητής στο έργο ανοίγει την καρδιά του έγραψε, οι σπουδές του επηρέασαν σημαντικά τόσο στους μαθητές του. Οι μαθητές του τον αγαπούν τη θεματογραφία του, όσο και το στυλ των γρα- και δηλώνουν ότι θα τον υπερασπιστούν αν χρεια­πτών του. «Μου αρέσει κάθε μου γραφτό, να είναι στεί. Χαρακτηρίζεται σεβαστός, σοβαρός, άγριος στη μορφή τόσο απλό, ώστε με την ίδια σχεδόν ά- αλλά και αφηρημένος, ξεχασιάρης και σχολαστι­νεση να μπορεί να το διαβάζει κι ένας μαθητής του κός. Εκδηλώνει έντονα την απορία του όταν οι άλ­Γυμνασίου και ο κύριος Παλαμάς. Δεν αγαπώ τα λοι δεν καταλαβαίνουν απλούστατα, κατά την αντί­σύννεφα και τα σκοτάδια · με τραβά περισσότερο η ληψή του, πράγματα. Παραδίδει μάλιστα (το 1915) ξαστεριά, το φως. Εσπούδασα μαθηματικά στα νιά- και ιδιαίτερα μαθήματα. τα μου, κι εσυνήθισα να λέγω, τα πράγματα απλά, Ο Ξενόπουλος βραβεύτηκε για το λογοτεχνικό καθαρά και ξάστερω), σημείωνε. του έργο, το 1929, από την Ακαδημία Αθηνών. Ιρ. «αυτό το ξέρατεj» [η απάντηση]

Στιγμιότυπα από την τρίωρη διάλεξη του Γιάννη Ξενάκη, που έγινε στιςΙ 7/9/1975 στην αίθουσα της Εθνικής Λυρικής Σκη­νής, με την ευκαιρία της «Εβδομάδας Ξενάκη», από τις 17 ως τις 21/9/1975 (Ηταν η δεύτερη. Η πρώτη έγινε στις 1 3/1 1/1974, στο Ινστιτούτο «Γκαίτε»). Εδώ, όταν ο Ξενάκης μιλά για «τύπους», εννοεί μαθηματικούς τύπους και μαθηματικές παραμέτρους. Είναι γνωστό πως ο Ιάννης Ξενάκης είναι από τους πρωτοπόρους που "πάντρεψε" τα Μαθηματικά με τη μουσική. Το κύριο υλι­κό που χρησιμοποίησε ήταν η θεωρία των ομάδων (συμπλεγμάτων) και ο Η/Υ. [πηγή: περιοδικό «ΘΕΑΤΡΟ», τεύχος 46, σελ. 106] . Κάποτε, η στήλη θα κάνει ειδικό αφιέρωμα με τις λεπτομέρειες αυτής της ιστορικής συνέντευξης.

ΕΥΚΛΕΙΔΗΣ Β' λθ ' τ.4/14

Page 17: Ευκλειδης Β 60

22η ΒΑLΚΑΝΙΚΗ ΜΑΘΗΜΑΤΙΚΗ ΟΛΥΜΠΙΑΔΑ 10 ΜΑΪ ΟΥ 2005

Η 22η Βαλκανική Μαθηματική Ολυμmάδα διεξήχθη στα Ιάσιο της Ρουμανίας από τις 4 έως 10 Μαίου 2005 . Η Ελληνική αποστολή αποτελείτο από τους μαθητές: Δουβρόπουλο Θεοδόσιο, Παγωνάκη Μηνά, Παναγέα Ιωάννη, Παπαγιαννάκη Νικόλαο, Παπαμιχάλη Θεοφάνη και Παπαμιχάλη Μάριο.

Αρχηγός της αποστολής ήταν ο Μαθηματικός Δημήτριος Κοντογιάννης και υπαρχηγός ήταν ο Μαθη­ματικός Ανδρέας Τριανταφύλλου.

Συμμετείχαν 10 χώρες: Ελλάδα, Αλβανία, Βοσνία Ερζεγοβίνη, Βουλγαρία, Σερβία και Μαυροβούνιο, Πρώην Γιουγκοσλαβική Δημοκρατία της Μακεδονίας, Τουρκία, Ρουμανία, Κύπρος και Μολδαβία.

Οι Έλληνες μαθητές κατέκτησαν τέσσερα μετάλλια ως εξής: Παπαμιχάλης Μάριος - Χρυσό μετάλλιο, Παπαμιχάλης Θεοφάνης - Αργυρό μετάλλιο Δουβρόπουλος Θεοδόσιος - Χάλκινο μετάλλιο Παγωνάκης Μηνάς - Χάλκινο μετάλλιο

ΘΕΜΑΤΑ

Πρόβλημα ι Δίνεται οξυγώνιο τρίγωνο ABC του οποίου ο εγγεγραμμένος κύκλος εφάπτεται στις πλευρές ΑΒ και AC στα σημεία D και Ε αντίστοιχα. Έ­στω Χ και Ψ τα σημεία τομής των διχοτόμων των γωνιών C και Β με την ευθεία DE και Ζ το μέσο της BC. Να αποδειχθεί ότι το τρίγωνο

ΧΨΖ είναι ισόπλευρο αν και μόνο αν Α = 60° • Λύση

Α

-Παρατηρούμε ότι EXC = AED - ACX ( 1 ) . Δ Όμως από το ισοσκελές τρίγωνο ADE έχουμε ότι

AED = 90° - Α . Επίσης έχουμε ότι .ACX = C . 2 2

Οπότε η (1) γίνεται ExC = 90° - Α - C = Β . 2 2 2 Επομένως το τετράπλευρο BIXD είναι εγγράψιμο σε κύκλο με iDB = 90° , άρα fXB = 90° δηλαδή ΧΖ = BC . Όμοια ΖΨ = BC . 2 2 Τελικά ΖΧ = ΖΨ = BC . 2 Αυτό σημαίνει ότι το τρίγωνο ΖΧΨ είναι ισοσκε­λές.

- c c λ - :Β :Β λ Όμως XZB = - + - = C και ΨΖC = - + - = Β 2 2 2 2 Τελικό συμπέρασμα: Ζ = Α = 60° με ΖΧ = ΖΕ Δ <::::> ΧΖΨ ισόπλευρο Πρόβλημα 2 Να βρεθούν όλοι οι πρώτοι αριθμοί p ώστε το

p2 - p + 1 να είναι τέλειος κύβος.

. Λύση. Θέλουμε p2 - ρ + 1 = b3 ή p(p -1) = b3 -1 ή p(p -1) = (b -1) (b2 + b + 1) .

Από τη σχέση b3 ,;, p2 - ρ + 1 έχουμε b3 < ρ2 < ρ3 οπότε b<p (ας μη ξεχνάμε ότι αναφερόμαστε σε

ΕΥΚΛΕΙΔΗΣ Β' λθ' τ.4/15

Page 18: Ευκλειδης Β 60

Μαθηματικοί Διαγωνισμοί - Μαθηματικές Ολυμπιάδες

ακέραιους με p πρώτο) . Από την σχέση p(p - 1) = (b - 1)(b2 + b + 1) κατα­νοούμε ότι ο p είναι διαιρέτης του b2 + b + 1 αφού p>b � p > b - 1 με p πρώτο. Άρα b2 + b + 1 = κ · p και βέβαια p - 1 = κ(b - 1) για κάποιους ακέραιους κ � 2 . Έτσι παίρνουμε b2 + b + 1 = κ2b + κ - κ2 με b2 + b < κ2b και κ2 (b - 1) � b2 + b - 1 . Έτσι για b>2 έχουμε b + 1 < κ2 και

b2 + b - 1 1 κ2 � = b + 2 +-- < b + 3 b - 1 b - 1 Τελικά κ2 = b + 2 οπότε κ = 3 , b = 7, p = 19 . Πρόβλημα 3 Έστω a, b, c θετικοί πραγματοικοί αριθμοί. Να αποδειχθεί ότι

� � � �α - bf - + - + - � α + b + c +----b c α

Πότε ισχύει η ισότητα;

Λύση. Έχουμε ότι:

α + b + c

α2 (α - b)2 - = 2α - b + . Κυκλικά παίρνουμε: b b b2 (b - c)2 c2 (c - α)2 - = 2b - c + και - = 2c - α + ....:....____.:._ c c α α

Οπότε η ζητούμενη ανισότητα ισοδύναμα γίνεται: (α - b)2 (b - c)2 (c - α)2 b ...;,__....:...,_ + + + α + + c � b c α

4(α - b)2 α + b + c + <=> α + b + c (α - b)2 (b - c)2 (c - α)2 4(α - b)2 ...;,__....:...,_ + + � <=> b c α α + b + c

(α + b + c) + + � [ (α b)2 (b c)2 (c α)2 ] b c α

� 4(α - b)2 ( 1 ) . Από την ανισότητα B -C - S έ-[ (α b)2 (b-c)2 (c α)2 ] χουμε: (α + b+c) -- + -- + -- � b c α

(lα - bl +lb - cl +lαl)2 � (2lα - b l )2 ή [ (α - b)2 (b - c)2 ( )2 ] (α + b + c} b + c + : � 4( α - b) 2

Οπότε η ( 1 ) ισχύει: Η ισότητα ισχύει όταν α = b = c και σε ειδική πε­ρίπτωση όταν α>Ο, c=αφ, b = cφ2 όπου

1 + J5 φ =-- . 2 Πρόβλημα 4 Δοθέντος ενός ακεραίου η � 2 να αποδειχθεί ότι

ξ,

λ ' [π + 2] θ , ' λο μετα υ το πο υ -4- αρι μων του συνο υ

{1, 2, • • • , η} είτε υπάρχουν δύο σχετικά πρώτοι

μεταξύ τους είτε ο ένας διαιρεί τον άλλο. Επι-

λέ δ θ ' ' θ ' [η + 2] ' π ον να ειχ ει οτι ο αρι μος -4- ειναι ο

μικρότερος που έχει την ιδιότητα αυτή. Λύση. Έστω ένα υποσύνολο του συνόλου { 1, 2, . . . η} που δεν περιέχει ένα τέτοιο ζεύγος αριθ­μών. Α ν περιέχει στοιχεία μικρότερα ή ίσα του Ε.. . Θε-2 ωρούμε S το μικρότερο από αυτά. Το στοιχείο 2S δεν μπορεί να ανήκει στο υποσύ­νολο αυτό. Αντικαθιστούμε το S από το 2S στο υποσύνολο ο αριθμός των στοιχείων του παραμέ­νει αναλλοίωτος αφού κανένα ζεύγος των ζητου­μένων δεν σχηματίζεται. Ας συνεχίσουμε την διαδικασία αυτή μέχρι που όλα τα στοιχεία του υποσυνόλου να γίνουν μεγα-λ , η υτερα του '2 .

Όμως δύο οποιοιδήποτε διαδοχικοί ακέραιοι από το σύνολο { 1, 2, . . . , η} που να είναι και οι δύο με-

γαλύτερη του Ε.. είναι σχετικά πρώτοι. Αυτό ση-2 μαίνει ότι το υποσύνολο μπορεί να περιέχει το πο­λύ ένα στοιχείο από κάθε τέτοιο ζεύγος. Έτσι θα , λ ' [ η + 2 ] , , , δ εχει το πο υ -4- στοιχεια εκτος απο την ει ι-

κή περίπτωση όπου η = 4κ + 1, κ � 1 .

τ , [ η + 2] , , , οτε -4- = κ που ομως μπορουμε να εχουμε

κ+ 1 στοιχεία στο υποσύνολο σύμφωνα με την πα­ραπάνω επιλογή διαδοχικών στοιχείων μόνο όταν και τα δύο 2κ + 1 και 4κ + 1 ανήκουν στο υποσύ­νολο. Μόνο τότε αυτά τα δύο είναι σχετικά πρώτα στοιχεία πράγμα που απαγορεύεται. Τελικά το συμπέρασμα ισχύει για όλα τα η όπου οποιοδήποτε υποσύνολο με περισσότερα από ( η + 2 ) ' θ ' ' ζ ' ' -4- στοιχεια α περιεχει ενα ευγος απο τα

ζητούμενα.

ΕΥΚΛΕΙΔΗΣ Β' λθ ' τ.4/16

Page 19: Ευκλειδης Β 60

••ιιιι•"�••• JU �· �� 'Ι�Iι ,. Α•-Ι•

1) Αν ν είναι φυσικός αριθμός, με ν -:�; Ο , να βρε­

θεί η τιμή της παράστασης: Α = (-1)ν + (-1)ν+1 + (-1)ν+2 + (-1)ν+3 •

ΛΥΣΗ Αν ν άρτιος τότε ν+ 1 περιττός, ν+2 άρτιος και ν+3 περιττός. Άρα: Α = 1 + ( -1) + 1 + ( - 1) = Ο . Αν ν περιττός, τότε ν+ 1 άρτιος, ν+2 περιττός και ν+ 3 άρτιος. Άρα: Α = ( - 1) + 1 + ( - 1) + 1 = Ο . Ώστε Α = Ο, για κάθε ν Ε w* .

2) Αν α,β,γ είναι τα μήκη πλευρών τριγώνου και

α2 - βγ 2α - β - γ ισχύει η σχέση : = (1), να βρε-

β + γ 2

θεί το είδος του τριγώνου.

Από την ( 1 )

Ισοδύναμα, έχουμε:

ΛΥΣΗ

2( α 2 - βγ) = (β + γ )(2α - β - γ) � 2α2 - 2flγ + β2 + γ2 + 2_βγ - 2αβ - 2αγ = Ο � 2α2 + β2 + γ2 - 2αβ - 2αγ = 0 � (α2 + β2 - 2αβ) + (α2 + γ2 - 2αγ) = ο �

(α - β)2 + (α - γ)2 = 0 � α- β = Ο και α - γ = Ο � α = β και α = γ, δηλαδή α = β = γ. Άρα, το τρίγωνο είναι ισόπλευρο.

3) Αν α>Ο, i) Να δειχθεί ότι:

α-1 + α-3 > α-2 (1) και ii) Να απλοποιηθεί το

, α-3 - α-2 + α-1 κλασμα: Κ = _4 _3 _2 •

α - α + α

ΛΥΣΗ i) Από την ( 1 )

jUγεfJpα Γ. Αποστολόπουλος

'Εχουμε: α-3 - α-2 + α-1 = ..!. +-1 - -1 = α α3 α2 α2 + 1 - α α2 - α + 1 , = 3 3 > Ο γιατι

α α

{::�3 < 0��ι> 0} ii) Το κλάσμα γράφεται:

1 1 1 1 - α + α2 - - - + -

κ = α3 αz α = α3 =

1 1 1 1 - α + α2 - - - + -α4 α3 α2 α4

α4 · (1 - α + α2 ) α4 = = - = α α3 · (1 - α + α2 ) α3 ·

4) Αν για τους πραγματικούς αριθμούς α, β με α · β * Ο , ισχύουν οι σχέσεις;

(;)' + β' � 2α ( 1) και (�)' + α' � 2β (2), να

δειχθεί ότι: α = β = 1.

ΛΥΣΗ Οι σχέσεις ισοδύναμα γράφονται:

α2 Από ( 1 ) : 2 + β2 - 2α = Ο και

β

(2) : β: + α2 - 2β = Ο � α

α2 + β4 - 2αβ2 = 0 και β2 + α4 - 2α2β = 0 � ( 2 )2 2 2 2 α - β = 0 και (α -β) =0� α = β και α2=β.

Τις πολλαπλασιάζουμε κατά μέλη, έχουμε: α3 = β3 � α3 - β3 = 0 � (α - β) (α2 + αβ + β2 ) = 0 ,

αλλά α2 + αβ + β2 > Ο γιατί Δα = -3β2 , Δp = -3α2 , 1>0

ΕΥΚΛΕΙΔΗΣ Β' λθ' τ.4/17

Page 20: Ευκλειδης Β 60

Μαθηματικά για την Α ' Λυκείου

άρα α - β = Ο <=> α = β . Οπότε η σχέση α = β2 γίνεται α = α2 <=> α = 1 , γιατί α :;t: Ο . Ώστε α = β = 1 .

5) Α ν α, β>Ο και ισχύει: 2α + 3β = 60 , να βρεθεί μεταξύ ποιων αριθμών παίρνει τιμές το άθροι­σμα α+β.

ΛΥΣΗ

Είναι: 2α + 3β = 60 <=> 2α + 2β = 60 - β <=>

α + β = 30 - � και επειδή β>Ο, έχουμε α+β < 30. 2 Επίσης: 2α + 3β = 60 <=> 3α + 3β = 60 + α <=>

α α + β = 20 +- και επειδή α > Ο, έχουμε: 3 α + β > 20 . Ώστε: 20 < α + β < 30 .

6) Αν α,β,γ ε R και ισχύει: α+β+γ = 9, να βρε­θεί η ελάχιστη τιμή της παράστασης: α2+β2+'?.

ΛΥΣΗ

Έχουμε ότι: α2 + β2 + γ2 + 2αβ + 2βγ + 2γα = (α + β + γ)2 <=> α2 + β2 + γ2 = 8 1 - 2αβ - 2βγ - 2γα <=> 3α2 + 3β2 + 3γ2 = 8 1 - 2αβ - 2βγ - 2γα + +2α2 + 2β2 + 2γ2 <=> 3 (α2 + β2 + γ2 ) = 8 1 + (α2 + β2 - 2αβ) +

+ (β2 + γ2 - 2βγ) + (γ2 + α2 - 2γα) <=> 3 ( α2 + β2 + γ2 ) = 8 1 + (α - β)2 + (β - γ)2 +

+(γ - α)2 <=> 3 ( α2 + β2 + γ2 ) - 8 1 =

= (α - β)2 + (β - γ)2 + (γ - α)2 � 0 . Άρα: 3 (α2 + β2 + γ2 ) - 8 1 � 0 <=>

α2 + β2 + γ2 � 27 . Επομένως η ελάχιστη τιμή του α2 +β2 +γ2 είναι: 27 .

7) Αν α > β > γ, να βρεθεί το πρόσημο της πα­ράστασης: Κ = αβ2 + βγ2 + γα2 _ α2β _ β2γ _ γ2α .

ΛΥΣΗ

Για να βρεθεί το πρόσημο της παράστασης Κ, θα πρέπει αυτή να γίνει γινόμενο παραγόντων. Έχουμε:

Κ = ( αβ2 _ α2β ) _ ( γ2α _ βγ2 ) + ( γα2 _ β2γ) =

= -αβ(α - β) - γ2 (α - β) + γ (α2 - β2 ) =

= -αβ(α - β) - γ2 (α - β) + γ( α + β)( α - β) = = (α - β) · [ -αβ - γ2 + γ(α + β)] =

= (α - β) · {-α β - γ2 + α γ + βγ) = = (α - β) · [β(γ - α) - γ(γ - α) ] = = (α - β)( γ - α)(β - γ) < Ο γιατί α - β > Ο , γ - α < Ο και β - γ > Ο .

8) Α ν χ > y > Ο, να δειχθεί ότι: ( χ4 - y4 ) . ( χ3 + y3 ) > ( χ3 - y3 ) . ( χ4 + y4 ) •

ΛΥΣΗ

'Εχουμε {χ4 - y4 ) {x3 + y3 ) - {x3 - y3 ) {x4 + y4 ) = = χ7 + x4y3 - x3y4 - y7 - χ7 - x3y4 + x4y3 + y7 = = 2x4y3 - 2x3y4 = 2x2y2 (x - y) > Ο , γιατί x,y > Ο και χ-y > Ο.

9) Α ν α,β > Ο, διάφοροι του 1 και ισχύει: α-2 + β-2 = α-1 + β-1 (1), να δειχθεί ότι οι αριθ-μοί α-1, β-1 είναι ετερόσημοι.

ΛΥΣΗ

Α , ( 1 ) , 1 1 1 1 πο την εχουμε: 2 +2 = -+- <=> α β α β

β2 + α2 = αβ2 + α2β <=> α2 _ α2β = αβ2 _ β2 <=> 2 2 β;<Ι α2 α - 1 α (1 - β) = β (α - 1)<=>- = - > 0 <=>

β2 1 - β α - 1 Ο Ά θ ' 1 β 1 ' ' -- < . ρα οι αρι μοι α- , - ειναι ετεροση-β - 1 μοι, αφού έχουν αρνητικό πηλίκο.

10) Αν για το πραγματικό αριθμό χ ισχύει: χ3 + χ2 + χ = 2006 , να δειχθεί ότι χ > Ο.

ΛΥΣΗ

'Εχουμε: χ { χ2 + χ + 1 ) = 2006 > Ο αλλά επειδή

Δ = -3 < Ο, 1 > Ο χ 2 + χ + 1 > Ο , για κάθε χ ε R , άρα επειδή οι παράγοντες χ, χ 2 + χ + 1 είναι ομό­σημοι, θα είναι χ > Ο.

ΕΥΚΛΕΙΔΗΣ Β ' λθ ' τ.4/18

Page 21: Ευκλειδης Β 60

Μαθηματικά για την Α' Λυκείου

1 1) Ν α λυθεί στο Ζ , η εξίσωση

(3χ - 6)3 = (2χ - 1)3 + (χ - 5)3 • ΛΥΣΗ

Ισοδύναμα έχουμε: (3χ - 6) 3 - (2χ -1) 3 - (χ - 5) 3 =ο <:::::> (3χ - 6) 3 + (1- 2χ) 3 + (5 - χ) 3 =0. Είναι γνωστό όμως, ότι: αν α+β+γ = Ο τότε α3 + β3 + γ3 = 3αβγ. Εδώ έχουμε: (3χ - 6) + (1- 2χ) + (5 - χ)= Ο

' (3χ - 6) 3 + (1- 2χ) 3 + (5 - χ) 3 = αρα = 3(2χ - 6) (1- 2χ(5 - χ) =0

δηλαδή 3χ - 6 =Ο ή 1- 2χ =Ο ή 5 - χ= Ο οπότε χ = 2 ή χ=_!.. � Ζ (απορ.) ή χ = 5. 2 Άρα οι ακέραιες λύσεις της αρχικής εξίσωσης εί­ναι χ = 2 ή χ = 5.

12) Αν lxl � 1 , να απλοποιηθεί η παράσταση :

Α = 2 lx - 1l - 3 lx l + lx + 11 . ΛΥΣΗ

Έχουμε lxl � 1 <:::::> χ � -1 ή χ � 1. Α ν χ � -1 , τότε I χ + 11 = - χ -1 και lx -11 = - χ+ 1, lxl = - χ οπότε: Α = 2(- χ + 1) - 3(- χ) - χ -1 = =- 2χ + 2+ 3χ - χ -1 = 1 Αν χ � 1, τότε lx -11 = χ -1 , lxl = χ και lx + 11 = χ+ 1, οπότε: Α= 2(χ -1) - 3χ + χ+ 1 = 2χ - 2 - 3χ + χ+ 1 = -1.

Άρα Α= . {1 , Χ � -1 -1,χ � 1

13) Αν α, β ε � να δειχθεί ότι: [d<�β)Τ + [d<α�-β ] � αβ .

ΛΥΣΗ

Ξέρουμε ότι η απόσταση δύο σημείων στην ευθεία των πραγματικών αριθμών είναι d(α,β) = Ια - βl, άρα d(α,- β)= Ια+ βΙ. Αρκεί λοιπόν να δείξουμε ότι: ( Ι α�βl )' + ( Ια:βΙ )' - αβ � ο.

Πράγματι: ( Ι α;βΙ )' + ( Ι α: βl )' - αβ=

= (α - β) 2 + (α+ β) 2 - αβ= 4 . 4 α2 - 2αβ + β2 + α2 + 2αβ + β2 - 4αβ = = 4

= 2α2 + 2β2 - 4αβ = 2 (α2 - 2αβ+ β2 }

= 4 4 = α2 - 2αβ+ β2 = (α - β) 2 � 0. 2 2

14) Να απλοποιηθεί το κλάσμα:

�29 + 12J5 - �29 - 12J5 �9 + 4J2 - �9 - 4J2

ΛΥΣΗ

'Ε χουμε: �29 + 12-JS > �29 -12-JS . Τότε χ= �29 + 12-JS - �29 -12.J5 > Ο, επομένως χ2 = 29+12.J5 + 29 -12.J5 - 2J292 - (12.J5)2 <:::::> χ2 =58 - 2 · .Jϊ2ι <:::::> χ2 = 36 <:::::> χ= 6, γιατί χ > Ο. Επειδή �9+ 4-fi > �9 - 4.fi τότε έχουμε y = �9 + 4.fi - �9 - 4.fi > Ο, επομένως

y2 = 9 + 4.fi + 9 - 4.fi - 2J92 - ( 4.fi}2 <:::::>

y2 = 18 - 2.J49 <:::::> y2 = 4 <:::::> y = 2, γιατί y> Ο ο ' κλ ' ' χ 6 3 ποτε το ασμα, ισουται με: -= -= . Υ 2

15) Να βρεθεί το πεδίο ορισμού της συνάρτη-2χ - 1 2χ 1

σης: f(x) = -2-x- + -1---2-x + -4χ--::2---2-χ

και να απλοποιηθεί ο τύπος της.

ΛΥΣΗ

Για να ορίζεται η συνάρτηση πρέπει: (2x ;,t: O και 1- 2x ;,t: O και 4x2 - 2x ;,t: O) <:::>

1 (χ * Ο και χ * - και 2χ(2χ -1) * 0) <:::::> 2 (χ * Ο και χ * Ξ)

Άρα το πεδίο ορισμού της f είναι:

ΕΥΚΛΕΙΔΗΣ Β' λθ ' τ.4/19

Page 22: Ευκλειδης Β 60

Μαθηματικά για την Α ' Λυκείου

. 2χ - 1 2χ 1 Εχουμε: f (x) = -- ---+ = 2χ 2χ - 1 2χ(2χ - 1)

(2χ - 1)2 - (2χ)2 + 1 = = 2χ(2χ - 1)

-4χ + 2 -2(2χ - 1) = = = 2χ(2χ - 1) 2χ(2χ - 1)

Ώστε: f (x) = _ _!_ , με χ =F- Ο και χ =F- _!_ . χ 2

16) Να λυθεί στο IR , η εξίσωση : 1 2 3

χ

---,---+ = . χ2 + χ + 1 χ2 - χ + 1 χ4 + χ2 + 1

ΛΥΣΗ

Επειδή Δ = -3 < Ο και 1>0 είναι χ2 + χ + 1 > Ο και χ2 - χ + 1 > Ο για κάθε τιμή του χ ε IR . Επίσης: (χ2 + χ + 1) · (χ2 - χ + 1) = χ4 + χ2 + 1 .

Κάνουμε απαλοιφή παρανομαστών και έχουμε: χ2 - χ + 1 + 2 (χ2 + χ + 1) = 3 <:>

χ 2 - χ + 1 + 2χ 2 + 2χ + 2 - 3 = ο <:::> 3χ 2 + χ = ο <:::> χ(2χ + 1) = ο <:::>

χ = ο ή 3χ + 1 = ο <:::> χ = ο ή χ = _ _!_ • 3

17) Α ν η μικρότερη ρίζα της εξίσωσης: χ ( 32 - χ) = 31 επαληθεύει την εξίσωση

λ2 (χ + 4μ 2 ) + 2λ( χ + 2λμ) + χ + λ2 = ο ' να βρεθούν οι πραγματικοί αριθμοί λ, μ.

ΛΥΣΗ

Έχουμε: 32χ - χ 2 = 3 1 <:::> χ 2 - 32χ + 3 1 = Ο , Δ = 900, άρα:

χ - 32 ± 30 (3 1 1 ,2 - 2 1

Η άλλη εξίσωση επαληθεύεται για χ = 1 . Δηλαδή : λ2 ( 1 + 4μ 2 ) + 2λ(1 + 2λμ) + 1 + λ2 = ο <:::>

λ2 + 4λ2μ2 + 2λ + 4λ2μ + 1 + λ2 = 0 <:::> (λ2 + 2λ + 1) + (4λ2μ2 + 4λ2μ + λ2 ) = 0 <:>

(λ + 1)2 + (2λμ + λ)2 = Ο , οπότε;

{λ + 1 = Ο και 2λμ + λ = Ο} <:::>

{λ = -1 και -2μ - 1 = 0} <=> {λ = -1 και μ = -i} 18) Αν η παράσταση : (α + 3χ)2 + (β + 4χ)2 , α, β ε JR* ,

είναι τέλειο τετράγωνο, να βρεθεί ο λόγος α . β

ΛΥΣΗ

Έχουμε: (α + 3χ)2 + (β + 4χ)2 = 25χ2 + 2(3α + 4β)χ + α2 + β2

Για να είναι τέλειο τετράγωνο πρέπει, η διακρί­νουσα του τριωνύμου να είναι μηδέν. Δηλ. Δ = Ο <:::> 4(3α + 4β)2 - 4 · 25 · ( α2 + β2 ) = Ο <:::>

(3α + 4β)2 - 25α2 - 25β2 = Ο <:::> 9α2 + 1 6β2 + 24αβ - 25α2 - 25β2 = Ο <:::> - 16α 2 - 9β 2 + 24αβ = Ο <:::> <:::> (4α)2 + (3β)2 - 2 · 4α · 3β = Ο <:::>

β;eΟ α 3 ( 4α - 3β)2 = Ο <:::> 4α - 3β = Ο <:::> 4α = 3β <=>- = - . β 4

19) Δίνεται η εξίσωση : αχ2 + βχ + γ = Ο , με αγ =F- Ο και β Ε

JR • Α ν ισχύει l lα l - lγ l l = Ι α + γ Ι , να

δειχθεί ότι εξίσωση έχει δύο ρίζες πραγματικές και άνισες.

ΛΥΣΗ

Έχουμε: l lα l - lγ l l2 = Ια + γ l2 <=> ( lα l - lγ l )2 = (α + γ)2 <=> α2 + γ2 - 2 lαγ 1 = α2 + γ2 + 2αγ <=> -2 1αγ 1 = 2αγ <=> Ιαγ l = -αγ <=> α · γ < Ο <:::> -4αγ > Ο . Οπότε β2 - 4αγ > Ο , δηλαδή Δ > Ο. Άρα έχει δύο ρίζες πραγματικές και άνισες.

20) Να βρεθούν οι ρίζες της εξίσωσης: χ2 - Sx + λ = Ο , λ Ε

JR , αν το άθροισμα των τε-τραγώνων τους είναι 34.

ΛΥΣΗ

Πρέπει Δ � Ο <:::> 64 - 4λ � Ο <:::> λ ::; 16 . Είναι: χ 1 + χ2 = 8 , χ 1 χ2 = λ και χ� + χ � = 34

ΕΥΚΛΕΙΔΗΣ Β ' λθ ' τ.4/20

Page 23: Ευκλειδης Β 60

Μαθηματικά για την Α ' Λυκείου

Έχουμε: ( χ 1 + χ2 )

2 - 2χ ι χ2 = 34 <=::> 64 - 2λ = 34 <=::> -2λ = -30 <=::> λ = 1 5 .

Άρα, η εξίσωση γίνεται: χ 2 - 8χ + 1 5 = Ο

' 8 ± 2 (5 Δ = 4, οποτε Χι 2 = --, 2 3

21) Για ποιες τιμές του λ ε JR , οι ρίζες είναι χ1,χ2 της εξίσωσης χ2 - 20χ + Sλ + 6 = Ο , είναι ανάλο­γες αντίστοιχα των αριθμών 2 και 3;

ΛΥΣΗ

Θα πρέπει Δ > Ο <=::> 400 - 4(5λ + 6) > Ο <=::> λ < 94 . 5 'Ε χουμε � = 2 και από τους τύπους VIETA ότι

2 3 Χ ι + χ2 = 20 και Χ ι χ 2 = 5λ + 6 .

Ε, 2 3 3χι , ιναι: χ2 = Χ ι <=::> χ2 = - οποτε: 2

3χ Χ ι + -ι = 20 <:::> 5χι = 40 <:::> Χ ι = 8 2

3 · 8 άρα χ2 = - = 12 . 2 Αντικαθιστούμε στην Χ ιχ 2 = 5λ + 6 , έχουμε:

5λ + 6 = 96 <=::> 5λ = 90 <=::> λ = 1 8 . Ώστε: λ = 1 8 .

22) Να λυθεί στο JR* , το σύστημα: {! + ! = � (1), _!_ + _!_ = �(2)} . χ Υ 2 xz yz 4 ΛΥΣΗ

Από την (2) έχουμε: (_!_ + _!_)2 - 2 = 1 χ y xy 4

ή λόγω της ( 1 ) 2. _ 2 = 1 <=::> xy = 2 4 xy 4

από την ( 1 ) έχουμε: χ + Υ = � <=::> χ + y = 3 . xy 2

Δηλαδή, έχουμε να λύσουμε το σύστημα: χ + Υ = 3} xy = 2

Τα x,y θα είναι ρίζες της εξίσωσης: ω2 - 3ω + 2 = Ο <=::> ω = Ι ή ω = 2

άρα: (x , y) = (1, 2) ή (x , y) = (2, 1) .

23) Σε καρτεσιανό επίπεδο δίνονται τα σημεία Α(-2,1), Β(3,2). Να βρεθεί σημείο Μ του άξονα y 'y, ώστε (ΜΑ) = (ΜΒ). Είναι ορθογώνιο το τρίγωνο ΑΜΒ;

ΛΥΣΗ

Αν Μ(ο,λ), λ ε !R είναι το σημείο του y 'y ώστε (ΜΑ) = (ΜΒ), τότε: �(-2 - 0)2 + (1 - λ)2 = �(3 - 0)2 + (2 - λ)2 <=::> 4 + 1 - 2λ + λ2 = 9 + 4 - 4λ + λ2 <=::> 2λ = 8 <=::> λ = 4 . Άρα Μ (0,4) , Θα βρούμε τα μήκη των πλευρών του τριγώνου ΑΜΒ. (ΑΒ) = �( -2 - 3)2 + (1 - 2)2 = .J25 + 1 = J26

(ΜΑ) = �(-2 - 0)2 + (1 - 4)2 = .J4 + 9 = Jl3} (MB) = Jl3 (ΜΑ) = (ΜΒ) και επειδή

2 �2 (ΑΒ) = ν26 = 26 = 1 3 + 1 3 =

= Jl32 + Jl32 = (ΜΑ)2 + (ΜΒ)2

δηλαδή εφαρμόζεται το πυθαγόρειο θεώρημα για το ΑΜΒ τρίγωνο. Επομένως το τρίγωνο ΑΜΒ είναι ορθογώνιο και ισοσκελές, με υποτείνουσα την ΑΒ.

24) Να λυθεί στο Ζ , η aνίσωση: �3� - �� -� < 1 . ΛΥΣΗ

Ισοδύναμα έχουμε: - 1 < 3x2 - l x l - 2 < 1 και επειδή χ ε Ζ , θα είναι 3χ2 - l x l - 2 = 0 <:::> 3 l x l2 - l x l - 2 = 0 . Θέτουμε l x l = ω > Ο και η εξίσωση γράφεται 3ω2 - ω - 2 = Ο , Δ = 25 άρα

1 ± 5 (1 ( ' ) ω = -- 2 απορριπτεται 6 --3

ώστε l x l = 1 <=::> χ = 1 ή χ = - 1 .

2x + y = 264} 25) Να λυθεί στο JR , το σύστημα: r::

χ - νΥ = 92 ΛΥΣΗ

. JΎ = χ - 92 } Ισοδύναμα, έχουμε: y = 264 - 2x

ΕΥΚΛΕΙΔΗΣ Β' λθ ' τ.4/21

Page 24: Ευκλειδης Β 60

Μαθηματικά για την Α' Λυκείου

Πρέπει: y 2: Ο , οπότε: χ - 92 2: Ο και 264 - 2χ 2: Ο � χ 2: 92 και χ � 1 32 . Δηλαδή y 2: Ο και 92 � χ � 1 32 . Από τις δύο εξισώσεις του συστήματος, προκύπτει: .J264 - 2χ = χ - 92 , υ­ψώνουμε τα μέλη της, στο τετράγωνο. Έχουμε: 264 - 2χ = (χ - 92)2 �

χ 2 - 1 82χ + 8 · 200 = Ο � χ = 100 ή χ = 82 ( απορ­ρίπτεται) . Ώστε χ = 100, οπότε y = 264-200 = 64. (x,y) = ( 1 00,64) .

Λuμcνc( cnαναληnτιιιc� αα«ήacι� στην iU γcfJpα

Α' ΛΥΜΕΝΕΣ

1 . Έστω Α = (λ- 1)2 - (λ + 1)2 και Β = fi! - .fil , όπου λ είναι πραγματικός αριθ­μός. i) Βρείτε το ανάπτυγμα της παράστασης Α. ίί) Απλοποιήστε την παράσταση Β. iii) Επιλύστε την εξίσωση Α · χ = Β • iv) Προσδιορίστε τις τιμές του λ :�; Ο , για τις οποίες η λύση της εξίσωσης δεν υπερβαίνει το 13 v) Υπολογίστε τη ρίζα της εξίσωσης με λ =2005.

ΛΥΣΗ

ί) Έχουμε, Α = (λ - 1)2 - (λ + 1)2 = = (λ2 - 2λ + 1) - (λ2 + 2λ + 1) = = λ2 - 2λ + 1 - λ2 - 2λ - 1 = -4λ

ίίί) Ισχύουν: Αν λ > Ο , Α · χ = Β � (-4λ)χ = λ - λ2 , οπότε αν -4λ :�; Ο � λ :�; Ο η εξίσωση έχει μοναδική λύση

λ - λ2 -λ(λ - 1) 1 την χ = -- = = -(λ - 1) , ενώ αν λ = Ο -4λ -4λ 4

επειδή (-4 · 0)χ = 0 - 02 � Ο · χ = Ο , η εξίσωση εί-ναι αόριστη.

Γ. Στρατήγης

Αν λ<Ο, Α · χ = Β � ( -4λ)χ = -λ - λ2 και εφόσον λ<Ο είναι λ :�; Ο , η εξίσωση έχει μοναδική λύση

την χ = -λ - λ2 = -λ(λ + 1) = .!_ (λ + 1) . -4λ -4λ 4 iv) Θέλουμε χ � 1 3 , επομένως: Αν λ>Ο,

1 χ � 1 3 � -(λ - 1) � 1 3 � λ - 1 � 4 · 1 3 � λ � 53 4 δηλαδή ο< λ � 53 ' ( 1 ) .

=1 513 : Αν λ<Ο,

1 χ � 1 3 � - (λ + 1) � 1 3 � λ + 1 � 4 · 1 3 � λ � 5 1 4 δηλαδή λ<Ο, (2).

ο 51

Τελικά από τις σχέσεις ( 1 ), (2) προκύπτει ότι λ ε ( -οο, Ο) υ (0, 53] .

ν) Επειδή λ = 2005>0 η ρίζα της εξίσωσης είναι ο αριθμός χ = .!.. (2005 - 1) = 50 1 . 4 2. Ν α επιλυθεί η εξίσωση :

I 2 3 2 1 2 �,..:..( χ-+-3--:-)2 ___ 12

-χ 4

χ - χ + + = • 3 - χ ΛΥΣΗ

Με χ :�; 3 έχουμε . . .

lx 2 - 3χ + 2 1 + 2 · �,....(χ---3)

-2 = 4 �

3 - χ

� ιχ2 - 3χ + 2 Ι = 4 + 2 ιχ - 3 ι ( 1 ) . χ - 3

ΕΥΚΛΕΙΔΗΣ Β ' λθ ' τ.4/22

Page 25: Ευκλειδης Β 60

Μαθηματικά για την Α ' Λυκείου

Διακρίνουμε τώρα τις περιπτώσεις: i) Αν χ - 3 > Ο <=> χ > 3 από (1) έχουμε Ιχ2 - 3χ + 2 1 = 4 + 2 · 1 = 6 <=>

<=> χ2 - 3χ + 2 = 6 ή χ2 -3χ+2=-6<=>(χ-4Χχ+1) =0 ή χ2 - 3χ + 8 = 0 , (αδ. , Δ=-23<0)<=>χ=4, αφού χ>3 . ii) Αν χ<3 , τότε Ιχ2 - 3χ + 2 1 = 4 + 2(- 1) = 2 <=> χ2 - 3χ + 2 = 2 ή

χ2 - 3χ + 2 = -2 <=> <::> χ(χ - 3) = 0 ή χ2 - 3χ + 4 = 0 (αδ. γιατί Δ = -7 < Ο) <=> χ = Ο . Άρα οι ρίζες της εξίσωσης είναι χ = Ο ή χ = 4.

3. Έστω οι ευθείες: .J' _ ε1 : y = (λ - l)(λ + 2)χ + 2λ + ι ,

f/' ε2 : y = -(λ + 3)χ + (λ - 1)2

i) Να βρεθεί η πραγματική τιμή του λ ώστε οι ευθείες να είναι παράλληλες. ii) Για την τιμή του λ που βρήκατε, αφού σχε­διάσετε τις ευθείες ε1 και ε2, να υπολογίσετε την απόστασή τους.

ΛΥΣΗ

i) Οι ευθείες είναι παράλληλες μόνο όταν (λ - 1)(λ + 2) = -(λ + 3) <=> λ2 + 2λ + 1 = ο <=> (λ + 1)2 = ό <=> λ = -1 διπλή ρίζα ii) Για λ = -1 οι εξισώσεις των ευθειών γράφονται: ει : y = -2χ - 1 , ε2 : y = -2χ + 4 . Η ευθεία ε2 τέμνει τον άξονα χ 'χ στο σημείο Α(2,0). Θα βρούμε την εξίσωση της ευθείας ε3 που διέρχεται από το Α και είναι κάθετη στην εz . Είναι

1 δ ' ε3 : y = αχ + β με α( -2) = -1 <=> α = "2 και επει η

1 διέρχεται από το σημείο Α : Ο = - · 2 + β <=> β = -1 , 2

1 ο θ ' ' οπότε ε3 : y = -χ - 1 . ι ευ ειες ει , ε3 τεμνονται 2 στο σημείο Β του οποίου οι συντεταγμένες προκύ-{Υ = -2χ - 1 πτουν από τη λύση του συστήματος Υ =� χ _ 1

Απ' όπου βρίσκουμε Β(Ο,- 1 ) .

Υ \ Α

1\ 1\ \

\ 1\ -1, !\ ο \�

� \ / v r\2 �

�' 1'\. \< ε2

Υ I'

Η απόσταση των παράλληλων ευθειών ει και εz εί­ναι το μήκος του τμήματος ΑΒ. Έχουμε (ΑΒ) = �(0 - 2)2 + (-1 - 0)2 = J5.

� χ3 - 1 4. Δίνεται η συνάρτηση f(x) = --1 •

x -i) Να βρεθεί το πεδίο ορισμού της και η τιμή f(-1) • ii) Να απλοποιηθεί ο τύπος της. iii) Να επιλυθεί η ανίσωση f(x) > f (- 1) .

ΛΥΣΗ

i) Πρέπει χ - 1 * Ο <=> χ * 1 , οπότε το πεδίο ορι­σμού της είναι το σύνολο Α = ( -οο, 1) υ (1, +οο) .

(- 1)3 - 1 -2 Έχουμε f (- 1) _1 _ 1 = _2 = 1 .

. . , (x - l)(x2 + χ + 1) 2 η) Ειναι f (x) = = χ + χ + 1 . χ - 1 iii) Θέλουμε

i ),ii ) f (χ) > f ( - 1) <=> χ 2 + χ + 1 > 1 <=> χ(χ + 1) > ο .

------------j___JL__JL ___ :==========.�x Χ' +-Ο ___ __,J ι__::

Επομένως οι λύσεις της ανίσωσης χ ε (-οο, - 1) υ (0, 1) υ (1, +οο) .

είναι

5. Για τις διαφορές πραγματικές τιμές του α- )I; ριθμού λ, να επιλύσετε την εξίσωση: / λ( χ - 1)2 + χ(2λ - χ) = 3(χ2 + λ) .

ΛΥΣΗ

Έχουμε λ( χ - 1)2 + χ(2λ - χ) = 3(χ 2 + λ) <=> <=> λ(χ2 - 2χ + 1) + χ(2λ - χ) = 3(χ2 + λ) <=> <=> λχ2 - 2,ix + λ + 2,ix - χ2 = 3χ2 + 3λ <::> <=> -4χ2 + λχ2 - 2λ = ο <=> χ2 (λ - 4) = 2λ ( 1 ) . Αρχικά εξετάζουμε την περίπτωση μηδενισμού του συντελεστή λ-4 του χ2• Είναι λ - 4 = 0 <::> λ = 4 .

ΕΥΚΛΕΙΔΗΣ Β ' λθ ' τ.4/23

Page 26: Ευκλειδης Β 60

Μαθηματικά για την Α' Λυκείου

Με λ = 4 η εξίσωση ( 1 ) γράφεται χ 2 · Ο = 2 · 4 , που είναι αδύνατη . Αν λ :;t: 4 , η ( 1 ) γράφεται χ2 = � (2), που για λ - 4 να έχει λύσεις πρέπει � � Ο � 2λ(λ - 4) � Ο � λ ::; Ο ή λ>4 (γιατί;) . λ - 4 Άρα με λ ::; Ο ή λ>4 η εξίσωση έχει τις λύσεις

χ = ±J 2λ , ενώ αν Ο < λ ::; 4 είναι αδύνατη . λ - 4 I .

_j, 9· Αν χ1, χ2 είναι πραγματικές ρίζες της δευτε-/ /ροβάθμιας εξίσωσης: 2λχ2 - 4χ + λ - 1 = Ο , να

1 προσδιορίσετε τις πραγματικές τιμές του λ ώστε 2 2 2 Χ ι + Χ 2 � •

ΑΥΣΗ Για τις πραγματικές ρίζες χ 1 και χ2 της εξίσωση πρέπει λ :;t: Ο και

Δ � ο � 16 - Sλ(λ - 1) � ο � -8(λ2 - λ - 2) � ο � � -8(λ - 2)(λ + 1) � ο � (λ - 2)(λ + 1) ::; ο Επομένως -1 ::; λ < 0 ή 0 < λ $ 2 ( 1 ) . ' β -4 2 Εχουμε Χ ι + χ2 = -- = -- = - και α 2λ λ

γ λ - 1 ' Χ ι Χ2 = - = -- , οποτε α 2λ

χ� + χ � � 2 � (χι + χ2 )2 - 2χ 1χ 2 � 2 � ( 2)2 λ 1 λ -j ;λ

� 2 � 4 - λ(λ - 1) � 2λ2 , αφού

λ :;t: Ο , δηλαδή 3λ2 -λ-4 $ 0� (λ + 1)(3λ - 4) ::; Ο .

Άρα -1 ::; λ < Ο ή Ο < λ ::; i , που φυσικά πληρούν 3 την σχέση ( 1 ) .

• ΓΕΩΜΕΤΡΙΑ

Λuμένες Εnαναληnτιιιές Αιτιιήιτεις ιττην

rεωμετpία της Α ' Λuιιείοu

Ευσταθίου Ευάγγελος Καρδαμίτσης Σπύρος

Οι παρακάτω ασκήσεις έχουν σκοπό να συμβάλλουν στην προσπάθεια των μαθητών για μια ουσιαστική

επανάληψη στην Γεωμετρία της Α ' Λυκείου. Σ' αυτές τις ασκήσεις απαιτείται συνδυασμός γνώσεων από

διαφορετικά κεφάλαια της θεωρίας. Για κάθε άσκηση αναφέρουμε την αντίστοιχη θεωρία που απαιτείται

για την επίλυσή της.

ΑΣΚΗΣΗ 1 '1 Δίνεται ισοσκελές τρίγωνο ΑΒΓ με ΑΒ=ΑΓ και

Λ Α >90°. Από την κορυφή Α φέρνουμε τις ημιευ-θείες Αχ κάθετη στην πλευρά ΑΒ και By κάθετη στην πλευρά ΑΓ που τέμνουν την ΒΓ στα ση­μεία Δ και Ε αντίστοιχα. Να αποδείξετε ότι: α) Το τρίγωνο ΕΑΔ είναι ισοσκελές β) Τα τρίγωνα ΑΒΕ και ΑΔΓ είναι ίσα.

Λ Λ

γ) ΒΑΕ = Α- 2Β 2

(Ιδιότητες ισοσκελών τριγώνων-Ισότητα τριγώ­νων-Άθροισμα γωνιών τριγώνου)

ΑΥΣΗ

Α

Β Γ Υ χ

α) Τα ορθογώνια τρίγωνα ΑΒΔ και ΑΕΓ έχουν: Λ Λ

ΑΒ=ΑΓ και Β = Γ (γιατί το ΑΒΓ είναι ισοσκε-λές), συνεπώς είναι ίσα, τότε ΑΔ=ΑΕ άρα το τρί­γωνο ΕΑΔ είναι ισοσκελές. β) Τα τρίγωνα ΑΒΕ και ΑΔΓ έχουν ΑΒ=ΑΓ,

Λ Λ Β = Γ και ΒΕ=ΔΓ (γιατί είναι ΒΔ=ΕΓ από την ισότητα των τριγώνων ΑΒΔ και ΑΓΕ, επομένως

ΕΥΚΛΕΙΔΗΣ Β' λθ ' τ.4/24

Page 27: Ευκλειδης Β 60

Μαθηματικά για την Α' Λυκείου

ΒΔ-ΕΔ=ΕΓ-ΕΔ άρα ΒΕ=ΕΓ) συνεπώς τα τρίγωνα είναι ίσα. γ) Στο ορθογώνιο τρίγωνο ΑΕΓ έχουμε:

Λ Λ Λ Λ ΑΕΓ+ Γ = 90° άρα ΑΕΓ = 90° - Γ ( 1 )

Λ Λ Λ Αλλά είναι ΑΕΓ = Β+ ΒΑΕ (2) ως εξωτερική γωνία του τριγώνου ΑΒΕ, τότε από τις σχέσεις ( 1 )

Λ Λ Λ και (2) έχουμε: Β+ ΒΑΕ = 90° - Γ ή

Λ Λ Λ Λ Λ Λ Λ Α Β Γ Λ Λ ΒΑΕ = 90° - Γ- Β ή ΒΑΕ=- +- +- -Γ-Β ή

2 2 2 Λ Λ Λ Λ Λ Λ Α Β Β Λ Λ . Λ Α 2Β ΒΑΕ= - + - +- -Β-Β η ΒΑΕ = -

2 2 2 2

ΑΣΚΗΣΗ 2'1 Δίνεται το πολύγωνο ΑΒΓΔΕ (αστέρι). Να απο-

Λ Λ Λ Λ Λ δείξετε ότι: Α+ Β+ Γ+ Δ+ Ε = 180° (Άθροισμα γωνιών τριγώνου- Άθροισμα γωνιών πολυγώνου- Άθροισμα εξωτερικών γωνιών πο­λυγώνου)

Ε

Β Γ ΛΥΣΗ

Ε

Β Γ Έστω ΚΛΜΝΡ το εσωτερικό πεντάγωνο του σχή-ματος, τότε τα ζεύγη των παρακάτω γωνιών είναι ίσα

Λ Λ Λ ΑΚΡ = ΒΚΛ = α ,

Λ Λ Λ ΛΜΓ = ΔΜΝ = γ

Λ Λ Λ

Λ Λ Λ ΒΛΚ = ΓΛΜ = β ,

Λ Λ Λ ΜΝΔ = ΕΝΡ = δ ,

ΝΡΕ = ΑΡΚ = ε ως κατακορυφή γωνίες.

Οι εσωτερικές γωνίες του πενταγώνου ΑΒΓ ΔΕ εί-Λ Λ Λ Λ

ναι οι 1 80° - α , 1 80° - β , 1 80° - γ , 1 80° - δ , Λ

1 80° - ε και έχουν άθροισμα 2 · 5 - 4 = 6 ορθές ή Λ Λ

540° είναι δηλαδή ( 1 8σ -α)+( 1 80° - β )+ Λ Λ Λ

+( 1 80° - γ )+( 1 80° - δ )+( 1 80° - ε )=540° ( 1 ) Λ Λ Λ Λ Λ

Είναι α+ β+ γ+ δ+ ε = 360° (2) ως άθροισμα των εξωτερικών γωνιών του πενταγώνου ΑΒΓ ΔΕ. Από τα εξωτερικά τρίγωνα του «αστεριού» έχουμε τις ισότητες: Λ Λ Λ Α+ α+ ε = 1 80° Λ Λ Λ Β+ β+ α = 1 80° Λ Λ Λ Γ+ γ+ β = 1 80° Λ Λ Λ Δ+ δ+ γ = 1 80° Λ Λ Λ Ε+ ε+ δ = 1 80°

ή

Λ Λ Λ Α = (1 80° - α) - ε Λ Λ Λ Β = (1 80° - β) - α Λ Λ Λ Γ = (1 80° - γ) - β Λ Λ Λ Δ = (1 80° - δ) - γ Λ Λ Λ Ε = (1 80° - ε) - δ

προσθέτοντας κατά μέλη τις τελευταίες ισότητες λαμβάνοντας υπόψη τις σχέσεις ( 1 ) και (2) έχουμε: Λ Λ Λ Λ Λ Α+ Β+ Γ+ Δ+ Ε =

Λ Λ Λ Λ ( 1 80° - α )+( 1 80° - β )+( 1 80° - γ )+( 1 80° - δ )+(

Λ Λ Λ Λ Λ Λ 1 80° - ε )-( α+ β+ γ+ δ+ ε )=540°-360° = 1 80°

ΑΣΚΗΣΗ 3'1 Δίνεται τρίγωνο ΑΒΓ. Στην διάμεσό του ΒΔ

' ' ΒΚ ΒΔ λευ ' παιρνουμε τμημα = 4 και στην π ρα του

ΒΓ ΒΓ τμήμα ΒΛ= - . Α ν Μ είναι το μέσο της

8 πλευράς ΒΓ, να αποδείξετε ότι:

ΔΜ ΑΒ α) ΚΛ = // 4 β) ΚΛ = // S

(Εφαρμογές παραλληλογράμμων)

ΛΥΣΗ Α

α) Έστω Ε και Ζ τα μέσα των ΒΔ και ΒΜ

ΕΥΚΛΕΙΔΗΣ Β ' λθ' τ.2/25

Page 28: Ευκλειδης Β 60

Μαθηματικά για την Α ' Λυκείου

Ε. ΒΚ ΒΔ , , ιναι =- και Ε το μεσο του τμηματος ΒΔ, 4 τότε θα είναι ΒΕ , ΒΓ ΒΚ= - .Ειναι ΒΛ= - και Μ, 2 8 Ζ τα μέσα των τμημάτων ΒΓ και ΒΜ αντίστοιχα, ' θ ' ΒΛ ΒΖ τοτε α ει ναι =-2

Στο τρίγωνο ΒΕΖ έχουμε τα Λ, Κ μέσα των ΒΖ και ΒΕ, συνεπώς είναι ΚΛ=// ΕΖ ( 1 ) 2 Στο τρίγωνο ΒΔΜ έχουμε τα Ε, Ζ μέσα των ΒΔ

και ΒΜ συνεπώς ΕΖ=// ΔΜ (2) 2 Από τις σχέσεις ( 1 ) και (2) έχουμε :

ΔΜ ΕΖ 2 . ΔΜ ΚΛ=/1-=11-- αρα ΚΛ=//-2 2 4

β) Στο τρίγωνο ΑΒΓ έχουμε Μ, Δ μέσα των ΒΓ και ΑΓ, επομένως ΔΜ=// ΑΒ , τότε η σχέση του 2

ΑΒ (α) ερωτήματος γίνεται: ΚΛ=//_1__ ή ΚΛ=// ΑΒ 4 8

ΑΣΚΗΣΗ 4η

Δίνεται ισοσκελές τραπέζιο με μια γωνία του 135°. Αν υ το ύψος του, δ η διάμεσός του και Β, β η μεγάλη και η μικρή βάση του, να αποδείξε­τε: α) Β = β+2υ β) δ+υ = Β (Ιδιότητες ισοσκελών τριγώνων-Ιδιότητες τρα­πεζίων- Ισοσκελές τραπέζιο) ΛΥΣΗ

Α β Β

δ '�Ι: Λ υ "λ Δ Ζ Ε Γ

------ Β ------Έστω το ισοσκελές τραπέζιο ΑΒΓΔ με ΑΒ// ΓΔ και Γ Δ = Β, ΑΒ= β, ΒΕ = ΑΖ = υ και ΚΛ = δ η μεγάλη, η μικρή βάση, το ύψος και η διάμεσός του αντίστοιχα.

Λ Αφού ΑΒΓ = 1 35° και ΒΕ το ύψος του τραπεζίου,

Λ τότε ΕΒΓ = 1 35°-90° = 45°, συνεπώς το τρίγωνο ΒΕΖ είναι ορθογώνιο και ισοσκελές, άρα ΒΕ = ΔΖ =υ. Όμοια στο ορθογώνιο και ισοσκελές τρίγωνο ΑΖΔ είναι ΑΖ =ΔΖ = υ. Από το ορθογώνιο ΑΒΕΖ είναι ΑΒ = ΖΕ = β εΠο­μένως: α) Β= ΔΓ= ΔΖ+ΖΕ+ΕΓ= υ+β+υ = β+2υ β) Από τα (α) ερώτημα είναι β = Β-2υ επομένως η διάμεσος δ του τραπεζίου ΑΒΓ Δ είναι:

δ = Β + β = Β + Β - 2υ = 2Β - 2υ = Β _ υ άρα 2 2 2 δ+υ =Β

ΑΣΚΗΣΗ Sη

Σε ευθεία (ε) παίρνουμε τα σημεία Α, Β, Γ και Δ τέτοια ώστε ΑΒ = ΒΓ = Γ Δ = α. Από τα σημεία Β και Γ φέρνουμε δύο παράλληλες ημιευθείες Βχ και Γy προς το ίδιο ημιεπίπεδο και σ' αυτές παίρνουμε τα σημεία Ε και Ζ τέτοια ώστε ΒΕ=ΓΖ=2α. Να αποδείξετε ότι: α) Η ευθεία ΑΖ διέρχεται από το μέσο του ΒΕ. β) ΑΖ.lΔΕ (Παράλληλες ευθείες- Άθροισμα γωνιών τριγώ­νου)

Α Β

ΛΥΣΗ Υ, z

α) Είναι ΑΓ=ΓΖ=2α, συνεπώς το τρίγωνο ΑΓΖ εί-Λ Λ Λ

ναι ισοσκελές, επομένως ΖΑΒ = ΑΖΓ =ω . Λ Λ Λ

Είναι ΓΖΚ = ΕΚΖ = ω (λόγω των παραλλήλων Λ Λ Λ

ΒΕ και ΓΖ) και ΕΚΖ = ΑΚΒ =ω (ως κατακορυφή Λ Λ Λ Λ

γωνίες), άρα ΚΑΒ = ΓΖΚ = ΑΚΒ =ω δηλαδή το τρίγωνο ΑΒΚ είναι ισοσκελές, επομένως ΚΒ = α συνεπώς η ευθεία ΑΖ διέρχεται από το μέσο του ΒΕ.

ΕΥΚΛΕΙΔΗΣ Β' λθ ' τ.4/26

Page 29: Ευκλειδης Β 60

Μαθηματικά για την Α ' Λυκείου

Λ β) Στο τρίγωνο ΑΚΒ η Β ι είναι εξωτερική γωνία

Λ •

ΒΛ

2 Λ • Λ Β ι λέ . του, τοτε ι = ω η ω=- . Στο ισοσκε ς τρι-2

Λ Λ Λ

Είναι και ΓΕ = Γ Δ γιατί είναι εφαπτόμενα τμήματα άρα τα σημεία Ρ και Γ ισαπέχουν από τα άκρα του τμήματος ΕΔ, συνεπώς η ευθεία ΡΓ είναι μεσοκά­θετος του τμήματος ΕΔ.

γωνο ΒΕΔ (ΒΔ=ΔΕ=2α) είναι ΒΕΔ = ΕΔΒ = φ γ) Επειδή το τρίγωνο ΔΡΕ είναι ορθογώνιο και ι-Λ

και η Β 2 είναι εξωτερική γωνία του, επομένως

τέλος στο τρίγωνο ΚΕΤ Λ Λ Λ Λ Β ι B z

ω + φ =- + - =90° άρα ΑΖl_ΔΕ. 2 2 ΑΣΚΗΣΗ 6η

Λ

είναι

Δίνεται ορθογώνιο τρίγωνο ΑΒΓ ( Α =90°) και ο εγγεγραμμένος κύκλος του. Α ν Δ, Ε, Ζ είναι τα σημεία επαφής του κύκλου με τις πλευρές ΒΓ, ΑΓ και ΑΒ αντίστοιχα και ΕΡ η κάθετη στην ΖΔ, να αποδείξετε ότι:

Λ α) ΑΡΕ =45° β) Η ευθεία ΡΓ είναι μεσοκάθετος της ΔΕ

Λ γ) ΑΡΓ =90° ( Γωνία χορδής και εφαπτομένης- Ιδιότητα με­σοκαθέτου- εγγράψιμα και εγγεγραμμένα τε­τράπλευρα)

ΛΥΣΗ Β

α) Τα ΑΖ και ΑΕ είναι εφαπτόμενα στο κύκλο τμήματα, συνεπώς είναι ίσα, άρα το τρίγωνο ΑΖΕ είναι ορθογώνιο ισοσκελές. Το τετράπλευρο ΑΕΡΖ είναι εγγράψιμο γιατί Λ Λ Λ Λ Λ Λ Α+ Ρ = 1 80° άρα ΑΕΖ = ΑΖΕ = ΑΡΕ = ω =45°

Λ Λ Λ β) Είναι ΑΕΖ = ΕΔΖ = ω = 45° ( από θεώρημα χορδής και εφαπτομένης ), συνεπώς το τρίγωνο ΔΡΕ είναι ορθογώνιο και ισοσκελές, άρα ΡΕ = Ρ Δ.

σοσκελές η ΡΓ ως μεσοκάθετος είναι και διχοτό-Λ Λ

μος της γωνίας ΕΡΔ άρα ΕΡΓ =45°.

ΑΣΚΗΣΗ 7η

Από το σημείο τομής Α δύο κύκλων φέρνουμε τρεις ευθείες που τέμνουν τον ένα κύκλο στα σημεία Δ, Ε και Ζ και τον άλλο κύκλο στα ση­μεία Κ, Λ και Μ. Να αποδείξετε ότι τα τρίγωνα ΔΕΖ και ΚΛΜ είναι όμοια. (Εγγεγραμμένα τετράπλευρα- Ομοιότητα τρι­γώνων)

ΛΥΣΗ Δ Α κ

z Μ

Από τα εγγράψιμα τετράπλευρα ΑΔΕΖ και ΑΚΛΜ έχουμε τις ισότητες των γωνιών

Λ Λ Λ Λ Λ ΕΖΔ = ΕΑΔ = ΛΑΜ = ΛΚΜ = φ και

Λ Λ Λ Λ Λ ΕΔΖ = ΕΑΖ = ΚΑΛ = ΚΜΛ = ω

Τότε τα τρίγωνα ΔΕΖ και ΚΛΜ έχουν από δύο γωνίες ίσες, συνεπώς είναι όμοια.

ΑΣΚΗΣΗ 8'1 Λ Λ

Δίνεται ισοσκελές τρίγωνο ΑΒΓ με Β =Γ =40° και ΒΔ η διχοτόμος του. Στην πλευρά ΒΓ παίρ­νουμε σημείο Ε τέτοιο ώστε να είναι ΒΔ = ΒΕ. Ν α αποδείξετε ότι: α) Το τρίγωνο ΔΕΓ είναι ισοσκελές. β) Το τετράπλευρο ΑΔΕΒ είναι εγγράψιμο γ) Το τρίγωνο ΑΔΕ είναι ισοσκελές δ) ΑΔ+ΒΔ =ΒΓ

ΕΥΚΛΕΙΔΗΣ Β' λθ ' τ.2/27

Page 30: Ευκλειδης Β 60

Μαθηματικά για την Α' Λυκείου

(Ιδιότητες ισοσκελούς τριγώνου-Άθροισμα γω­νιών-εγγράψιμα τετράπλευρα)

ΛΥΣΗ Α

~ Β Ε Γ

Λ α) Η γωνία Α του τριγώνου ΑΒΓ είναι:

Λ αφού η ΒΔ είναι διχοτόμος, τότε ΒΔΓ = 100° + 20° = 1 20° ως εξωτερική γωνία του τριγώνου ΑΒΔ

Λ Στο ισοσκελές τρίγωνο ΔΒΕ είναι Β ι =20°, τότε θα

είναι ΒΕΔ = ΒΔΕ = 1 800 - 200 = 80° άρα 2 Λ Λ Λ

ΕΔΓ = ΒΔΓ- ΒΔΕ = 120° -80°=40° είναι και Λ Γ =40° συνεπώς το τρίγωνο ΔΕΓ είναι ισοσκελές. β) Στο τετράπλευρο ΑΔΕΒ είναι

Λ Λ ΒΑΔ+ ΒΕΔ =100°+80°= 1 80° συνεπώς είναι εγ-γράψιμο. γ) Από το εγγράψιμο τετράπλευρο ΑΔΕΒ έχουμε

Λ Λ Λ Λ ΔΑΕ = Β ι =20° και ΑΕΔ = Β 2 =20°

Λ Λ άρα ΔΑΕ = ΑΕΔ συνεπώς το τρίγωνο ΑΔΕ είναι ισοσκελές. δ) Είναι

ΑΔ+ΔΒ= =ΔΕ+ΒΕ=(από τα ισοσκελή τρίγωνα ΑΔΕ και ΒΔΕ) =ΕΓ +ΒΕ = ΒΓ ( από το ισοσκελές ΔΕΓ)

ΑΣΚΗΣΗ 9η

Δίνεται τρίγωνο ΑΒΓ και Μ τυχαίο σημείο της πλευράς του ΒΓ. Έστω Δ, Ε τα μέσα των πλευ­ρών του ΑΒ και ΑΓ αντίστοιχα. Η ευθεία ΕΜ τέμνει την ΑΒ στο σημείο Λ και η ευθεία ΜΔ την ΑΓ στο σημείο Κ. Αν είναι ΑΘ//ΔΕ τότε να αποδείξετε ότι: α) Τα τρίγωνα ΑΘΔ και ΔΒΜ είναι ίσα.

ΚΑ ΒΜ β) ΚΕ

Ε

γ) ΑΜ//ΚΛ (Θεώρημα Θαλή-Εφαρμογές παραλληλογράμμου

-Ομοιότητα - Ισότητα τριγώνων) ΛΥΣΗ

κ

Λ α) Τα Δ και Ε είναι τα μέσα των πλευρών του τρι­γώνου ΑΒΓ συνεπώς ΔΕ//ΒΓ. Αφού ΑΘ//ΔΕ τότε

Λ Λ θα είναι και ΑΘ//ΒΓ άρα Α ι = Β ως εντός εναλ-

Λ Λ λάξ γωνίες, Δ ι = Δ 2 ως κατακορυφή γωνίες και ΑΔ=ΒΔ, συνεπώς τα τρίγωνα ΑΘΔ και ΒΔΜ είναι ίσα. β) Τα τρίγωνα ΚΑΘ και ΚΔΕ είναι όμοια τότε ΚΑ ΑΘ , , , -- = -- και απο την ισοτητα των τριγωνων ΚΕ ΔΕ

ΑΘΔ και ΔΒΜ είναι ΑΘ=ΒΜ, οπότε η σχέση γί­ΚΑ ΒΜ νεται-- = --ΚΕ ΔΕ

γ) Από την ομοιότητα των τριγώνων ΛΒΜ και , ΛΜ ΒΜ ΛΔΕ εχουμε --

ΑΕ ΔΕ και με βάση το β) ερώτημα έχουμε ΚΑ ΛΜ ' ' Θ ' Θ λ ' ' ΚΕ

= ΑΕ

οποτε απο το � εωρημα του � α η ει-

ναι ΑΜ/ΙΚΛ

ΑΣΚΗΣΗ 10η

Δίνεται τρίγωνο ΑΒΓ. Αν η διάμεσός του ΒΜ και το ύψος του Γ Δ είναι ίσα και Ν η προβολή του σημείου Μ στην πλευρά ΑΒ, τότε: α) Να αποδείξετε ότι ΜΝ=// Γ

Δ 2

β) Να υπολογίσετε σε μοίρες το μέτρο της οξεί­ας γωνίας που σχηματίζουν η διάμεσος ΒΜ και το ύψος ΓΔ του τριγώνου ΑΒΓ.

ΕΥΚΛΕΙΔΗΣ Β' λθ ' τ.4/28

Page 31: Ευκλειδης Β 60

Μαθηματικά για την Α' Λυκείου

(Εφαρμογές παραλληλογράμμων- Παράλληλες

ευθείες) ΛΥΣΗ

Α

Β Γ Έστω Κ το σημείο τομής της διαμέσου ΒΜ και του ύψους Γ Δ. α) Στο τρίγωνο ΑΔΓ είναι το Μ μέσο της πλευράς ΑΓ και ΜΝ/ΙΓΔ ( κάθετες στην ίδια ευθεία ΑΒ) άρα και το σημείο Μ θα είναι μέσο της ΑΔ, τότε

ΓΔ MN=II -2 β , , , I

ΓΔ, ) Απο το (α) ερωτημα εχουμε ΜΝ= /- η 2

ΒΜ , θ ΜΝ=/1 -- , συνεπως στο ορ ογώνιο τρίγωνο 2

Λ ΒΜΝ η γωνία ΝΒΜ =30° άρα η γωνία της διαμέ-

Λ σου ΒΜ και του ύψους Γ Δ είναι: ΔΒΚ = 60°

ΑΣΚΗΣΗ l lη

Δίνεται τετράγωνο ΑΒΓ Δ. Έστω τυχαίο σημείο Ε της πλευράς του Γ Δ και ΑΖ η διχοτόμος της

Λ

γωνίας ΒΑΕ . Φέρνουμε την ΔΙ κάθετη στην ΑΖ που την τέμνει στο σημείο Θ και την ΑΕ στο σημείο Η. Να αποδείξετε ότι: α) Το τρίγωνο ΗΔΕ είναι ισοσκελές β) Τα τρίγωνα ΑΔΙ και ΒΑΖ είναι ίσα. γ) ΑΕ=ΒΖ+ΔΕ (τετράγωνο-Παράλληλες ευθείες- Ισότητα τρι­γώνων)

ΛΥΣΗ

α) Στο τρίγωνο ΑΗΙ η ΑΘ είναι διχοτόμος και ύψος, συνεπώς το τρίγωνο είναι ισοσκελές, άρα Λ Λ Λ Λ Ι ι = Η ι , ακόμη Η ι = Η 2 ως κατακορυφή γωνίες

Λ Λ και Ι ι = Δ ι ως εντός εναλλάξ γωνίες που σχηματί-ζονται από τις παράλληλες πλευρές του τετραγώ-

Λ Λ νου. Από τις παραπάνω σχέσεις έχουμε Δ ι = H z

συνεπώς το τρίγωνο ΗΔΕ είναι ισοσκελές. β) Τα τρίγωνα ΑΔΙ και ΒΑΖ έχουν ΑΒ=ΑΔ (πλευρές τετραγώνου), είναι ορθογώνια και έχουν Λ Λ Ι ι = Ζι (πλεύρές κάθετες) συνεπώς είναι ίσα. γ) Έχουμε: ΑΕ=ΑΗ+ΗΕ=

= AI+HE (από το ισοσκελές τριγωνο AHI)

= ΒΖ+ΗΕ (από την ισότητα των τριγώνων ΑΔΙ, ΒΑΖ)

= ΒΖ+ ΔΕ (από το ισοσκελές τρίγωνο ΗΔΕ)

ΑΣΚΗΣΗ 12η

Δίνεται ισόπλευρο τρίγωνο ΑΒΓ. Προεκτείνου­με τη ΓΒ κατά τμήμα ΒΔ = ΒΓ και τη ΔΑ κατά τμήμα ΑΕ = ΔΑ. α) Να αποδείξετε ότι το τρίγωνο ΑΔΓ είναι ορ­θογώνιο. β) ΑΒ // ΔΕ. (Ιδιότητες ισοσκελών τριγώνων- Άθροισμα γω­

νιών τριγώνου - Παράλληλες ευθείες) ΛΥΣΗ

Β Γ α) Αφού ΑΒ = ΔΒ το τρίγωνο ΑΒΔ είναι ισοσκε-

Λ λές με γωνία ΑΒΔ = 1 80°-60°= 1 20° συνεπώς έχει

Λ Λ Δ = ΔΑΒ =30°.

Λ Λ Λ Η

Λ γωνία ΔΑΓ είναι:

ΔΑΓ = ΔΑΒ + Α =30° +60°=90° συνεπώς το τρί-γωνο ΔΑΓ είναι ορθογώνιο. β) Το τρίγωνο ΔΓΕ είναι ισοσκελές γιατί η Γ Α εί-

Λ Λ ναι διάμεσος και ύψος άρα Δ = Ε =30°, τότε

ΕΥΚΛΕΙΔΗΣ Β ' λθ ' τ.2/29

Page 32: Ευκλειδης Β 60

Μαθηματικά για την Α' Λυκείου

Λ Γ ι =60°. Οι ευθείες ΑΒ και ΓΕ τεμνόμενες από την ΑΓ σχηματίζουν τις εντός εναλλάξ γωνίες ίσες, συνεπώς είναι παράλληλες.

ΑΣΚΗΣΗ 13η Λ Λ Λ

Οι γωνίες Α , Β και Γ είναι ανάλογες προς τους αριθμούς 5, 3 και 2 αντίστοιχα. α) Να υπολογίσετε σε μοίρες το μέτρο των γω-

Λ Λ Λ

νιών Α , Β και Γ . β) Α ν η διάμεσος ΑΔ τέμνει την διχοτόμο ΓΕ στο σημείο Ζ, τότε να αποδείξετε ότι το τρίγωνο ΑΕΖ είναι ισοσκελές (Ιδιότητες ισοσκελών τριγώνων-Άθροισμα γω­

νιών τριγώνου) ΛΥΣΗ

Λ Λ Λ · α) Αφού οι γωνίες Α , Β και Γ είναι ανάλογες προς τους αριθμούς 5, 3 και 2 ισχύει: τότε

Λ Λ Λ Λ Λ Λ Α = Β = Γ = Α+ Β+ Γ = 1 80°

= 1 8ο 5 3 2 5 + 3 + 2 1 0 Λ Λ Λ

άρα είναι

Α = 1 8° , Β = 1 8° , Γ = 1 8° συνεπώς έχουμε 5 3 2

Λ Λ Λ Α =90° , Β =54° και Γ =36°, δηλαδή το τρίγωνο είναι ορθογώνιο και έχουμε το παρακάτω σχήμα.

Γ

Α Ε Β β) Αφού η ΓΕ είναι διχοτόμος τοί> τριγώνου τότε:

Λ Λ Γ ι = Γ 2 = 1 8°· Επειδή η διάμεσος ΑΔ χωρίζει το ορθογώνιο τρίγωνο σε δύο ισοσκελή τρίγωνα

Λ Λ Λ έχουμε Αι = Γ ι = 36° άρα Α2 =90°-36°=54° και

Λ

Β =90°-36°=54° συνεπώς Λ η γωνία Ε ι =54°+ 1 8° = 72° ως εξωτερική γωνία

του τριγώνου ΓΕΒ. Τέλος είναι

Λ Λ Λ ΑΖΕ = 1 80°- Α2 - Ε ι = 1 80°-54°-72° = 54°

Λ Λ είναι δηλαδή Α2 =ΑΖΕ δηλαδή το τρίγωνο ΑΕΖ είναι ισοσκελές.

ΤΟ ΓΡΑΦΕΙΟ ΔΙΑΣΥΝΔΕΣΗΣ ΜΕ ΤΗΝ ΑΓΟΡΑ ΕΡΓ ΑΣΙΑΣ

της Ελληνικής Μαθηματικής Εταιρείας Ε.Μ.Ε.

Το Δ.Σ. της Ελληνικής Μαθηματικής Εταιρείας και το Γραφείο Διασύνδεσης με την αγορά εργασί­

ας της Ε.Μ.Ε διοργανώνει εκδήλωση που θα γίνει στο Αμφιθέατρο Δρακόπουλου του Πανεπιστημίου Αθηνών (Πανεπιστημίου 30) στις 2 Μα'ϊου 2006 ημέρα Τρίτη και ώρα 7.00 μ.μ. με θέμα:

«Διαδικασίες Επιλογής Εκπαιδευτικών στην εκπαίδευση: Μορφή - Περιεχόμενο - Προτάσεις.

Η Εμπειρία του ΑΣΕΠ» Στην εκδήλωση θα χαιρετήσει ο Α ' Αντιπρόεδρος της ΕΜΕ, Επίκουρος Καθηγητής του Πανεπι-

στημίου Αθηνών κ. Γεώργιος Δημάκος. Το θέμα θα αναπτύξουν: Ο καθηγητής του Πανεπιστημίου Αιγαίου κ. Φραγκίσκος Καλαβάσης και

Ο λέκτορας του Πανεπιστημίου Ιωαννίνων Θωμάς Μπάκας Στόχος της εκδήλωσης είναι να διαμορφώσει η ΕΜΕ συγκεκριμένες προτάσεις για τη βελτίωση

των διαδικασιών του ΑΣΕΠ και να εκπονήσει επιμορφωτικά προγράμματα για την διδασκαλία των

Μαθηματικών.

ΕΥΚΛΕΙΔΗΣ Β' λθ ' τ.4/30

Page 33: Ευκλειδης Β 60

.......... ,.. I'� W 1'�111 I'• Α•ά•

ΑΛfΕΒΡΑ

Η εκθετική συνάρτηση

Μέχρι τώρα ο εκθέτης σε μια δύναμη μπορούσε να είναι οποιοσδήποτε ακέραιος αριθμός, αρκεί βέ­βαια η βάση να ήταν διάφορη του μηδενός. Στη συνέχεια ο εκθέτης θα μπορεί να είναι ρητός

� όπου μ,ν ακέραιοι με ν>Ο καθώς για α>Ο θα ο­ν

μ ρίσουμε α; =� . Ας δούμε εδώ ένα παράδοξο:

� 2·! -1 = (-1) 1 = (- 1) 2 = (-1) 2 = I I δηλ.

= [ (- 1)2 ]2 = (+1)2 =.J+Ϊ = +1

-1 = +1 <:::> 2 = 0 ! ! ! 2

Που έγινε το λάθος; [ ( - 1 ) 1 δεν γράφεται ( - 1) 2 ] . Απλά κάθε φορά που γράφουμε ως εκθέτη ένα κλάσμα δεν πρέπει να ξεχνάμε ότι παριστά ρίζα και επομένως η βάση θα βρεθεί ως υπόρριζος πο­σότητα και άρα πρέπει να είναι θετική για αυτό για να ορίσουμε μια συνάρτηση με μορφή f(x) = αχ , χ Ε JR πρέπει αρχικά α>Ο, η περίπτωση δε για α = 1 οδηγεί στη σταθερή συνάρτηση f(x) = 1x = 1 . Για τη συνάρτηση f (x) = αχ με χ Ε JR και α>1 ή 0<α<1 ισχύουν τα εξής: i) αχ>Ο για κάθε χ Ε JR , οπότε η γραφική της πα­ράσταση βρίσκεται πάνω από τον άξονα χ ' χ. ii) Η γραφική παράσταση διέρχεται πάντα από το σημείο (0, 1 ) (δηλ. το 1 του y 'y). iii) Αν 0<α<1 τότε η f είναι γνησίως φθίνουσα ενώ αν α> 1 τότε η f είναι γνησίως αύξουσα .

Π. Χριστόπουλος - Θ. Χριστόπουλος

(Αυτό μας βοηθάει να κατανοήσουμε ότι δεν υ­πάρχουν διαφορετικά χ που να έχουν ίδια εικόνα) δηλ. χ1 * χ2 � f (x 1 ) * f (x2 ) ή ισοδύναμα: f (χι ) = f (χ2 ) � χι = χ2 Για να κατανοήσουμε τη μονοτονία της εκθετικής ας δούμε τα παρακάτω παραδείγματα:

( 8 )χ+\ ( 7 )

3χ-5 ( 8 )χ+\ ( 8 )5-

3 χ 1 ) - < - <:::> - < -7 8 7 7

δ ' 8 1 ' ' και επει η - > επεται οτι 7

χ + 1 < 5 - 3χ <:::> 4χ < 4 <:::> χ < 1 .

2) (�)4-χ > (�yx-S

<::> 2χ - 5 > 4 - χ <::>

<:::> 3χ > 9 <:::> χ > 3

3) Για ποιες τιμές του α η συνάρτηση f με f (χ) = ( 2α - 1 )

χ είναι γνησίως φθίνουσα. α + 1

Λύση: ο 2α - 1 1 δ λ 2α - 1 Πρέπει <--< η . -->0 (i) και

α + 1 α + 1 2α - 1 1 ( " " ) --< 11 α + 1

(i) (2α - 1)( α + 1) > Ο� α < - 1 ή α > .!.. 2

2α - 1 1 α - 2 ----< 0<:::>--< 0<:::> (ii) α + 1 1 α + 1 (α - 2)(α + 1) <:::> - 1 < α < 2

. λ ' 1 2 οποτε τε ικα - < α < . 2

- -1 .1z 2 +

��-----��_-_-_;! ---

ΕΥΚΛΕΙΔΗΣ Β ' λθ ' τ.4/31

Page 34: Ευκλειδης Β 60

Μαθηματικά Β' Λυκείου

• Μία εξίσωση (ανίσωση) με τον άγνωστο χ στον εκθέτη λέγεται εκθετική εξίσωση ( ανίσωση). Να λυθούν οι παρακάτω εκθετικές εξισώσεις -ανισώσεις: 1 ) 2X+I + 5 · 2χ = 56 2) 3Χ+2 - 9χ = 3χ - 32 3) 3χ - 4./i + 3 = 0

χ' -2χ 4) (χ2 - 5χ + 6) = 1

5) 2 · 5χ-2 - 3 · 2χ-3 = 1 2 · 5χ-3 - 2χ

6) 1 88-4χ = { 54.J2)3x-2 7) .J2ημ2χ = �

8) 9Jx' +3 - 31+Jx' +3 = 54 9) e2x + e :s; ex + ex+l Λύση ί) 2X+I + 5 · 2χ = 56 <::::> 2χ · 2 + 5 · 2χ = 56 <=> 7 . 2χ = 56 <=> 2χ = 8 <=> 2χ = 23 <=> χ = 3 . ii) 3χ+2 - 9χ = 3χ - 32 <=> 3χ · 32 - (32 )χ = 3χ - 9 <=> 9 · 3χ - 32χ - 3χ + 9 = 0 <=> 8 · 3χ - (3χ )2 + 9 = 0 θέτουμε 3χ = y. Οπότε 8 · y - y2 + 9 = Ο <=> y2 - 8y - 9 = 0 , Δ = 64 + 36 = 100 , y1 = -1 απορ. Υ 2 = 9 άρα 3χ = 9 <=> χ = 2 .

iii) 3χ - 4J3 + 3 = 0 <::::> .fj - 4J3 + 3 = 0 χ ( 2 )χ χ

θέτουμε .fjx = y οπότε y2 - 4y + 3 = Ο και άρα Υ ι

= 1 , Υ2 = 3 δηλ. J3x = l <=> x = O ή .fjx = 3 <=> { .fjγ = .fj2 <=> Χ = 2 .

iv) (χ2 - 5χ + 6)χ' -2χ = 1 lη περίπτωση :

2 2 2 (5 + J3

χ - 5χ + 6 = 1 <=> χ - 5χ + 5 = 0 <=> χ1 2 = r;;

2η περίπτωση :

χ2 - 2χ = Ο και x2 - 5x + 6 :;t 0 οπότε χ(χ - 2) = 0

· 5 - ν5 2

χ 1 = Ο ή χ2 = 2 απορρίπτεται διότι μηδενίζει την βάση : χ - 5χ + 6 . ν) 2 · 5χ · 5-2 - 3 · 2χ · Τ3 = 12 · 5χ · 5-3 - 2χ

(%) χ = (%)4 <=> χ = 4

νί) l 88-4x = { 54.J2)3x-2 <=>

<=> (2 . 32 )8-4χ = ( 2 . 33 . .J2)3x-2 <=> .J2 . 32 = .J2 . 33 <=> ( 2 )8-4χ ( 3 )3χ-2 <=> ( .J2 . 3 )2·(8-4χ ) = ( .J2 . 3 )3(3χ-2) <=>

2 · (8 - 4χ) = 3(3χ - 2) <=> 16 - 8χ = 9χ - 6 <=> 22 <=> 17χ = 22 <=> χ = -17

I I �ημ2χ 4� -ημ2χ - ι vii) ν L. = t2 <=> 22 = 24 <=> ημ2χ = - <=>

2χ = 2κπ + � ή 2χ = 2κπ + 5π 6 6

π ' 5π '71 Χ = ΚΠ + - η X = KΠ + - , K E tL.. . 1 2 12

viii) 9Jx' +3 - 31+Jx ' +3 = 54 <=>

2

32.Jx' +3 - 3 · 3Jx' +3 = 54 , θέτουμε 3Jx' +3 = Υ > Ο y2 - 3y - 54 = 0 , Δ = 9 + 2 16 = 226

6 ' 9 ' 3Jx ' +3 9 Υ ι = - απορ. η Υ2 = οποτε = <=> �χ2 + 3 = 2 <=> χ2 + 3 = 4 <=> <=> χ2 - 1 = 0 <=> χ = ±1 ix) e2x + e :s; ex + ex+l <=> (ex )2 + e :s; ex + e · ex θέτουμε ex = y ( 1 ) οπότε y2 + e :S; y + e · y <=> y2 - (1 + e )y + e :S; Ο , y ι = Ι , Υ2 = e

1 e + ? - ? +

α = l>Ο άρα l :S; y :S; e<::::> l :S; ex :S; e <::::> O :S; x :S; l . ( I )

Αν δοθεί η εξίσωση 2χ = 8 τότε 2χ = 23 <=> χ = 3 αντίθετα αν δοθεί 2χ = 5 τότε πρέπει να γράψουμε

ΕΥΚΛΕΙΔΗΣ Β ' λθ ' τ.4/32

Page 35: Ευκλειδης Β 60

Μαθηματικά Β ' Λυκείου

το 5 σαν δύναμη του 2 ο εκθέτης που αναζητάμε για το 2 συμβολίζεται Rog2 5 .

Γενικά αν θ>Ο τότε ορίζουμε Rοgαθ = χ <=:> θ = αχ με 0 < α < 1 ή α> 1 . Ως βάση α χρησιμοποιούμε συνήθως το 10 και τον αριθμό e (όπου e = 2, 7 1 828 ) στην περίπτωση που η βάση είναι 1 - δεν αναγράφεται δηλ. Rog10θ γρά­φεται Rogθ ενώ στην περίπτωση που η βάση είναι e αντί να γράψουμε Rogeθ γράφουμε Rnθ και δια­βάζουμε Νεπέρειος λογάριθμος ή φυσικός Λογά­ριθμος του θ ή Ελ-Εν-θ δηλ. Rnθ = χ <:::::> θ = ex

f(x) = Rnx , Α = (Ο, +οο) .

f(Χ)=Ιπχ

Επομένως: Rogαx = y <:::::> χ = α Υ <:::::> χ = αfog.x οπό­τε Rogx = y <:::::> χ = 10Υ και Rnx = y <:::::> χ = eY � χ = e enx (Αυτή η ταυτό­τητα θα αποδειχθεί πολύ χρήσιμη) Άμεσα συνεπάγονται τα παρακάτω: ί) Rogα 1 = Ο ίί) Rοgαα = 1 ίίί) Rοgααχ = χ Επίσης αποδεικνύονται οι παρακάτω ιδιότητες

ίν) Rogα ( :� ) = Rοgαθι - Rοgαθ2 οπότε

Rogα (.;;) = -Rοgαθ νί) Rοgαθν = ν · Rοgαθ η τελευταία για ρητό εκθέτη γίνεται

Rogα {/θ = _!_Rοgαθ . ν

Ο τύπος «αλλαγής βάσης» είναι :

Rοgβθ = Rοgαθ και άμεσα απ' αυτόν για θ = α Rοgαβ

προκύπτει: Rοgβα · Rοgαβ = 1 .

Επίσης ισχύει : αfοgβ = βeοgα

[ Rog ( αfοgβ ) = Rog (βeogα ) J Rogβ · Rogα = Rogα · Rogβ (όπου θ ι , θ2, θ, α, β θετικοί και α, β * + 1 )

Ασκήσεις στους λογαρίθμους

1) Να λυθούν οι λογαρ. εξισώσεις χ fog9 + 3eogx = 1 oeogx + 32+fogx

Χ eog9 = 32 . 3eogx _ 3eogx + 9

Χ 2fog3 = g . 3eogx + 9

(Χ fog3 = 3fogx = Υ> Ο)

(3eogx )2 - 8 · 3fogx - 9 = 0

Υ2 - 8 · g

fogx - 9 = 0

y2 - 8 · y - 9 = 0 Δ = 64 + 36 = 1 00

Υ ι = - 1 απορ. , Υ2 = 9

οπότε 3togx = 32 <:::::> Rogx = 2 <:::::> χ = 102

2) Να βρεθεί το Πεδίο Ορισμού της f αν: f (x) = ../1 - Rn(x - 1) Πρέπει χ- 1>0 για να έχει νόημα ο λογάριθμος και 1 - Rn(x - 1) � Ο σαν υπόρριζος ποσότητα απ' αυ-

τές προκύπτει: {χ > 1

<:::::> {χ > 1

Rn(x - 1) � 1 χ - 1 � e

άρα 1 < χ � 1 + e ή Α = (1, e + 1] . 3) Προσοχή στις ιδιότητες των λογαρίθμων. Τα ο­ρίσματα των λογαρίθμων πρέπει να είναι πάντα θετικά

π.χ. δίνεται η συνάρτηση f(x) = Rn χ- 2 ( 1 ) χ - 1

Το πεδίο ορισμού της είναι: χ - 2 -- > ο <:::::> (χ - 2)(χ - 1) > ο χ - 1 <=:> χ < 1 ή χ > 2 δηλ. Α = (-οο, 1)υ (2,+οο) . Άραγε η συνάρτηση ( 1 ) μπορεί να γραφτεί f(x) = Rn(x - 2) - Rn(x - 1) .

Ναι αν χ-2 > Ο και χ- 1 > Ο δηλ. χ > 2. 1 - χ

4) Δείξτε ότι η συνάρτηση f(x) = Rn-- είναι 1 + χ

περιττή . 1 - χ Για το πεδίο Ορισμού της f πρέπει -- > Ο 1 + χ

(1 - χ )(1 + χ) > Ο <:::::> -1 < χ < 1 ή Α = ( - 1 , 1 ) οπότε για κάθε χ Ε Α � -χ Ε Α θα εξετάσουμε τώρα αν

ΕΥΚΛΕΙΔΗΣ Β' λθ' τ.4133

Page 36: Ευκλειδης Β 60

Μαθηματικά Β' Λυκείου

f (-x) = -f (x) ή ΑΣΚΗΣΕΙΣ ΠΡΟΣ Λ ΥΣΗ 1 + χ 1 - χ f (-x) + f (x) = Ο <:=> ln-- + ln-- = 0 <:::::> 1 - χ 1 + χ

fn ( 1 + χ · 1 - χ ) = Ο <:::::> fn1 = Ο ισχύει. · 1 - χ 1 + χ

5) Να υπολογιστεί η τιμή της παράστασης log341!og, 9

log 41!og, 9 = fog 2Hog, 9 = fog 21!og, 9' = είναι 3 3 3

= fog392 = log3 34 = 4 6) Για ποιες τιμές του θ η εξίσωση χ2 - 2xlnθ + 3fnθ - 2 = 0 έχει 2 ρίζες πραγματικές και άνισες. Πρέπει Δ > Ο <:::::> (2fnθ)2 - 4(3fnθ - 2) > Ο <:::::> 4fn 2θ - 12fnθ + 8 > Ο fn 2θ - 3fnθ + 2 > Ο θέτουμε fnθ = y y2 - 3y + 2 > 0 Δ = 9 - 8 = 1 � y1 = 1 Υ2 = 2 άρα y<1 ή y>2 οπότε fnθ < 1 ή fnθ > 2 <:::::> Ο < θ < e ή θ > e2 . 7) Αν f (χ) = (χ - 1)χ . Να βρεθεί το πεδίο ορισμού και να λυθεί η aνίσωση f (x) > ex πρέπει χ - 1 > Ο <:::::> χ > 1 οπότε χ - 1 = el!n ( x-Ι)

f (χ) = ( ein ( x-1 ) )χ = exfn ( x-1) οπότε

f (χ) > ex <:::::> exl!n ( x-Ι) > ex <:::::> xln(x - 1) > χ <:::::> fn(x - 1) > 1 <:::::> χ - 1 > e <:::::> χ > 1 + e

Α. Να λυθούν οι εξισώσεις: 1 . log(4x+l )=log3+log(x2 - 1) 2. (χ5 )2+Ιη χ' = es 3 . ln(2e2x - 3ex + 2) = ln 2 + ln(3ex - 1)

4. ιn [_C__!_) = 2 ln 2 ex + 5

5 . 2Ι+JΧ + 2.,J4;. = 23 Β. Να λυθούν οι ανισώσεις: 1 . x log x ::; 0, 01χ3

2. ιn [ e2x + 1 ) > ο 3ex - 1

3 . .Jx2 + 2x - 2 ln x + 2 > x + 1 Γ. Να λυθούν τα συστήματα: {4χ · 8Υ = 32 1 . 3Υ

- = 3 9χ { log χ + 2 log y = 3

2. log2 χ + log2 y = 5

Δ. Αν οι αριθμοί: ln 3, tn.J2x+1 + 3χ+2 , ln 2x+Ι είναι διαδοχικοί όροι Αριθμητικής προόδου, δείξτε ότι χ = - 2.

του Γιώργου Κατσούλη ΕΠΑΝΑΛΗΠΤΙΚΕΣ ΑΣΚΗΣΕΙΣ

Άσκηση lη Δίνεται τρίγωνο ΑΒΓ εγγεγραμμένο σε κύκλο (O,R). Αν ΑΒ2 + ΑΓ2 = ΒΓ2 + 4(ΑΒΓ) (1) να

-βρείτε το μήκος του τόξου ΒΓ .

Λύση

Από το νόμο των συνημιτόνων έχουμε ΒΓ2 = ΑΒ2 + ΑΓ2 - 2ΑΒΑΓσυνΑ (2)

Επίσης (ΑΒΓ) = _!_ ΑΒ · ΑΓημΑ (3) 2

Οπότε η ( 1 ) λόγω των (2), (3) γράφεται: � + _w2 = � + � --2AB · ΑΓσυνΑ + 2ΑΒ · ΑΓημΑ <:::::> 2ΑΒ · ΑΓσυνΑ = 2ΑΒ · ΑΓημΑ <:::::> ημΑ = συν Α . Άρα Α = 45° , οπότε Br = 90° .

π · R · 90° πR Επομένως RΒΓ = 1 80

<=> f8r = τ ·

ΕΥΚΛΕΙΔΗΣ Β' λθ' τ.4/34

Page 37: Ευκλειδης Β 60

Μαθηματικά Β ' Λυκείου -------------

Άσκηση 2η

Από σημείο Α εκτός κύκλου (O,R) φέρνουμε τέ­μνουσα ΑΒΓ ώστε ΑΒ = 2ΒΓ. Αν ΟΑ = R.Jϊ3 τότε: α) Να αποδείξετε ότι ΒΓ = λι. β) Να βρείτε το λόγο Εκυκλου (Ο,R) . (ΑΟΒ)

Λύση

α) Έστω ΒΓ = χ, οπότε ΑΒ = 2χ.

ΑΒ · ΑΓ = Α02 - R2 � Είναι

2χ · 3χ = (R .Ji3)2 - R 2 � · · · � Χ2 = 2R 2 � x = R.Ji . Άρα ΒΓ = χ = R J2 = λ4 • β) Έχουμε Εκυιcλου (Ο, R) = πR 2 ( 1 )

1 1 ,.f (ΑΟΒ) = 2ΑΒ - ΟΗ = j ,L BΓ · OH =

ΒΓ · ΟΗ = λ4 · α4 = R J2 · R J2 2

οπότε: (ΑΟΒ) = R 2 (2) .

Α , ( 1 ) (2) , , Β.αιοο,(Ο,R) 11R2 πο τις , προκυπτει οτι (ΑΟΟ)

= R2

=π ·

Άσκηση 3η

Δίνεται αμβλυγώνιο τρίγωνο ΑΒΓ με Α > 90° 15 και ΑΒ = 5, ΑΓ = 3, (ΑΒΓ) = - . 4 ......

α) Να υπολογίσετε τη γωνία Α . β) Να βρείτε το ύψος υα. γ) Να βρείτε το εμβαδόν του εγγεγραμμένου κύ­κλου στο τρίγωνο ΑΒΓ.

Λύση

α) Α

Β

Γ

Έχουμε 1 15 1 1 (ΑΒΓ) =ΊΑΒΑΓημΑ� 4" ="25 · 3 ·ψΑ� ημΑ = 2 .

Άρα Α = 1 20° , αφού Α > 90° . , 1 2(ΑΒΓ) β) Εχουμε (ΑΒΓ) = - α · υα � υα = ( 1 ) .

2 α Αλλά α2 = β2 + γ2 - 2βγσυνΑ �

α2 = 9 + 25 - 2 · 3 · 5 ( -�) � α2 = 49 � α = 7 (2).

2· 15 Από τις ( 1 ), (2) προκύπτει ότι υα =____1_ �υα = 15 .

7 14 γ) Έστω Ε το εμβαδόν και ρ η ακτίνα του εγγε­γραμμένου κύκλου τότε Ε = πρ2 (3) . Αλλά (ΑΒΓ) = τ · ρ �

(ΑΒΓ) (ΑΒΓ) 2(ΑΒΓ) ρ --- - -- τ - α + β + γ - α + β + γ 2

2 -� 1 5 οπότε: ρ = 4 = ..1... = .!..

(4) . 5 + 3 + 7 15 2

Έτσι από την (3) λόγω της (4) είναι: Ε = π - (�)2 = : .

Άσκηση 4η

Α ν η διάμεσος ΑΜ τριγώνου ΑΒΓ τέμνει τον περιγεγραμμένο κύκλο στο Δ και ισχύει ΑΒ · ΑΓ = 3ΒΔ · ΔΓ (1). Τότε να αποδείξετε ότι: α) (ΑΒΓ) = 3(ΒΔΓ) β) ΑΜ = 3ΜΔ γ) β2+f = 2α2.

Λύση

α)

Είναι BAf + Mr = 1 80° (γιατί;), οπότε (ΑΒΓ) ΑΒ · ΑΓ 0) 3ΒΔ · ΔΓ , -- = = 3 οποτε (ΒΔΓ) ΒΔ · ΔΓ ΒΔ · ΔΓ (ΑΒΓ) = 3(ΒΔΓ) . β) Τα τρίγωνα ΑΒΓ και ΒΔΓ έχουν κοινή βάση ΒΓ

ΕΥΚΛΕΙΔΗΣ Β' λθ ' τ4/35

Page 38: Ευκλειδης Β 60

Μαθηματικά Β ' Λυκείου

οπότε (ΑΒΓ) = ΑΚ (2) (ΒΔΓ) ΔΗ

Αλλά ΑΚΜ�ΜΜΙ(γιατί;) , οπότε ΑΚ = ΑΜ (3). ΔΗ ΜΔ

Από τις (2), (3) έχουμε: (ΑΒΓ) = ΑΜ 6 ΑΜ = 3 � ΑΜ = 3ΜΔ . (ΒΔΓ) ΜΔ ΜΔ γ) Ισχύει ΑΜ · ΜΔ = ΒΜ · ΜΓ (τεμνόμενες χορ­

(β ) δές) �

ΑΜ α α ΑΜ2 α2 2 3α2 ΑΜ ·-=- ·-� -- =-� ΑΜ =-� 3 2 2 3 4 4

2β2 + 2γ2 α2 3α2 --=----=----=-�2β2 +2γ2 =4α2 �β2 +γ2 =2α2 4 4

Άσκηση Sη Α ν σε τρίγωνο ΑΒΓ είναι β · γ = 8Rρ να βρείτε

το λόγο β + γ . α Λύση Έστω (ΑΒΓ) = Ε .

'Εχουμε Ε = αβγ � R = αβγ ( 1 ) . 4R 4Ε

Επίσης Ε = τ · ρ � ρ = Ε (2). τ

OJ αβγ τ/ 4αβγ Έτσι βγ = 8Rρ�βγ = 8 ·- ·-ρ � βγ =--� (2) 4$ τ 2τ

2τ = 4α� α + β + γ = 4α � β + γ = 3α � β + γ = 3 α

Άσκηση 6η

Επειδή ΒΓ = ΓΔ = α, ο κύκλος (Γ,α) διέρχεται από τα σημεία Β και Δ. Φέρνουμε ΔΕ .l ΒΓ . Τότε ΓΕ = � ( 1 ) . Στο ορθογώνιο τρίγωνο ΔΓΕ είναι

2

ΓΕ = Γ Δ , οπότε Δ = 30° και f = 60° (2). Από το 2

πυθαγόρειο θεώρημα έχουμε 2 3α2 ΔΕ2 = ΓΔ2 - ΓΕ2 = α2 -� =-

οπότε ΔΕ = αJ3

(3). 2

4 4

Έστω Ε το εμβαδόν του μικτόγραμμου τριγώνου ΑΒΔ τότε: Ε = (ΑΒΓ) - (Γ ·Μ) = = ΑΔ + ΒΓ · ΔΕ - π · α

2 · 60° = 2 360°

α - + α αJ3 πα2 = -2-·---= 2 2 6

= 3J3α2 - πα

2 = α

2 (9J3 - 4π) . 8 6 24

Άσκηση 7η Δίνεται κύκλος (O,R) και η χορδή του ΑΒ = λι. Αν Μ σημείο του κυρτογώνιου τόξου ΑΒ ώστε ΜΑ = J2 και ΜΒ = 2 να βρείτε: α) την ακτίνα R του κύκλου. β) Το γραμμοσκιασμένο Εμβα­δόν.

Λύση

Δίνεται τραπέζιο ΑΒΓ Δ με Α = Β = 90° , ΒΓ = α)

ΓΔ = α και ΑΔ = α . Γράφουμε τον κύκλο (Γ,α). 2 Να βρείτε το εμβαδόν του μικτόγραμμου τριγώ­νου ΑΒΔ.

Γ

c. 60° α 2

'>"'-�---f! Ε α

α 2

Μ

Είναι:

Είναι ΑΒ = λ4 = R J2 άρα ΑΜΒ = 3600 = 90° .

4 - � 270° Επίσης: ΑΓΒ = 270° , οπότε Μ = --= 135° ( 1 ) 2

ΕΥΚΛΕΙΔΗΣ Β ' λθ' τ4/36

Page 39: Ευκλειδης Β 60

Μαθηματικά Β ' Λυκείου -------------� (I )

Αλλά ΑΒ2 = ΑΜ2 + ΜΒ2 - 2ΑΜ · ΜΒσυνΜ <::::>

(RJZ)' =Fl' + 22 - Fz z { -1)�

2R 2 = 2 + 4 + 4 <::::> 2R 2 = 10 <::::> R2 = 5 <::> R = J5 . β) Έστω Ε το γραμμοσκιασμένο εμβαδόν. Είναι: Ε = (οο ) - (οΑΒ) - (ΜΑΒ) <::::>

πR290° 1 1 � Ε = --RR --ΜΑΜΒημΜ <::::>

360° 2 2 πR 2 R 2 1 ο (α)

Ε =- ----ΜΑ · ΜΒημ1 35 <::::> 4 2 2

Ε = 5π -� -_!_Γ . j Γ2 <::> Ε = 5π - 14 . 4 2 j 2 2 4

Άσκηση 8η Δίνεται κύκλος (O,R) και η χορδή του ΑΒ = λ.;. Αν Μ σημείο της χορδής ΑΒ ώστε ΜΒ = 3ΑΜ και η δύναμη του σημείου Μ ως προς τον κύκλο (O,R) είναι Δ�ο.R ) = -12 , να βρείτε το εμβαδόν ε του κυκλικού τμήματος χορδής ΑΒ. Λύση

. --- 360° ο Εχουμε ΑΒ = λ6 = R <::::> ΑΒ = -6- = 60 ( 1 )

. ΜΒ ΑΒ Επισης ΜΒ = 3ΑΜ <::::> ΑΜ =-<::::> ΑΜ =-3 4

R <::::> ΑΜ =- (2) . 4

Φέρνουμε το απόστημα ΟΗ.

Είναι ΟΗ = α6 = R../3

(3) και ΑΗ = ΑΒ = R , 2 2 2

<2) R R R οπότε ΜΗ = ---<=> ΜΗ =- (4) . 2 4 4

1 3R2 Είναι: ΟΜ2 = ΟΗ2 + ΜΗ2 = . . . = -- (5) 16 • Μ 2 2 (S) Ομως: Δ(Ο,R ) = -12 <::> 0Μ - R = -12<::::>

1 3R2 2 ---R = -12 <::> · . . <::> R = 8 (6) 16

Έτσι

ε = (ο,ΑΒ ) - (οΑΒ) = πR2 · 60ο- R2J3

= 360° 4

= R' (: -�)� ε = 64( �-�} Άσκηση 9η

Δίνεται ορθογώνιο τρίγωνο ΑΒΓ (Α = 90° ) εγ-γεγραμμένο σε κύκλο (O,R). Αν ΑΔ ύψος του τριγώνου και (ΑΒΓ) = 4(ΑΔΟ) να υπολογίσετε την περίμετρο του τριγώνου ΑΒΓ ως συνάρτη­ση του R. Λύση

� � ο (ΑΒΓ) ΑΒ · ΑΓ Επειδή Α = Δ = 90 είναι = ---(ΑΔΟ) ΑΔ · ΔΟ

<=> 4(ΑΔΟ) = ΑΒ

. ΑΓ <=> ΑΒ. ΑΓ = 4ΑΔ . ΔΟ ( 1 )

(ΑΔΟ) ΑΔ · ΔΟ Όμως ΑΒ · ΑΓ = ΑΔ · ΒΓ (2) (γιατί;) . Από τις (1 ),(2) προκύπτει ότι ΑΔ . ΒΓ = 4 . ΑΔ . ΔΟ <::::>

ΒΓ = 4ΔΟ <::::> 2ΒΟ = 4ΔΟ <::::> ΔΟ = ΒΟ . Άρα ΑΔ 2

ύψος και διάμεσος στο τρίγωνο ΑΒΟ, οπότε

ΑΒ = ΑΟ = ΒΓ = ΒΟ = R δηλαδή το τρίγωνο 2

ΟΒΑ είναι ισόπλευρο άρα η Β = 60° . � ο ι;:; Έτσι ΑΓ = 1 20 <::::> ΑΓ = λ3 = Rν3 (3) και � ο • --- ο Γ = 30 , οποτε ΑΒ = 60 <::::> ΑΒ = λ6 = R (4) . Επομένως η περίμετρος Π του τριγώνου ΑΒΓ εί­ναι:

(3) Π = ΑΒ + ΑΓ + ΒΓ <::::> Π = R + R J3 + 2R <::::>

(4)

ΕΥΚΛΕΙΔΗΣ Β ' λθ ' τ4/37

Page 40: Ευκλειδης Β 60

Μαθηματικά Β ' Λυκείου

Άσκηση lΟη

Σε κύκλο (O,R) να φέρετε διάμετρο ΑΒ και χορδή ΑΓ = R. Να προεκτείνεται τη ΒΓ κατά τμήμα Γ Δ = ΓΒ και να φέρετε τη ΔΑ που τέμνει τον κύκλο στο σημείο Ζ. α) Να αποδείξετε ότι ΒΓ = RJ3 . β) Να υπολογίσετε τη χορδή ΑΖ. γ) Να υπολογίσετε το εμβαδόν του μεικτόγραμ­μου τριγώνου ΑΔΓ. Λύση

α) z

Η γωνία ArB = 90° (γιατί;) Στο ορθογώνιο λοιπόν τρίγωνο ΓΑΒ από Πυθαγό­ρειο θεώρημα έχουμε ΒΓ2 = ΑΒ2 - ΑΓ2 = (2R)2 - R2 = 3R2 οπότε ΒΓ = R J3 . β) Οι χορδές ΑΖ και ΒΓ τέμνονται στο σημείο Δ, οπότε έχουμε ΔΑ · ΔΖ = ΔΓ · ΔΒ ( Ι ) . Είναι ΔΑ = ΑΒ = 2R (αφού το Α ανήκει στη με­σοκάθετο του ΒΔ) και ΔΒ = R J3 + R J3 = 2R J3 . Από την ( 1 ) έχουμε )I( · ΔZ = RJ3 . )I( J3 � ΔΖ = 3R , οπότε ΑΖ = ΔΖ - ΑΔ � ΑΖ = 3R - 2R � ΑΖ = R . γ) Φέρουμε την ακτίνα ΟΓ και παρατηρούμε ότι το τρίγωνο ΟΑΓ είναι ισόπλευρο με πλευρά R.

Ε' (ο ΑΓ) = πR2 · 6Οο = πR2 ιναι , 360ο 6

.

Στο τρίγωνο ΑΒΔ έχουμε Ο μέσο ΑΒ και Γ μέσο της ΒΔ � ΟΓ /I ΑΔ δηλαδή το τετράπλευρο ΟΑΔΓ είναι τραπέζιο με βάσεις ΑΔ = 2R, ΟΓ = R και ύψος

ΟΜ = α6 = RJ3 αφού ΑΖ = R = �. 2

Έτσι RJ3

(ΟΑΔΓ) = (ΑΔ + ΟΓ) . ΟΜ =

(2R + R)-2- =

2 2 3R2J3 =

4 οπότε το ζητούμενο εμβαδό είναι: � Ε = (ΟΑΔΓ) - (Ο,ΑΓ) = 3R2J3 πR 2 (9J3- 2π)R 2 = =

4 6

β' τρόπος

1 2

( � ) πR260° πR2 Είναι Ο,ΑΓ = 0 =-- , οπότε το εμβα-360 6

δόν του κυκλικού τμήματος που περικλείεται από � τη χορδή ΑΓ και το τόξο ΑΓ είναι:

(ε) = (ο,ΑΓ) - (ΟΑΓ) = π:2 - R�J3

Οπότε το ζητούμενο εμβαδό είναι: 1 ( πR2 R2J3 J Ε = (ΑΔΓ) - (ε) =lΔΓ · ΑΓ -

6 - -4-=

(9J3 - 2π )R2 = . . . = ....:....__----'--

1 2

I<A ΤΕΥΘΥΝΕ.Η

Θέμα 1 Έστω α,β δύο διανύσματα για τα οποία ισχύει:

ΙαΙ = l β l = 1 και ( i,β) = ; α) Να βρείτε διάνυσμα w , παράλληλο στο

α + β ' ώστε το διάνυσμα w - β ' να είναι κάθετο στο α .

Νίκος Θ. Αντωνόπουλος β) Αν ίί = α + χβ , xεJR και v = 2α + β , τότε:

i) Να βρείτε για ποιες τιμές του χ ισχύει ίi .L v .

ii) Να βρείτε ως συνάρτηση του χ το εμβα­δόν Ε του τετραγώνου που σχηματίζεται με μήκος πλευράς ίσο με lίi + vl και να

ΕΥΚΛΕΙΔΗΣ Β' λθ ' τ4/38

Page 41: Ευκλειδης Β 60

Μαθηματικά Β ' Λυκείου

αποδείξετε ότι για κάθε πραγματικό α-ριθμό χ ισχύει Ε ;:::: 27 . 4

Λύση

α) Ισχύουν: w ιι (α + β) � w = λ(α + β) � w = λα + λβ ,

λε.ΙR. ( 1 ) και:

( -) ( -) - ( I )

α _1_ w - β � α w - β = Ο� αw - αβ = Ο�

� λα2 + λαβ - αβ = Ο (2)

Επίσης α2 = ΙαΙ2 = 1 και α · β = ΙαΙ - Ιβ l συν; = -Ξ , οπότε (2) � λ +_!._λ -_!._ = 0� λ = _!_

2 2 3

Ά - 1 (- -β ) ρα, w = "j α + .

β) i) Είναι: υ _1_ ν � υ · v=O� (α + χβ) (2α + β) = ο�

� 2α2 + αβ + 2χαβ + χβ = Ο 1 1 5 � 2 · 1 +- + 2 . -χ + χ = ο� 2χ + - = ο 2 2 2 5 � χ = --4

ii) Το εμβαδόν του τετραγώνου με μήκος πλευράς είναι:

E = lu + vl2 = (u + νγ = [3α + (χ + ι) βΤ

= α2 + 6(χ + l) αβ + (χ + 1)2 β2 =

= 9 + 3 (x + l) + (x + l)2 =

2 3 9 27 = (x + l) + 2 · 2(χ + 1) + 4 +4

[ 3 ]2 27 27 = (x + l) + - +-;:::: -2 4 4

Άρα, η ελάχιστη τιμή του εμβαδού του τετρα-. . 27 . . 5 γωνου ειναι 4 και προκυπτει οταν χ = -2

Θέμα 2 'Εστω α, β δύο διανύσματα με lii l = 4 , I P I = 6 και

- - π (α,β) =-3

) Α _ ι _ -β _ _ 2 -β . β . α ν u = l α + και v = α -"'3 , τοτε να ρ ει-

τε: i) Το εσωτερικό γινόμενο ϋ · v ii) Το συνημίτονο της γωνίας των διανυ­

σμάτωνϋ, v . β) Θεωρούμε τρίγωνο ΑΒΓ με ΑΒ = ii και - -ΑΓ = β .

i) Αν Μ είναι το μέσο της ΒΓ, να βρείτε το μέτρο του διανύσματος ΑΜ

ii) Α ν Δ είναι η προβολή του Β στην ΑΜ, να εκφράσετε το διάνυσμα ΒΔ ως συ­νάρτηση των διανυσμάτων α,β και να

- 14 -αποδείξετε ότι: ΑΔ =-ΑΜ . 19 Λύση

α) i) Ισχύει:

α - β = ΙαΙ - Iβlσυν ; = 4 · 6 · -i = 1 2 . οπότε:

- - ( 1 - β-J (- 2 β-J u·ν= 2α + α -

3 =

1 _2 ι _- -- 2 -2 =-α --αβ + αβ --β = 2 3 3

= _!._ · 16 - _!_ · 12 + 12 _3. · 36 = 2 3 3

= 8 - 4 + 12 - 24 = -8

ii) Είναι: συν ( �) = �:ι : ��� ( 1 ) και: lu l2 = I-Ξ α + β 12 = �α2 +αβ+β2 = _!._ · 16 + 1 2 + 36 = 52 , οπότε 4

lu l = .J52 = 2Jl3

1_ 1 2 _ 1_ 2 β- ι2 _ -2 4 _β_ 4 β-2 _ ν -

α -- - α --α +- -3 3 9

= 16 - 4 · 4 + 4 · 4 = 16 ' οπότε lvl = .Ji6 = 4

Το συνημίτονο της γωνίας των διανυσμά­των, όπως προκύπτει από τον τύπο ( 1 ) είναι:

(.:::-::;

) -8 -Jl3 συν

u, ν = 2 . 4Jl3

= """"""13

ΕΥΚΛΕΙΔΗΣ Β ' λθ ' τ4/39

Page 42: Ευκλειδης Β 60

Μαθηματικά Β ' Λυκεiου ------------

β)

Β�Γ Μ

i) Είναι: ΑΜ = �( a + β) , οπότε:

IAMI2 = ±( a2 + β2 + 2aβ) = .!.. ( 16 + 36 + 24} = 19 . 4 Άρα IAMI = .Jϊ9 . Ισχύει: ΑΔ // ΑΜ , οπότε υπάρχει λεffi. , ώ­

στε να ισχύει ΑΔ = λ · ΑΜ οπότε: ΒΔ = ΑΔ - ΑΒ = λΑΜ -ΑΒ = λ ( _ β-) _ ( λ ι) - λ -β 2 α + - α = 2 - α +2 και ΒΔ 1. ΑΜ <::::> ΒΔ · ΑΜ = Ο<::::> <=> �[(λ - 2} a + λβ] -�(a + β) = ο <=> (λ - 2) a2 + (λ - 2)a · β + λaβ + λβ2 = Ο <::::> 16(λ - 2) + 12(λ - 2) + 12λ + 36λ = ο <::::> 1 6λ + 1 2λ + 1 2λ + 36λ = 32 + 24 <::::> 76λ = 56 <::::> λ = 14 . 19

- 14 -Επομένως, ΑΔ =- · ΑΜ 1 9

Θέμα 3

Δίνονται τα σημεία Α (-2 λ - 2)

' λ + 1

Β ( �1 , -1) , λ > 1 . και

α) Να αποδείξετε ότι οι ευθείες που διέρχονται από τα σημεία Α, Β Ύια τις διάφορες τιμές του λ ανήκουν στην οικοοyένεια των ευθειών ( ελ ) : λχ + (λ + 1)y + λ + ε = Ο .

β) Να αποδείξετε ότι όλες οι ευθείες (ε λ ) διέρχονται από σταθερό σημείο.

γ) Να βρείτε την ευθεία (ε) της οικοΎένειας ( ελ ) ώστε το εμβαδόν του τριΎώνου ΟΑΒ να είναι ίσο με 1.

δ) Να αποδείξετε ότι η ευθεία (ε) εφάπτεται στον κύκλο χ2 + (y - 3)2 = 13 .

ε) Να βρείτε σημείο Λ του άξονα χ'χ ώστε το συμμετρικό του Κ ως προς την ευθεία (ε) να βρίσκεται στον άξονα y 'y .

Λύση

-1 - λ - 2 -λ - 1 - λ + 2 α) Είναι: λΑΒ = 1 λ + 1 =-_--'�:"-:'-:'�'-:-λ--- + 2 λ λ

λ(Ι - 2λ) λ = ---:---'---:--:-___:.__----:--(λ + Ι) (Ι - 2λ) λ + Ι οπότε οι ευθείες που διέρχονται από τα σημεία Α, Β για τις διάφορες τιμές του λ είναι οι: Υ + 1 = � (χ + .!..) <::::> (λ + Ι} ( y + Ι) = λ + Ι λ

= -λχ - Ι <::::> <::::> λχ + (λ + Ι} y + λ + 2 = 0

β) Με λ = 2 έχουμε: ( ε2 ) : 2χ + 3y + 4 = Ο . Με λ = 4 έχουμε (ε4 ) : 4χ + 5y + 6 = 0 . Εύκολα βρίσκουμε ότι οι ( ε2 ) , ( ε4 ) τέμνονται στο σημείο Μ (Ι, -2) . Θα αποδείξουμε ότι όλες οι ευθείες της οικογένειας διέρχονται από το σημείο Μ(Ι, -2) . Αρκεί: λ · Ι + (λ + Ι } (-2} + λ + 2 = 0<::::> 0λ + 0 = 0 , που ισχύει. Άρα όλες οι ευθείες (ε λ ) διέρχονται από το Μ.

γ) Είναι ΟΑ = ( -2, �: � }οΒ = ( �� , - 1) . οπότε

-2 λ - 2 (OAB} = .!_ I λ + Ι 1 =.!_ 2 + λ - 2 = 2 _.!_ - 1 2 λ(λ + l)

λ Ι 2λ2 + 2λ + λ - 2 = = 2 λ( λ + Ι}

Ι 1 2λ2 + 3λ - 21 12(λ2 - 1) + 3λι = 2 λ( λ + 1) = 2λ(λ + 1) = 2λ2 + 3λ - 2

= --,.-----.,.-2λ(λ + Ι} '

ΕΥΚΛΕΙΔΗΣ Β ' λθ ' τ4/40

Page 43: Ευκλειδης Β 60

Μαθηματικά Β ' Λυκείου

διότι 2(λ2 - 1) + 3λ > Ο , οπότε ( ΟΑΒ) = 1 <:::> 2"Κ + 3λ - 2 = 2λ2 + 2λ <:::> λ = 2 Άρα η ζητούμενη ευθεία είναι η ( ε2 ) : 2χ + 3y + 4 = 0

δ) Αρκεί να αποδείξουμε ότι: d ( Κ,ε2 ) = ρ , όπου Κ ( 0,3) είναι το κέντρο και ρ = Jϊ3 είναι η ακτίνα του κύκλου.

ε)

12 · 0 + 3 · 3 + 4 1 1 3 r;;; Είναι d (κ,ε ) = =-= ν13 , 2 ../4 + 9 Jϊ3 οπότε η ευθεία 2χ + 3y + 4 = Ο εφάπτεται στον κύκλο χ2 + (y - 3γ = 1 3 .

Υ

ε) χ

κ

Αν Λ (χ , Ο) το σημείο του χ 'χ και κ (Ο, y ) το συμμετρικό του ως προς την ευθεία ( ε2 ) , τότε

3 y 3 λΚΛ . λ ε = -1 <:::? λΚΛ = - <:::? - = - <:::? 2 2 -χ 2

3 <:::> y = -- X ( 1 ) . 2 Επίσης το σημείο Μ (; , Ξ) περιέχεται στην ( ε2 ) , οπότε 2� + 3L + 4 = 0 <:::> χ + 3Υ = -4 <:::> 2 3 2

9 16 24 <:::> χ --χ = -4 <:::> χ =- , οπότε y = -- . 4 5 5 Άρα το ζητούμενο σημείο είναι το Λ ( 156 , Ο) και το συμμετρικό του, το Μ ( Ο, -�4 ) .

Θέμα 4

Δίνεται η εξίσωση χ2 + y2 + 2( 2χ + 2y + xy) = 5

α) Να αποδείξετε ότι παριστάνει δύο ευθείες παράλληλες μεταξύ τους, έστω (ει ) , ( ε2 ) .

β) Να βρείτε την απόσταση των ευθειών.

γ) Να βρείτε μεσοπαράλληλη (ε) των ευθειών (ει ) , ( ε2 ) .

δ) Α ν το σημείο Ρ {κ, λ) κινείται στην ευθεία (ει ) και το σημείο Σ (μ, ν) κινείται στην ( ε2 ) , να αποδείξετε ότι το σημείο ( κ + μ λ + ν ) , Μ -2- + 1,-2- - 1 κινειται στην ευ-

θεία (ει ) και το εμβαδόν του τριγώνου πα­ραμένει σταθερό.

Λύση

α) Είναι: χ2 + y2 + 2 (2χ + 2y + xy) = 5 <:::> χ2 + y2 + 2xy + 4x + 4y - 5 = 0 <:::> <:::> (x + y)2 + 4(x + y) - 5 = 0 ( 1 ) .

Άν θέσουμε χ + y = ω , τότε η εξίσωση ( 1 ) γί-νεται ω2 + 4ω - 5 = 0 <:::> ω = 1 ή ω = -5 . Επομένως: ( l ) <:::> x + y = 1 ή x + y = -5 <:::> x + y - 1 = 0 ή χ + y + 5 = Ο , πράγμα που προφανώς σημαίνει ότι η ( 1 ) παριστάνει τις ευθείες (ει ) : x + y - 1 = 0 και (ε2 ) : x + y + 5 = 0 .

β) Ισχύει: d {ε ι , ε2 ) = d { Α,ε2 ) , όπου Α ένα τυχαίο σημείο της (ει ) . Το σημείο Α (Ο, 1) περιέχεται στην { ε1 ) , οπότε

10 + 1 + 5 1 6 � ct (ει , ε2 ) = ct (Α,ε2 ) = ....Γι+J = � = 3ν2 . 1 + 1 ν2

γ) Θεωρούμε το σημείο Β (Ο, -5 ) το οποίο περιέ­χεται στην ευθεία ( ε2 ) • Το μέσο του τμήματος ΑΒ είναι το Κ (Ο, -2) και περιέχεται στην με­σοπαράλληλη, η οποία έχει συντελεστή διεύ­θυνσης λ = -1 . Άρα η εξίσωση της μεσοπα­ράλληλης (ε) είναι:

y + 2 = -1 (χ - 0) <:::> χ + y + 2 = 0 .

δ)

ΕΥΚΛΕΙΔΗΣ Β ' λθ ' τ4/41

Page 44: Ευκλειδης Β 60

Μαθηματικά Β ' Λυκείου

Ισχύουν: κ + λ - 1 = Ο (2) και μ + ν + 5 = Ο (3) .

Αρκεί να αποδείξουμε ότι: 1 + κ + μ - 1 + λ + ν + 2 = 0 <:::> 2 2 <:::> κ + μ + λ +ν + 4 = 0 <:::>

<:::> (κ + λ - 1) + (μ + ν + 5) = 0 , που ισχύει εξαιτίας των (2), (3) . Άρα το Μ κινείται στην ευθεία (ε) . Είναι: ΣΜ = ( κ + μ + 1 - κ λ + ν - 1 - λ) = 2 ' 2 = ( μ + κ + 2 ν + λ - 2) και 2 ' 2 - ( κ + μ λ + ν ) ΡΜ = -2-+ 1 - μ, -2-- 1 - ν = ( κ - μ + 2 λ - ν - 2 ) . . = , , οποτε: 2 2

(ΡΜΣ) = _!_ l ιμ - κ + 2 ν - λ - 2 1 1 = 8 κ - μ + 2 λ - ν - 2

= _!_ I( μ - κ + 2)(λ - ν - 2) -1 8

-(κ - μ + 2)(ν - λ - 2)1 = _!_l(μ - κ ) (λ - ν - 2) + 2 (λ - ν - 2) +I

8

+ (μ - κ)( ν - λ - 2) - 2( ν - λ - 2)1 =i l (μ - κ)(λ - ν - 2 + ω - λ - 2) +1

+2(λ - ν - 2 - ν + λ + 2)

β) Να βρείτε την εξίσωση της ευθείας πάνω στην οποία βρίσκονται τα κέντρα των κύ­κλων (Cλ ) .

γ) Να εξετάσετε αν υπάρχει κύκλος (Cλ ) ο οποίος εφάπτεται στους άξονες χ'χ και y 'y .

δ) Να αποδείξετε ότι οι κύκλοι ( Cλ ) έχουν κοινή εφαπτομένη, την οποία να βρείτε.

Λύση

α) Η εξίσωση είναι της χ 2 + y2 + Αχ + By + Γ = Ο με: Β = 2(λ + 2) και Γ = 4λ + 4 . Είναι:

μορφής Α = 4λ

Α2 + Β2 - 4Γ = 16� + 4(λ + 2γ - 4(4λ + 4) = = 16� + 4λ2 + 16λ + 16 - 16λ - 16 = 20λ2 > ο ' για κάθε λ * Ο . Άρα για κάθε λ ::ι:. Ο η εξίσωση παριστάνει κύκλο με κέντρο Κ( -2λ, λ + 2) και ακτίνα ρ = νΓs 11 λ! . Α ν λ = Ο τότε η εξίσω­ση γίνεται χ 2 + (y - 2)2 = Ο και παριστάνει το σημείο (0,2).

β) Αν (χκ , Υκ ) είναι οι συντεταγμένες του κύ-κλου για τις διάφορες τιμές του λ, τότε έχουμε χ κ = -2λ και y κ = λ + 2 , οπότε Χ κ = -2(Υκ - 2) <=? Χκ + 2yκ - 4 = 0 . Άρα τα κέντρα των κύκλων βρίσκονται πάνω στην ευθεία χ + 2y - 4 = Ο .

= _!_ I( μ - κ) ( -4) + 4(λ - ν )I = _!_ I κ + λ - (μ + ν )I = γ) 8 2

Για να εφάπτεται ο κύκλος ( Cλ ) στον άξονα: • y'y αρκεί να ισχύει

= .!.11 - ( -5)1 = 3 , βάσει των (2), (3) . 2 Άρα το εμβαδόν του τριγώνου ΡΜΣ είναι στα­θερό και ίσο με 3 .

Θέμα 5 Δίνεται η εξίσωση

( cλ ) : χ2 + y2 + 4λχ - 2(λ + 2)y + 4λ + 4 = ο α) Να αποδείξετε ότ για κάθε λ * Ο παριστάνει

κύκλο. Τι συμβαίνει αν λ = Ο ;

l x κ l = ρ <:::> 2 lλl = νΓs"iλl , που αποκλείεται όταν λ * Ο .

• χ 'χ , αρκεί να ισχύει I Y κ i = ρ <=? iλ + 21 = J5"iλl <=?

(λ + 2)2 = 5λ2 <:::> 4λ2 - 4λ - 4 = 0 <=> λ� (1 -νΓs) ή <=> λ� (1 +νΓs) .

Άρα δεν υπάρχει κύκλος της οικογένειας ( Cλ ) ο οποίος να εφάπτεται στον άξονα y'y , ενώ υπάρχουν δύο κύκλοι της ίδιας οικογένειας οι οποίοι εφάπτονται στον άξονα χ 'χ .

ΕΥΚΛΕΙΔΗΣ Β ' λθ ' τ4/42

Page 45: Ευκλειδης Β 60

Μαθηματικά Β ' Λυκείου

δ) Η ευθεία (ε) : y = αχ + β εφάπτεται στους κύ­κλους ( Cλ ) αν και μόνο αν ισχύει:

( ) ι-2αλ - (λ + 2) + β l � ι ι d κ, ε = ρ <::::> Γ?---: = ν5 λ ,

να2 + 1 λ :;t: Ο <=> l2αλ + λ + 2 - βl = -J5Ιλi .Jα2 + 1 <=> <=> i(2α + 1)λ + 2 - β l = -J5Ιλi .Jα2 + 1 <::::> ( 2α + 1) z λz + +2 (2α + 1) (2 - β)λ + (β - 2)2 = 5λ2 (α2 + 1) <::::> [ ( 2α + 1 )2 - 5 (α 2 + 1) J λ2 +

+2 (2α + 1) ( 2 - β)λ + (β - 2)2 = 0 που ισχύει για κάθε λ :;t: Ο , αν και μόνο αν, εί­ναι: {4αz + 4α + 1 - 5α2 - 5 = 0 {α - 2 2 (2α + 1 ) (2 - β ) = Ο <::::> - . β = 2

(β - 2γ = 0 Άρα όλοι οι κύκλοι έχουν κοινή εφαπτομένη την ευθεία y = 2χ + 2 .

Θέμα 6

Δίνονται τα σημεία A (l,S) , B (S,-2) και Γ ( -3,-3)

α) Α ν Μ ( x,y) είναι τυχαίο σημείο του επιπέ­δου, να αποδείξετε ότι:

i) -2 -2 -2 ΜΑ + ΜΒ + ΜΓ � 70 -2 -2 -2 ii) Αν ισχύει ΜΑ + ΜΒ + ΜΓ = 85 , τότε

το σημείο Μ κινείται σε κύκλο (c) του ο­ποίου να βρείτε το κέντρο και την ακτίνα.

β) Να αποδείξετε ότι το σημείο Ρ(3,1) περιέχε­ται στον κύκλο αυτό.

γ) Να υπολογίσετε τη γωνία που σχηματίζουν οι εφαπτόμενες του κύκλου που άγονται από το σημείο Τ(0,3).

Λύση

α) i) Είναι: -2 -2 -2 2 2 ΜΑ + ΜΒ + ΜΓ = ( χ - 1) + ( y - 5) + + (χ - 5)2 + (y + 2)2 + ( χ + 3)2 + ( y + 3)2

β)

= χ2 - 2x + 1 + y2 - 10y + 25 + x2 - 10χ + 25 + +y2 + 4y + 4 + x2 + 6x + 9 + y2 + 6y + 9 =

= 3χ2 - 6x + 3y2 + 73 = 3 (χ2 - 2x + 1) + 3y2 + 70

= 3(χ - 1)2 + 3y2 + 70 � 70 , που είναι το ζητούμενο. Η ισότητα ισχύει όταν M( l ,O). Ισχύει: -2 -2 -2 ΜΑ + ΜΒ + ΜΓ = 85 <::::> 3 ( χ - 1 γ + 3y2 + 70 = 85

<=> 3(χ - 1)2 + 3/ = 15 <=> (χ - 1γ + y2 = 5

απ' όπου προκύπτει ότι το σημείο Μ κινείται σε κύκλο (c) με κέντρο Κ( 1 ,0) και ακτίνα ρ = -15 .

Υ

-1--=+-�---ι-τ-χ Λ(χ,y)

κ Παρατηρούμε ότι (3 - 1)2 + 12 = 4 + 1 = 5 , δη­λαδή οι συντεταγμένες του Ρ επαληθεύουν την εξίσωση του κύκλου. Άρα το σημείο ρ περιέχεται στον κύκλο (c). Το σημείο Λ(χ,y) ανήκει στην εφαπτομένη του κύκλου (c), αν και μόνο αν, ισχύει: ΚΡ ..L ΡΛ , ( 1 ) . Είναι ΚΡ (2, 1) και ΡΛ = ( χ - 3, y - 1) , οπότε: ( 1 )

<::::> ΚΡ · ΡΛ =0 <::::> 2 ( χ - 3) + (y - 1) = 0 <::::>

<::::> 2χ - 6 + y - 1 = Ο <::::> 2χ + y - 7 = Ο , που είναι η εξίσωση της εφαπτομένης του κύ­κλου στο σημείο Ρ.

γ) Από το σημείο Τ διέρχονται: • Η κατακόρυφη ευθεία χ = Ο (άξονας y'y )

που δεν εφάπτεται στον κύκλο. • Οι μη κατακόρυφες ευθείες της μορφής:

y - 3 = λχ , δηλ. οι ευθείες (�) :λχ-y+3=0 . Μία ευθεία της οικογένειας ( ελ ) εφάπτεται στον κύκλο αν και μόνο αν ισχύει:

d ( κ, ελ ) = -J5 <::::> lλ + 31 = -J5 .Jλ2 + 1

ΕΥΚΛΕΙΔΗΣ Β ' λθ ' τ4/43

Page 46: Ευκλειδης Β 60

Μαθηματικά Β' Λυκείου

� (λ + 3)2 = 5 (λ2 + 1) � � λ2 + 6λ + 9 - 5λ2 - 5 = ο � � -4λ2 + 6λ + 4 = 0 � � 2λ2 - 3λ - 2 = ο .

Οι ρίζες λι , λ2 της εξίσωσης είναι οι συντελεστές διεύθυνσης των εφαπτομένων του κύκλου που άγονται από το σημείο Τ και

-2 εξαιτίας της ισότητας λι · λ2 = - = -1 2 προκύπτει ότι οι εφαπτόμενες είναι κάθετες μεταξύ τους.

Θέμα 7 χ2 y2

Δίνεται η έλλειψη C1 : - + - = 1 . 25 9 α) Αν μία ευθεία διέρχεται από την εστία Ε 'και

τέμνει την έλλειψη στα σημεία Β, Γ, να υπο­λογίσετε την περίμετρο του τριγώνου ΕΒΓ.

β) Να βρείτε την εξίσωση της υπερβολής ( C2 ) η οποία έχει τις ίδιες εστίες με την έλλειψη και η εκκεντρότητα της είναι ίση με 2.

γ) Θεωρούμε την εφαπτομενη της υπερβολής ( c2 ) σε τυχαίο σημείο της Μ (Χι ' Υ 1 ) δια-

α)

φορετικό από τις κορυφές της και την κάθε­τη στην εφαπτομένη στο Μ. Α ν η εφαπτομέ­νη τέμνει τον άξονα χ 'χ στο σημείο Ρ και η κάθετη στο σημείο Κ, να αποδείξετε ότι: - -ΟΚ · ΟΛ = 16 .

Λύση Υ

(ε)

Στην έλλειψη ( Cι ) έχουμε: α2 = 25 , β2 = 9 ,

γ2 = 25 - 9 = 1 6 οπότε γ = 4 και Ε(4,0), Ε ' (-4,0) . Η περίμετρος του τριγώνου ΒΓΕ είναι:

(ΒΓ) + (ΓΕ) + (ΒΕ) = (ΒΕ') + (ΒΕ) + (ΓΕ') + (ΓΕ) = 2α + 2α = 20 ,

όπως προκύπτει από τον ορισμό της έλλειψης.

χ 2 y2 β) Α ν -2 --2 = 1 είναι η εξίσωσης της υπερβο­a b

λης ( C2 ) , τότε: c = γ = 4 και � = 2 � a = 2 . a Τέλος c2 = a2 + b2 � 4 + b2 = 16 � b2 = 12 , οπότε η εξίσωση της υπερβολής είναι η : χ2 y2 - -- = 1 . 4 1 2

γ) Η εφαπτομένη της υπερβολής στο τυχαίο ση­μείο της Μ (χ ι , y ι ) έχει εξίσωση :

ε -- ΧΧ ι - ΥΥ ι --1 4 12 και έχει συντελεστή διεύ-

θυνσης λε = 3χ ι . Αν λη είναι ο συντελεστή Υ ι διεύθυνσης της κάθετης (η) στην υπερβολή

' Μ ' · λ λ - 1 λ - Υι στο σημειο , τοτε. η · ε - � η - -- . 3χ ι

Άρα η εξίσωση της κάθετης στην υπερβολή στο σημείο Μ (χ ι • Υ ι ) είναι η :

y - yι = -2L (χ - χ ι ) · Με y = O στην εξίσω-3χ ι

ση της ευθείας (ε) βρίσκουμε χ = _i_ , οπότε Χ ι

Ρ ( : , Ο) . Με y = Ο στην εξίσωση της ευθείας

(η) βρίσκουμε 3χ ι = χ - Χ ι � χ = 4χ ι , οπότε Κ (4χ ι , Ο) .

- - ( 4 ) Είναι: ΟΚ = (4χ ι , Ο) και ΟΛ= � ,0 , οπότε:

- - 4 ΟΚ·ΟΛ=4χι ·-= 16 , που είναι το ζητούμενο. Χι

Θέμα 8 Δίνεται η παραβολή y2 = 4χ και η ευθεία

ε : � - Ι. + l = Ο . 3 4 α) Να βρείτε το σημείο Μ ( x1 ,y0 ) , με Υ ο > Ο ,

της παραβολής, ώστε αν Α είναι η προβολή του στη διευθέτουσα, να ισχύει: {MAE) = i ·

β) Να αποδείξετε ότι: ΕΥΚΛΕΙΔΗΣ Β ' λθ ' τ4/44

Page 47: Ευκλειδης Β 60

α)

Μαθηματικά Β ' Λυκείου

ί) Η ευθεία δεν έχει κοινό σημείο με την παραβολή και να βρείτε την απόσταση d

τυχαίου σημείου της παραβολή από την ευθεία, ως συνάρτηση της τεταγμένης του σημείου.

ii) Η ελάχιστη τιμή της απόστασης d είναι 39 β ' ' . 20 και να ρειτε το σημειο της παρα-

βολής το πλησιέστερο στην ευθεία (ε). Λύση

Υ

Α ν Μ ( χ0 , y 0 ) τότε η προβολή του Μ στην ευ­θεία χ = -1 (διευθέτουσα) είναι το σημείο Α ( -1 , y0 ) . Εξάλλου E( l ,0), οπότε:

- [ y02 ) -ΕΜ = 4 - 1, y0 , EA = (-2, y0 ) και

Επομένως 5 3 5 (MAE) = - <=> � +12_ = - <=> y 3 + 4y - 5 = 0 8 8 2 8 ο ο

<=> y03 - 1 + 4( y0 - 1) = 0 <::::> <=> ( Υ ο - 1 ) ( Υ ο 2 + Υ ο + 1 + 4) = Ο <=> <=> ( y 0 - 1) ( y 0 2 + y 0 + 5) = Ο <=> y 0 = 1 , οπότε το

ζητούμενο σημείο είναι Μ ( �, 1 J .

παραβολή κοινό το σημείο Κ ( χ , , y , ) . Τότε ισχύουν: y1 2 = 4χ1 και 4χ 1 - 3y1 + 1 2 = 0 οπότε y 1 2 - 3y1 + 1 2 = Ο , που αποκλείεται διότι το τριώνυμο y2 - 3y + 1 2 = 0 έχει δια­κρίνουσα Δ = -39 < Ο . Άρα η ευθεία δεν έ­χει κοινό σημείο με την παραβολή. Η από-

σταση του τυχαίου σημείου Ρ [: , α) της

παραβολή από την ευθεία, είναι:

d � 14'f - 3α + 12 1 � !α' - 3α + 1 2 1 � ..)16 + 9 5

�[(α -Ξ)' + 3:] = .!.(α -�]2 + 39 ;?: 39

5 2 20 20

Ρ [� , α) (Η ισότητα με α = �) 2

Άρα το πλησιέστερο σημείο της παραβολής στην ευθεία (ε) είναι το (_2_·�J .

16 2

Θέμα 9 χ2 y2

Δίνεται η έλλειψη - + - = 1 και το σημείο 16 12

της Ρ (2,3) .

α) Να βρείτε την εφαπτομένη (ε) της έλλειψης στο Ρ και την κάθετη (η) στην εφαπτομένη στο σημείο αυτό.

β) Ν α βρείτε τις συντεταγμένες των προβολών Μ και Ν της εστίας Ε στις ευθείες (ε) και (η).

γ) Ν α αποδείξετε ότι: ί) Τα σημεία Ο, Μ, Ν είναι συνευθειακά. ii) {ΜΝΕ) = 3{0ΝΕ)

β) ί) Η εξίσωση της ευθείας γράφεται:

δ) Αν για τους αριθμού x1 , x2 , y1 ,y2 ισχύει: 3χ/ + 4y / = 3χ/ + 4y / = 48 , να αποδείξε­τε ότι: ( χ1 - χ2 )2 + (y1 - y2 )2 � 64 . 4x - 3y + 1 2 = 0 και έστω ότι έχει με την

ΕΥΚΛΕΙΔΗΣ Β ' λθ ' τ4/45

Page 48: Ευκλειδης Β 60

Μαθηματικά Β ' Λυκείου

Λύση

Α'

Είναι: α2 = 16 , β2 = 12 , γ2 = 4 , οπότε γ = 2 . α) Η εφαπτόμενη της έλλειψης στο σημείο Ρ εί­

ναι η ευθεία 2χ + 3Υ = 1 <=:> � + r = 1 <=:> χ + 2y = 8 και 1 6 12 8 4 λε = -� , οπότε λη = 2 . Η εξίσωση της κάθε-

της στην εφαπτομένη στο σημείο ρ είναι η : y - 3 = 2 ( χ - 2) <=:> y = 2χ - 1 .

β) Η εστία Ε της έλλειψης έχει συντεταγμένες (2,0). Η ευθεία ΕΜ έχει εξίσωση y = 2( χ - 2) .

Η ευθεία ΕΝ έχει εξίσωση y = _..!_(χ - 2) . 2 Λύνοντας τα αντίστοιχα συστήματα, εύκολα βρίσκουμε ότι Μ ( 1: , 1:) και Ν(� ·�}

- ( 16 1 2 ) - ( 4 3 ) γ) i) Είναι: ΟΜ = 5,5 και ΟΝ = 5'5 ,

οπότε ΟΜ = 40Ν , απ' όπου προκύπτει ότι τα σημεία Ο, Μ, Ν είναι συνευθειακά.

ii) Είναι: ΕΝ = ( -�,�) και ΕΜ = ( � · 1:) ,

οπότε 6

(MNE) � .!_ I 5 2 6

3 S l = .!.. l( 72 + 1 8 )1 = 2. και 12 2 25 5

5 5

1 � � 3 (ΟΝΕ) = - 1 5 5 1 = - , οπότε 2 2 ο 5

(ΜΝΕ) = 3 (0ΝΕ) . χ 2 2 χ 2 2

δ) Ισχύει -1 + 1L = -2-+ 22.. = 1 , οπότε τα ση-16 1 2 1 6 12 μεία K(x, , y, ) , Λ(χ2 , y2 ) βρίσκονται πάνω στην έλλειψη. Από την τριγωνική ανισότητα (ΚΛ) :s; (ΟΚ) + (ΟΛ) προκύπτει ότι:

ναι 2 2 2 2 � +1L ς�L +22.. = 1 16 12 16 1 2 οπότε

χ , + y1 2 :s; 16 <=:> (0K) ::; 4 . Ομοίως (ΟΛ) ::; 4 . Επομένως (χ2 - χ 1 )2 + ( y2 - y, )2 ::; (4 + 4)2 = 64 , που εί-ναι το ζητούμενο.

Θέμα 10 Σε ορθοκανονικό σύστημα αξόνων θεωρούμε τα σημεία Α (-1,0) καιΒ (1,0) . Σημείο M (x0 , y0 ) του επιπέδου κινείται ώστε να ισχύει: IMAI · IMBI + 2M02 - ΜΑ · ΜΒ = 18 .

α) Ν α αποδείξετε ότι: i) Το σημείο Μ κινείται σε έλλειψη της

οποίας να βρείτε την εξίσωση. Ι· ι· ) 9 8

> 4 .. ' θ , -2 + -2 - , με την προυπο εση οτι Χο Υο

Χο ,Υο * Ο . β) Να βρείτε την εξίσωση της ευθείας η οποία

διέρχεται από το σημείο y 1 = -1 και τέμνει την έλλειψη στα σημεί Ρ, Τ έτσι ώστε το Σ να είναι το μέσο του τμήματος ΡΤ.

Λύση

α) i) Τα σημεία Α, Β είναι συμμετρικά ως προς την αρχή Ο, οπότε ισχύει: ΜΟ =� (ΜΑ + ΜΒ) . 'Ετσι η δοσμένη ισό-

τητα γράφεται:

IMA I · IMB I + 2± (ΜΑ + ΜΒ )2 - ΜΑ . ΜΒ = 1 8 <=:>

<=:> IMA I · IMB I +�(ΜΑ 2 + ΜΒ2 ) + - - - -ΜΑ · ΜΒ - ΜΑ · ΜΒ = 1 8 <=:>

<=:> ΜΑ2 + ΜΒ2 + 2 1MA I · IMB I = 36 <=:>

<=:> (I ΜΑ Ι + IMB I )2 = 36 <=:> IMA I + IMB I = 6 ' πράγμα που σημαίνει ότι το σημείο Μ κινείται σε έλλειψη με εστίες Α, Β και 2α = 6 , οπότε α = 3 , γ = 1 , β2 = 8 . Άρα το Μ κινείται στην

ΕΥΚΛΕΙΔΗΣ Β ' λθ ' τ4/46

Page 49: Ευκλειδης Β 60

Μαθηματικά Β ' Λυκείου

χ2 y2 έλλειψη - +- = 1 . 9 8 χ 2 Υ 2 ii) Ισχύει: -0- + -0- = 1 , οπότε: 9 8 9 8 ( χ 2 y 2 )( 9 8 ) - +- � 4 <::> _ο_+_ο_ --+- � 4 <::> χ 2 y 2 9 8 χ 2 y 2 ο ο ο ο 8χ 2 9 2 8χ 2 9 2 <::> 1 + -0- + _2Q__ + 1 � 4 <::> -0- + _2Q__ � 2 9y 2 8χ 2 9y 2 8χ 2 ο ο ο ο

<::> 82 χ 4 + 92 Υ 4 - 2 . 8χ 2 . 9y 2 > Ο <::> ο ο ο ο -( 8χ0 2 - 9y0 2 ) � Ο που ισχύει. Επομένως ισχύει και η αρχική που είναι ισοδύ­ναμη με αυτή.

β) Η κατακόρυφη ευθεία που διέρχεται από το Σ, δεν είναι λύση του προβλήματος (γιατί;) . Κάθε άλλη ευθεία η οποία διέρχεται από το Σ είναι της μορφής y + 1 = λ ( χ - 1) . Έστω ότι μία τέ-τοια ευθεία τέμνει την έλλειψη στα σημεία Ρ (χ ι , Υ ι ) και T(x2 , y2 ) ώστε το σημείο Σ (1, - 1) να είναι μέσο του τμήματος ΤΡ. Τότε ισχύουν: χ 2 y 2 χ 2 y 2

_ι + -ι- = 1 ( 1 ) -2-+ -2- = 1 (2) και 9 8 ' 9 8 ' Χ ι + χ2 = 1 Υι + y2 = -1 (3). 2 ' 2 '

Αν από την (2) αφαιρέσουμε την ( 1 ) έχουμε:

Θέμα 1 1

Έστω α, β ακέραιοι για τους οποίους ισχύει: 4(3α + β + 4) = 7 ( α + β + 2) α) Να αποδείξετε ότι:

ί) Ο αριθμός α είναι της μορφής α = 3κ + 2 , κ ε .Ζ και να βρείτε τη μορ­φή του β.

ii) Το υπόλοιπο της διαίρεσης ( Sα - β) : 10 είναι 6.

β2 2 Ο αριθμός - α είναι ακέραιος. 4

β) Α ν ο αριθμός κ είναι πολλαπλάσιο του 5, να αποδείξετε ότι: 2α + β + 3 = πολ.ll

γ) Να βρείτε το υπόλοιπο της διαίρεσης Α:Β

αν:

α) ί)

Α = 1 · 2 · 3 · 4 . . . 40 + 2006 και Β = 41 ( 3β - Sα) + 29 •

Λύση 4 (3α + β + 4) = 7 ( α + β + 2) <::> 1 2α + 4β + 16 = Ία + 7β + 14 <::>

5α + 2 <::> 5α - 3β + 2 = Ο <::> β + -- . 3

Διακρίνουμε τις παρακάτω δυνατές περι­πτώσεις:

, 5α + 2 2 • α = 3κ , κ ε .Ζ . Τοτε -- = 5κ + - ,

3 3 που δεν είναι ακέραιοι, οπότε α * 3κ .

, 5α + 2 • α = 3κ + 1 , κ ε .Ζ . Τοτε -- =

3 5 7 δ ' ' ' = κ +- , που εν ειναι ακεραιοι, οποτε 3

α :;t: 3κ + 1 .

3 2 '71 τ ' 5α + 2 • α = κ + , κ ε � . οτε -- = 3

= 5κ + 4 οπότε α = 3κ + 2 και β = 5κ + 4 , κ ε .Ζ .

ii) Ισχύει: 5α - β = 5 (3κ + 2) = (5κ + 4) =

1 5κ + 10 - 5κ - 4 ::;: 10κ + 6 , οπότε το υπόλοιπο της διαίρεσης ( 5α - β ) : 1 Ο , είναι 6.

βz - α2 1 iii) Έχουμε: = - (β - α ) (β + α) = 4 4

ΕΥΚΛΕΙΔΗΣ Β ' λθ ' τ4/47

Page 50: Ευκλειδης Β 60

Μαθηματικά Β ' Λυκείου

1 =- (5κ + 4 - 3κ - 2) (5κ + 4 + 3κ + 2) = 4 1 = - (2κ + 2) (8κ + 6) = (κ + 1) ( 4κ + 3) 4

κ Ε Ζ οπότε είναι ακέραιος. β) Ισχύει: κ = 5λ , λ Ε Ζ , οπότε

α = 3 · 5λ + 2 = 1 5λ + 2 , β = 5 · 5λ + 4 = 25λ + 4 και 2α + β + 3 = 2 ( 1 5λ + 2) + 25λ + 4 + 3 =

= 55λ + 1 1 = 1 1 (5λ + 1) = πολ. 1 1 ,

διότι ( 5λ + 1) Ε Ζ .

γ) Είναι: β = 4 1 ( 15κ + 12 - 15κ + 1 0) + 29 =

4 1 · 2 + 29 = 1 1 1 και 1 1 1 = 3 · 37 . Επειδή στο γινόμενο 1 · 2 · 3 · 4 . . . 40 περιέχει ως όρους του τους αριθμούς 3 και 37, ισχύει: 1 · 2 · 3 · 4 . . . 40 = πολ. 1 1 1 , οπότε το ζητούμενο υπόλοιπο θα είναι ίσο με το υπόλοιπο της διαίρεσης 2006 : 1 1 1 . Εύκολα πλέον βρίσκουμε ότι το ζητούμενο υ­πόλοιπο είναι 8 ( 2006 = 1 1 1 · 1 8 + 8) .

Θέμα 12 Να αποδείξετε ότι: α) Το τετράγωνο οποιουδήποτε ακέραωυ όταν

διαιρείται με το 8 δίνει υπόλοιπο διαφορετικό του 5.

β) Αν οι αριθμοί β, γ είναι ακέραιοι, τότε σε τουλάχιστον μία από τις εξισώσεις χ2 + βχ + γ = Ο και 2χ2 + (β + l)χ + γ + 1 = = Ο , οι ρίζες δεν είναι ακέραιοι αριθμοί.

Λύση

α) Αν ρ ένας τυχαίο ακέραιος αριθμός, θα απο­δείξουμε ότι ρ2 "# 5 + πολ.8 .

Διακρίνουμε περιπτώσεις:

τις παρακάτων

• Αν ρ = 8κ , Κ Ε Ζ , τότε ρ2 = πολ.8 .

δυνατές

• Αν ρ = 8κ + 1 , Κ Ε Ζ , τότε ρ2 = (8κ + 1)2 =

= 1 + πολ.8 .

• Αν ρ = 8κ + 2 , κ Ε Ζ , τότε ρ2 = (8κ + 2)2 = = 4 + πολ.8 .

• Αν ρ = 8κ + 3 , Κ Ε Ζ , τότε ρ2 = (8κ + 3)2 = = 1 + πολ.8 . (γιατί;)

• Αν ρ = 8κ + 4 , κ Ε Ζ , τότε ρ2 = (8κ + 4γ = = πολ.8 .

• Αν ρ = 8κ + 5 , κ Ε Ζ , τότε ρ2 = (8κ + 5 )2 = = 1 + πολ.8 .

• Αν ρ = 8κ + 6 , Κ Ε Ζ , τότε ρ2 = (8κ + 6)2 = = 4 + πολ.8 .

• Αν ρ = 8κ + 7 , Κ Ε Ζ , τότε ρ2 = (8κ + 7 )2 = = 1 + πολ.8 .

Επομένως, σε κάθε περίπτωση, ισχύει ρ2 * 5 + πολ.8 .

β) Έστω ότι η δεύτερη εξίσωση έχει ρίζες ρ 1 , ρ2 , με ρ1 , ρ2 Ε Ζ . Τότε ισχύουν:

β + 1 ' θ ' β 1 ' ' ρ1 + ρ2 = -2- , οποτε ο αρι μος + ειναι αρ-

τιος, που σημαίνει ότι ο β είναι περιττος. γ + 1 ' θ ' 1 ' ' ρ1 • ρ2 = -2- , οποτε ο αρι μος γ + ειναι αρ-

τιος, που σημαίνει ότι ο γ είναι περιττος. Στην πρώτη εξίσωση τώρα έχουμε: Δ = β2 - 4γ . Ο αριθμός β είναι περιττός, οπότε β 2 = 8λ + 1 , λ Ε Ζ . ΑΝ γ = 2 κ + 1 , κ Ε Ζ τότε

Δ = 8λ + 1 - 4 (2κ + 1) = 8λ - 8κ - 3 =

= 8 (λ - κ - 1) + 5 = 5 + πολ.8 .

Για να έχει η πρώτη εξίσωση ακέραιες ρίζες, πρέπει η διακρίνουσα της, που είναι της μορφής 5 + πολ.8 , να είναι τετράγωνο ακέραιου. Κάτι τέτοιο όμως αποκλείεται, από το (α) ερώτημα, οπότε αποκλείεται και οι δύο εξισώσεις να έ­χουν ακέραιες ρίζες.

ΕΥΚΛΕΙΔΗΣ Β ' λθ ' τ4/48

Page 51: Ευκλειδης Β 60

••ιιιι•τ••• ,_ �"' r ��'" �- Α•�ι-rι

Εnαναληιπιιιά θέματα, ι<ατειίθuνιιης

ΑΣΚΗΣΗ 1η

Έστω συνάρτηση f ορισμένη και συνεχής στο διάστημα [-1,1] , παραγωγίσιμη στο (-1,1) , με f '(χ) :;ι!: Ο για κάθε χ ε (-1,1) - {0} και f(-1) = f(1) = Ο , f(O) = .fi . Εάν η κάθετη σε κάθε σημείο M(x0 ,f(x0 )) του γραφήματος της f με χ0 * -1, Ο, 1 τέμνει τον άξονα ψψ' σε σημείο με τεταγμένη ίση με το μισό της τε­ταγμένης του Μ , τότε να βρεθεί ο τύπος της f και να υπολογισθεί το εμβαδόν που περικλείεται από την γραφική παράσταση της f και τον άξονα n' . (Σημείωση : Κάθετη γραφήματος σε σημείο Μ , θε­

ωρούμε την κάθετη στην εφαπτόμενη του γραφήμα­

τος στο ίδιο σημείο ). ΛΥΣΗ

M(x,,f(x,)}

0.2 -1 -0.5 0 .5

Εφόσον η εφαπτόμενη στο σημείο M(x0 , f (x0 )) έχει συντελεστή κατεύθυνσης f '(x0 ) άρα η κάθετη

θα έχει συντελεστή λ = --1- . f '(x0 ) Η εξίσωσή της κάθετης του γραφήματος της f στο M(x0 , f (x0 )) θα είναι :

1 ψ - f(χσ ) = --- (χ - χσ ) (ε) f '(x0 )

Θ. Χαραλάμπους - Β. Ευσταθίου

Εφόσον το σημείο Ν( Ο, f (;o ) ) επαληθεύει την

(ε) , τότε θα ισχύει : f (xo ) f Χο , f ' -- - (χσ ) =-- η (χσ ) · f (χσ ) = -2χσ ( 1 )

2 f '(x0 ) Δηλαδή η συνάρτηση f επαληθεύει την σχέση f '(x) · f (x ) = -2x , για κάθε χ ε (-1, 1) - {0} ή

2f '(x) · f (x ) = -4x ή [f 2 (x )J ' = -4χ ή

f 2 (x) = -2x2 + c} 2 Ο 2 => - + c = <=:> c = . f (l) = ο

Άρα f\x) = -2x2 + 2 η f 2 (x) = 2(1 - x2 ) 2 0 α­φού χ ε [l, l] Άρα f(x) = �r-2(-l --x�2 ) η f (x) = -�2(1 - χ2 ) . Όμως f (O) = .J2 , άρα τελικά f(x) = �2(1 - χ2 ) αφού η f δεν έχει άλλη ρίζα στο [ - 1 , 1 ] και συνεχής, άρα θα έχει μόνιμο πρόσημο. Για το εμβαδόν πού περικλείεται από το γράφημα της f και τον n' . Αναζητούμε το ολοκλήρωμα :

π π - -2 2

Ω = J f (x)dx =.fi J vfι - x2 dx . π

2 π

2 Θεωρούμε τον μετασχηματισμό :

χ = ημu => dx = συνudu π χ = -1 ημu = -1 u = --' ' 2

χ = 1 ' π ημu = 1 , u = '2

Και το ολοκλήρωμα γίνεται : π

2 Ω = .J2 J �1 - ημ2u · συνudu =

π

2 ΕΥΚΛΕΙΔΗΣ Β' λθ ' τ.4/49

Page 52: Ευκλειδης Β 60

Μαθηματικά για την Γ Λυκείου

π 2 Ji J .J συν2u · συνudu = π 2

π π - -· Ji J συν2u du =Ji J 1 + συν2u du =

π π 2 2 l π π ] Ji 2 2 - J du + J συν2udu =

2 π π -- --2 2

ΑΣΚΗΣΗ 2η Έστω η συνάρτηση f ορισμένη στο διάστημα (Ο,+οο) με f(x) > Ο για κάθε χ ε (Ο,+οο) . Α. Εάν η εφαπτόμενη σε τυχαίο σημείο Μ της

γραφικής παράστασης της f , τέμνει τον · θε­

τικό ημιάξονα Οψ σέ σημείο Β έτσι ώστε το

τραπέζιο ΟΒΜΑ να έχει σταθερό εμβαδόν

ίσον με α2 ,όπου Α σημείο του θετικού ημιά­

ξονα Οχ με την ίδια τετμημένη με αυτήν του

Μ , να υπολογισθεί ο τύπος της f αν είναι

γνωστο ότι τό σημείο ( 1,α2 ) ανήκει στην

γραφική παράσταση της f.

Β .a. , , ( ) f(x) . σεωρουμε την συναρτηση φ χ = -- . χ

i. Να βρεθεί η aσύμπτωτη του γραφήμα­τος της φ .

ii. Να βρεθεί τό εμβαδόν Ω(t) πού περι­κλείεται από το γράφημα της φ , την aσύμπτωτη του γραφήματός της και τίς ευθείες χ = 1 και χ = t > 1 .

iii. Να υπολογισθεί το όριο lim Ω(t) •

ΛΥΣΗ

Β

Μ(χ .. f(χ,) )

σ'

Ο Α(χ,.Ο)

\--++«>

Α. Έστω τυχαίο σημείο M(x0 , f (x0 )) της γραφι-κής παράστασης της f .

Τότε η εξίσωση της εφαπτόμενης στην γραφική παράσταση της f στο Μ , έχει εξίσωση : ψ - f(x0 ) = f 1(X0 )(x - x0 ) η ψ = f 1(X0 )x + f (x0 ) - χ0 • f 1(X0 ) Γιά χ = Ο : ψ = f (x0 ) - x0 · f 1(X0 ) Άρα το B(O, f (x0 ) - x0 · f 1(X0 ) ) και ΟΒ = f(x0 ) - χ0 · f 1(X0 ) Το εμβαδόν του τραπεζίου ΟΒΜΑ είναι : Ε = _!_(ΟΒ + ΜΑ) · ΟΑ = α2 ή

2 (ΟΒ + ΑΜ) · ΟΑ = 2α2 ή (f(x0 ) - X0 · f 1(X0 ) + f(x0 )) · x0 = 2α2 ή (2f (x0 ) - x0 · f 1(X0 ) ) · x0 = 2α2 ή 2f (x0 )x0 - x2 · f 1(Χ0 ) = 2α2 . ο

Διαιρώ με -χ� < Ο 2 2α2

f 1(X0 ) --f(x0 ) = --2 ( 1 ) Χο Χο

Η σχέση ( 1 ) ισχύει για κάθε χ0 ε (Ο, +οο) . Άρα για την συνάρτηση f , ισχύει :

2 2α2 f 1(X) --f(x) = --2 (2) .

χ χ Πολλαπλασιάζω την (2) , με � και έχω: χ

1 1 2 2α2 f (x)- --f(x) = --

x2 χ3 χ4

1 1 ( 1 )1 2α2 f (χ)-+ - f(x) = --

x2 χ2 χ4

( 1 )1 2α2 f(x) ·- = --

χ2 χ4 1 2 f 1 f (χ) ·- = -2α -dx + c χ2 χ4

f(χ) ·- = -2α -- +c 1 2 ( 1 ) χ2 3χ3 2α2 1 f(x) =--+cx2 3 χ

Άρα , τελικά f (x) = cx2 + 2α2 _!_ . 3 χ

Όμως το σημείο ( 1, α2 ) είναι σημείο της γραφι-

κής παράστασης της f , άρα :

f (1) = α2 ή c + 2α2 = α2 ή c = .!α2 και 3 3

ο τύπος της f είναι: f (x) = � ( χ 2 +�) ΕΥΚΛΕΙΔΗΣ Β ' λθ ' τ.4/50

Page 53: Ευκλειδης Β 60

Μαθηματικά για την Γ Λυκείου

Β 'Ε f (x) ι 2 ( 2 ) . στω φ(χ) = -- = -α χ + - . χ 3 χ2 ί. Για την ασύμπτωτη της γραφικής παράστασης φ: lim φ( χ ) = lim [.!.α2 (ι + _3_)] = .!.α2

X -H<XJ Χ X-H<XJ 3 Χ) 3

lim [φ( χ) _.!.α2χ] = ιim [.!.α2 (χ +_3_) _.!.α2χ] = X-H<XJ 3 X-H<XJ 3 Χ2 3

= lim - = 0 ( 2α2 ) x -H<XJ 3Χ2

Άρα η ζητούμενη ευθεία έχει εξίσωση 1 ψ = -α2χ 3

ίί. Για το ζητούμενο εμβαδόν Ω(t) έχουμε : t � 1 ( 2 ) 1 } Ω(t) = J(φ(χ) - ψ):iχ = -α2 χ +-2 --α2χ χ = I I 3 Χ 3

= f2α: dx = 2α2 J�x = 2α2 (-.!.)lt = 2α2 (-!+ ι) I 3Χ 3 I Χ2 3 Χ I 3 t . . . ι · Ω( ) ι· 2α2 ( 1 ) 2α2 111. lffi t = lffi - -- + 1 = -

Χ-Η«> X -H<XJ 3 t 3 ΑΣΚΗΣΗ 3η

Α. Εάν χ ε (Ο, �) , να υπολογισθεί το ολοκλή-

ρωμα : J-1-dx ημχ Β. Να βρείτε τον τύπο της συνάρτησης f για

την οποία ισχύει : ί. f'(χ) · ημχ-f(χ) = l+συνχ , για κάθε

χ ε ( 0,�) ίί. f '(x) · ημx = f(x) · ln (f(x)) , όπου

f(x) > Ο , για κάθε χ ε (Ο, �) . ίίί. f '(x) + f(x) · συνχ = ημχ · συνχ

ΛΥΣΗ

Α.

1 2συν2 � J�dx = J χ

1 χ dx = J χ

2 χ dx = ημ 2ημ-συν- ημ-συν-2 2 2 2

συν2 � 2

Β.

ι 2συν2 �

J 1 dx � J 2 dx = χ χ χ χ 2ημ-συν- ημ-συν-2 2 2 2 συν2 �

2

J 1 1 dx = J-1-(εφ�) ,dχ = 2 χ χ χ 2 2συν - εφ- εφ-2 2 2

= ιη (εφ ;) + c

ί. f '(χ) · ημχ - f (χ) = 1 - συνχ , για κάθε

χ ε ( Ο, �) f '(χ)ημχ - f (x) = 2συν2 �

2 χ 2συν2 -

f '(χ) __ 1_f (x) = 2 -ημχ ημχ

f l 1 (x) --f(x) ημχ

2συν2 � -------'2� = σφ� 2ημ�συν� 2

2 2 f '( 1 χ χ) --f(χ) = σφ- . ημχ 2

Πολλαπλασιάζω με σφ� και έχω : 2 f l χ 1 χ 2 χ (χ)σφ- - σφ-f(χ) = σφ -2 2ημ�συν� 2 2

2 2 f '( ) χ 1 2 χ χ · σφ- - f(χ) = σφ -

2 2ημ2 � 2 2

f '(χ) · σφ� + (σφ�)' f (χ) = σφ2 � 2 2 2 (f (χ ) · σφ ;} = σφ2 ; χ J 2 χ f(χ) · σφ- = σφ - dx + c . 2 2

z X z X συν - 1 -ημ -'Ομως Jσφ2 � dx = J 2dx = J 2dx = 2 2 χ 2 χ ημ - ημ -2 2

ΕΥΚΛΕΙΔΗΣ Β ' λθ' τ.4/51

Page 54: Ευκλειδης Β 60

Μαθηματικά για την Γ Λυκείου

J�ctx - χ + c = -2 ( σφ ;)' dx - x + c = ημ -2

χ = -2σφ- - χ + c 2

Άρα f(x) ο σφ� = -2σφ� - χ + c ο 2 2 Πολλαπλασιάζουμε με εφ� και έχουμε 2

f (χ) = -2 - χ ο εφ� + c ο εφ� ή f(χ) = (c - χ)εφ� - 2 2 2 2

ίί. f '(x) o ημx = f(x) o ln (f (x)) f '(x) � -ημχ = ln (f (x) ) . f (x)

Αν θέσουμε Η(χ) = ln(f (x)) τότε έχουμε :

Η'(χ)ημχ - Η(χ) = Ο ή Η'(χ) --1-Η(χ) = Ο ο ημχ

Πολλαπλασιάζουμε με σφ� και έχουμε : 2 Η'(χ) ο σφ� - 1 σφ� ο Η(χ) = Ο ή

2 χ χ 2 2ημ-συν-2 2 Η'(χ) ο σφ� - 1 Η(χ) = Ο ή

2 2 2 χ ημ -2

Η'(χ) ο σφ ; + ( σφ�)' Η(χ) = 0 ή

( Η(χ) ο σφ ;)' = 0 ή

Η(χ) ο σφ� = c ή 2 Η( χ) = c ο εφ� ο Όμως Η( χ) = ln(f (x)) , άρα

2 χ Χ c·εφ-

ln(f (x)) = c ο εφ- και τελικά : f (x) = e 2

2 ίίί. f '(χ) + f (χ) ο συνχ = ημχ ο συνχ ο

Πολλαπλασιάζουμε την παραπάνω σχέση με eημχ και έχουμε : f '(x) ο eημχ + συνχ ο eημ'f (χ) = ημχ ο συνχ ο eημχ

f '(x) ο eημχ + { eημχ )' f(x) = ημχ ο συνχ ο eημχ

{ f(x ) ο eημχ )' = ημχ ο συνχ ο eημχ

f (χ) ο eημχ = Jημχ ο συνχ ο eημ•ctx

Όμως : Jημχ ο συνχ ο eημ•ctx = Jημχ { eημχ )' dx =

= ημχ ο eημχ - J( ημχ )' eημ•ctx = ημχ ο eημχ - eημχ + c = eημχ (ημχ - 1) + c ο f (χ) ο eημχ = eημχ ( ημχ - 1) + c και τελικά: f (x) = ημχ - 1 + c ο e-ημχ

ΑΣΚΗΣΗ 4η

"Εστω συνάρτηση f , ορισμένη και παραγωγί-

σιμη στο (Ο,π), με f ( �) = Ο και για την οποία

ισχύει : f '(x) = - ( ι + f2(x) ) . Να αποδειχθεί ότι : f(x) = σφχ .

ΑΠΟΔΕΙΞΗ

Θέλουμε να αποδείξουμε ότι f (x) = σφχ ή

f(χ) = συνχ ή ημχ ο f (χ) = συνχ ημχ

Έστω η συνάρτηση Η(χ) = ημχf (χ) - συνχ ο Θα αποδείξω ότι Η( χ) = Ο για κάθε χ του (Ο,π) ο

Έχουμε : Η'(χ) = συνχf (χ) + ημχf '(χ) + ημχ Η'(χ) = συνχf (χ) - ημχ {1 + f\x) ) + ημχ Η'(χ) = συνχf (χ) - ημχf\χ) Η'(χ) = f (x) ( συνχ - ημχf (χ)) Η'(χ) = f(x) ( -Η( χ)) Η'(χ) = -f(x)H(x) Η'(χ) + f(x)H(x) = Ο Θεωρούμε F μία αρχική της f ο Τότε F'(x) = f (x) και F(x) = Jf (x)dx ο Η τελευταία γίνεται : Η'(χ) + F'(x) ο Η(χ) = Ο Πολ/ζω την τελευταία με eF< • > και έχω : Η'(χ) ο eF( x )F(x) + F'(x)eF( x JH(x) = Ο

{Η( χ) ο eF( x ) )' = Ο Η(χ) ο eF< x ) = c Η( χ) = c ο e-F( x )

ΕΥΚΛΕΙΔΗΣ Β ' λθ ' τ.4/52

Page 55: Ευκλειδης Β 60

Μαθηματικά για την Γ Λυκείου

Όμως Η (%) = Ο και επειδή e-F( x ) > Ο , άρα c = Ο. Επομένως Η(χ) = Ο , δηλαδή ημχf (χ) - συνχ = Ο , ημχf (χ) = συνχ και επειδή ημχ =1- Ο , για κάθε

χ ε (0, π) f (x) = συνχ ή f (x) = σφχ . ημχ

ΑΣΚΗΣΗ 5η Έστω ότι η συνάρτηση f: 91 � 91 είναι συνεχής

στο 91 και τέτοια ώστε να είναι χ χ Jcx - t)f(t)dt � Jr(t)dt � ο Vx ε 91 ο ο

Να δείξετε ότι είναι f(x)=O Vx ε 91 ΑΠΟΔΕΙΞΗ lη

χ χ

Έστω Η(χ) = J<x - t)f (t)dt - Jf (t)dt � Ο, Vx > Ο ο ο

Έχουμε για χ = Ο , έχουμε Η(Ο) = Ο άρα Η( χ) � Η(Ο) άρα Hmin = Η(Ο) . Από θεώρημα του Fermat : Η'(Ο) = Ο .

χ χ χ

Η(χ) = χ Jf (t)dt - Jtf (t)dt - Jf (t)dt ο ο ο

χ χ

Η(χ) = (χ - 1) Jf (t)dt - Jtf (t)dt χ

ο ο

Η'(χ) = Jf (t)dt + (x - 1) f (x) - xf (x) ο

χ

Η'(χ) = Jf (t)dt - f (x) , ο

Η'(Ο) = -f (O) = Ο� f (O) = Ο χ

Αν F(x) = Jf(t)dt , τότε : ο

Η'(χ) = F(x) - F'(x) H'(x)e-x = F(x)e-x - e-xF'(x) H'(x)e-x = -{F(x )e-x )' χ

JH'(t)e- 1dt = -F(x)e-x + c ο

χ

tH(t) J� + JH(t)e-1dt = -F(x)e-x + c ο

χ

χΗ(χ) + JH(t)e-1dt = -F(x)e-x + c ο

για χ = Ο , c = O χ

χΗ(χ) + JH(t)e-1dt = -F(x)e-x � Ο ο

Άρα F(x) � Ο , αφού το πρώτο μέλος είναι μή αρ­νητικό .

χ

Όμως F(x) = Jf (t)dt � Ο , από την εκφώνηση ο

χ

Συνεπώς F(x) = Jf (t)dt =0 . ο

Άρα και F'(x) = f (x) = Ο . ΑΣΚΗΣΗ 6η Jf(f"-Έστω η συνεχής συνάρτηση f : JR � JR • Αποδείξτε ότι ισχύει :

Ρ Ρ+κ Jr(t + κ)dt = J f(t)dt α α+κ

Α ν για την f ισχύει : f(x) + f(x + 1) + f(x + 2) + ... + f(x + 2005) = 4012χ

2006 τότε να αποδειχθεί ότι ισχύει : J f(x)dx = 2006

ο

ΑΠΟΔΕΙΞΗ Θέτω t + k = u :::::> d(t + k) = du :::::> . (t + k)'dt = du :::::> dt = du Για t = α , u = α + κ και για t = β , u = β + κ .

β β+κ β+κ Επομένως Jf (t + κ)dt = J f (u)du = J f (t)dt .

α α+κ α+κ Ισχύει f(x) + f(x + 1) + f(x + 2) + . . . + f(x + 2005) = 4012χ Επομένως

I

J(f (x) + f(x + 1) + f (x + 2) + . . . + f(x + 2005) ) dx = ο

I

= J4012xdx ο

I Ι I

Jf (x)dx + Jf (x + l)dx + Jf (x + 2)dx + . . . ο ο ο

I I

. . . + Jf (x + 2005)dx = J401 2xdx ο ο

Σύμφωνα όμως με το πρώτο ερώτημα θα έχουμε :

I 2 3

Jf (x)dx + Jf (x)dx + Jf (x)dx + . . . . Ο Ι · 2

. . . + ·};�χ :a.)dx = J40 12xdx t Ο ο .5 ο

2006

Άρα J f (x)dx = 2006 . ο

ΕΥΚΛΕΙΔΗΣ Β ' λθ ' τ.4/53

Page 56: Ευκλειδης Β 60

Μαθηματικά ΎLα την Γ Λυκείου

• Θέματα επανάληψης στα μαθηματικά

κατεύθυνσης της Γ ' Λυκείου

1 . Δίνεται η συνάρτηση f : :R � :R για την ο­ποία ισχύουν: i) είναι συνεχής στο :R ii) χf (χ) + συνχ = l - χ2ημ.!. , χ * Ο ·

χ α. Να βρείτε το limf (χ) .

χ--+0

β. Να βρείτε τον τύπο της f. γ. Να βρείτε τα όρια: lim f (x) και Iimf(x) .

χ �-χι χ -++α:>

δ. Δείξτε ότι εξίσωση f (χ) = Ο έχει τουλά­χιστον μια θετική ρίζα.

Λύση

α. Για χ * Ο από υπόθεση έχουμε 1 - συνχ 1 f (x ) = - χημ- . χ χ

Για χ * Ο είναι

Ιχημ �� = lχ Ι · Iχημ � � � l x l · 1 = l x l , έτσι:

Ιχημ �� � l x l � - l x l � χημ� � l x l κάι επειδή

lim( - lx l ) = Ο = lim lx l , παίρνουμε Χ--+0 Χ--+0

lim(xημ..!..) = Ο , άρα: χ --+0 χ

limf (χ) = lim 1 - συνχ - Ιίm(χημ..!..) = Ο -Ο = Ο χ --+0 χ--+0 χ χ --+0 ' χ

β. Επειδή η f είναι συνεχής, θα είναι συνεχής και στο Ο, επομένως: f (Ο) = limf ( χ ) = Ο , έτσι:

Χ--+0

f (x ) = { 1 -:υνχ - χημ� ' χ * Ο

Ο χ = Ο

. · Ισυνχ l _ lσυνχ l 1 . . γ. Για χ * Ο ειναι. -χ- - -l-xl- � R , ετσι.

Ι συνχ l 1 1 συνχ 1 ' -χ- � R � -R � -χ

- � R και επειδη Ι . ( 1 ) ο ι· 1 θ · - -ιm --� I = = ιm -� I , α ειναι και

x--+±to Χ x--+±to Χ

Ι. συνχ 0 ιm -- = . x--+±to Χ

Γ. Τσαπακίδης

1 ( ) ημ-Είναι: lim χημ _!_ = lim -1

χ = lim ημψ = 1 χ -+±«> Χ x--+±to _ ψ--+0 ψ

χ (θέσαμε ψ = �) Έτσι:

lim f ( χ ) = lim (..!.. - συνχ - χημ ..!..) = Ο - Ο - 1 = -1 χ--+±οο χ --+±οο Χ Χ Χ

δ. Επειδή lim f (χ ) = -1 < Ο , για πολύ μεγάλες χ -++«>

τιμές του χ, οι αντίστοιχες τιμές του f (χ ) εί­ναι αρνητικές, δηλαδή υπάρχει χ ι > Ο με f (χ ι ) < 0 . Είναι

r(;;J = Ι -σ;!; -!;ημπ = π (ι - συv!;J > Ο

π

Κι επειδή η f είναι συνεχής στο [ � , χ ι J με f (.;} (Χ ι ) < Ο συμπεραίνουμε από το θεώ­

ρημα Bolzano ότι θα έχει μια τουλάχιστον ρίζα στο (� , Χ ι ) .

2. Δίνεται η συνάρτηση f : [α, β] � :R , η οποία είναι συνεχής στο [α,β) παραγωγίσιμη στο (α, β) με f (Ρ) * Ο .Δίνονται επίσης και οι μιγαδικοί αριθμοί z1 = α + βί και z2 = f( α) + if(β) για τους οποίους ισχύει lzι - ίzz l2 = lzι l2 + lzz l2 •

α. Δείξτε ότι � e :R και αf (β) = βf (α) . Zz

β. Α ν α < Ο < β , δείξτε ότι η εξίσωση/ f (χ) = Ο έχει μια τουλάχιστον ρίζα στο (α,β).

ΕΥΚΛΕΙΔΗΣ Β ' λθ' τ.l/54

Page 57: Ευκλειδης Β 60

Μαθηματικά για την Γ' Λυκείου

γ. Αν α > Ο, δ�ίξτε ότι η γραφική παρά­

σταση της f έχει μια τουλάχιστον

εφαπτομένη που περνάει από την αρχή

των αξόνων.

Λύση

α. lzι - iz2 l2 = lzι l2 + lz2 12 = = ( z1 - iz2 ) ( z1 - iz2 ) = z1 � + z2 z2 <:::> <:::> ( Ζ1 - iz2 ) (� + iz2 ) = z1 � + z2 z2 <:::> <:::> Ζ1 Ζ1 + iz1 z2 - iz2 Ζ1 + z2 z2 = Ζ1 Ζ1 + z2 z2 <:::> iz1 z2 - iz2 Ζ1 = Ο<:::> z1 z2 = z2 z 1 <:::>

Είναι z1 z2 = z1 z2 <:::> Ζ1 z2 = z1 z2 <:::> z1 z2 Ε JR <:::> (α + βi) (f (α) - if (β )) Ε JR <:::> <:::> αf (α) - iαf (β ) + iβf (α) + βf (β) Ε JR <:::> αf (α) + βf (β ) + i [βf (α) - αf (β )] Ε JR <:::> <:::> βf (α) - αf (β ) = Ο <:::> βf (α) = αf (β ) .

β. Είναι βf (α) = αf (β ) <:::> f ((α)) = � < Ο (αφού f β β

α < Ο < β ), άρα f (α) = f (β ) < Ο και επειδή η f είναι συνεχής στο [α, β] , σύμφωνα με το θεώ­ρημα Bolzano, η f (χ ) = Ο θα έχει τουλάχι­στον μια ρίζα στο (α, β) .

γ. Θωρούμε τη συνάρτηση g (x ) = f (x ) , που εί­χ ναι συνεχής στο [α, β] και παραγωγίσιμη στο

xf ' (x ) - f (χ ) (α, β ) με g' (x ) = 2 . χ Είναι g (α) = f (α) = f (β) = g (β ) (αφού α β βf (α) = αf (β ) ) . Έτσι σύiιφωνα με το θεώρημα του Rolle θα υπάρχει ξ Ε (α, β ) τέτοιο, ώστε

g' (ξ) = Ο<=:> ξf ' (ξ ); f (ξ) = 0<:::> ξf ' (ξ) = f (ξ) ( l ) . ξ Η εξίσωση της εφαπτομένης της γραφικής πα­ράστασης της f στο σημείο της (ξ, f {ξ)) είναι: ψ - f{ξ) = f ' (ξ) (χ - ξ) , που επαληθεύεται,

λόγω της ( 1 ), για χ = ψ = Ο , δηλαδή περνάει από την αρχή των αξόνων.

3��εται η συνάρτηση f : (Ο, +οο) � JR την οποία ισχύουν: • είναι παραγωγίσιμη στο JR • • f (x) = xf' (x) - x , χ > Ο . • f (l) = l

για

α. Ν α δείξετε ότι f (χ) = χ ln (χ) + χ , χ > Ο β. Να μελετήσετε την f ως προς τη μονοτο-

νία και τα ακρότατα.

γ. Να βρείτε το σύνολο τιμών της f. δ. Να μελετήσετε την f ως προς τα κοίλα

και τα σημεία καμπής. ε. Ν α βρείτε το εμβαδό Ε λ του μέρους του

επιπέδου, που περικλείεται από τη γρα­

φική παράσταση της f, τον άξονα χ'χ

και τις ευθείες χ = 1 και χ = λ με ο

λl ' β ' . < < - και μετα να ρειτε το lιm Ελ • e λ-+ο•

Λύση

α. Για κάθε χ > Ο , έχουμε: f (χ ) = xf ' (χ ) - χ <:::> xf ' (χ ) - f (χ ) = χ <:::>

xf ' (x ) - f (x ) 1 <:::> = -χ2 χ <o> ( f�x )} � ( Ιηχ )' <ο> f�x ) � In x + c <o:> <:::> f (χ ) = χ ln χ + cx ( 1 ) Για χ = 1 η ( 1 ) γίνεται: f ( 1) = Ι · ln 1 + c · 1 <:::> 1 = c , άρα: f (x ) = x ln x + x , χ > Ο .

β. Για κάθε χ > Ο έχουμε: f ' (χ ) = (χ ln χ + χ )' = ln χ + χ..!.. + 1 = ln χ + 2 χ f' (χ ) = Ο <:::> ln χ = -2 <:::> χ = e -2 Η μονοτονία και τα ακρότατα της f φαίνονται στον πίνακα μεταβολών.

χ ο -2 e -Γ( χ) - ? + f(x) � � /

ολικό ελαχιστό

γ. lim f (χ ) = lim (χ ln χ + χ ) = Ο + Ο = Ο χ -+0+ χ -+0+

ΕΥΚΛΕΙΔΗΣ Β ' λθ ' τ.4/55

Page 58: Ευκλειδης Β 60

Μαθηματικά για την Γ' Λυκείου

-«) (-) ( · ι · ( ι ) ι · ιη χ _, γιατι: ιm χ n x = ιm - =

Χ ----?0+ Χ -:-+0+ Ι

, ι χ

( ιη χ ) -= ιim -- = ιim -x- = lim (-x) = O )

Η0+ (�)Ή ΗΟ+ :2 ΗΟ+

ιim f (x) = ιim(x ιη χ + χ) = +οο χ-+οο χ �οο

(γιατί ιim χ = +οο και lim ( ιη χ ) = +οο . χ -++οο χ -++σ:>

Συνδυάζοντας τα προηγούμενα όρια και τον πίνακα μονοτονίας παίρνουμε:

χ ο -2 e + Γ( χ) - � + f(x) �/.e��

Άρα f (Α) = [ -e -2 , +οο) δ. Για κάθε χ > Ο είναι:

, Ι f " (χ ) = ( ι η χ + 2) = - > Ο χ άρα η f στρέφει τα κοίλα άνω στο Α = (Ο, +οο) , γι' αυτό η γραφική της παράσταση δεν έχει σημεία καμπής.

ε. Είναι f (χ ) = Ο <:::> χ ιη χ + χ = Ο<:::> <:::> χ ( ιη χ + Ι) = Ο <:::> ιηχ = -Ι ( αφοiι χ > Ο )

- ι

Ι

<=:> x = e = - . e

I f�x) ! � λ e-' 1 +

t � f Κατ' αρχήν παρατηρούμε ότι για κάθε χ ε (ο, e- 1 ) ισχύει f (x ) < O διότι Ο < χ < e- 1 � ιη χ < -Ι � ιη χ + Ι < Ο � χ (ιη χ + Ι) < Ο � f (χ ) < Ο .

I Όμοια δείχνουμε ότι: e- 1 < χ � f (χ ) > Ο . Έτσι σχηματίζουμε τον πίνακα προσήμου της f. Είναι:

ι

Ελ = ί lf ( x )J ctx = f Jf (x )J ctx + f Jf ( x )J ctx = I

= -ff ( χ ) dx + l f { χ ) dx ( 1 ) . Βρίσκουμε μία αρχική της f (χ ) . Έχουμε:

f χ2 f { χ ) dx = J(x ιn x + χ ) dx = Jx ιn xdx +Τ , J( x2 ) χ2 χ2 χ2 , χ2 = - ιη χdχ +- =-ιη χ - J-(ιηχ ) dx +-2 2 2 2 2

χ2 Ι J χ2 χ2 =τ(Ιη χ + Ι) -2 xdx =2(ιn x + I) -4+ C χ2 χ2 =-(2 ιη χ + 2 - I) + C =-(2ι η χ + Ι) + C . 4 4 Έτσι συμπεραίνουμε ότι μια αρχική της f (χ )

2 είναι η F(x ) =�(2ιn x + I) + C . Και συνεπώς: 4 Ελ = -F(±) + F(λ) + F(I) - F(±) = F(λ) + F(I) - 2F(±)

λ2 Ι ι Κι επομένως: Ελ = -( 2ιηλ+ Ι) + - + -2 . 4 4 2e Κι επειδή : ι . [λ2 (2ι λ Ι)] Ι ι · 2ιηλ+ Ι <:ι ιm - η + = - ιm = λ_.ο• 4 4 λ_.ο• Ι

ι2 2

Ι . (2ιηλ+1 )' 1 . λ - ιιm = - ιιm- = 4 λ->ο· ( Ι )' 4 λ_.ο· -2 λ2 ').!

- 1 ι · λ2 ο ' ' =- ιm = , συμπεραινουμε οτι: 4 λ-->0+

lim Ελ = .!_ +_Ι_ = e2 + 2 . λ->ο• 4 2e2 4e2

4. Δίνεται η συνάρτηση f : JR � JR για την ο­

ποία ισχύουν: • f (χ) � Ο για κάθε χ ε JR • •

υπάρχουν Χι , χ2 e JR με Χι < χ2 τέτοια,

ώστε f { Χι ) = Ο = f { χ2 ) , η f έχει τρίτη παράγωγο στο JR •

Δείξτε ότι:

α. f ' { Χι ) = Ο = f' { χ2 ) β. υπάρχει χ3 e (χι , χ2 ) με f' {x3 ) = 0 , γ. υπάρχουν ξι ,ξ2 e (χι ,χ2 ) με ξι * ξ2 τέ­

τοια, ώστε f" {ξι ) = Ο = f" {ξ2 ) , δ. υπάρχει ξ e (Χι ,χ2 ) με f<31 (ξ) = Ο .

ΕΥΚΛΕΙΔΗΣ Β ' λθ ' τ.4/56

Page 59: Ευκλειδης Β 60

Μαθηματικά για την Γ Λυκείου

Λύση

α. Ισχύει: f ( x ) 2 0 = f (x , ) = f(x2 ) για κάθε χ Ε IR , άρα η f παρουσιάζει ολικό ελάχιστο το Ο στα χ 1 , χ 2 και είναι παραγωγίσιμη στα χ 1 , χ 2 , οπότε σύμφωνα με το θεώρημα Fermat θα είναι: f ' ( x , ) = O = f ' ( x2 ) .

β. Επειδή η f είναι παραγωγίσιμη στα [χ 1 , χ 2 ] και f (χ , ) = f ( χ2 ) , σύμφωνα με το θεώρημα του Rolle, θα υπάρχει χ3 Ε ( χ 1 , χ2 ) τέτοιο, ώστε f ' ( x 3 ) = 0 .

γ. Επειδή η f' είναι πραγωγίσιμη στα [χ 1 , χ 3 ] και [χ 3 , χ 2 ] , f ' ( x , ) = O = f ' ( x 3 ) και f ' ( χ3 ) = Ο = f ' ( χ2 ) , σύμφωνα με το θεώρημα του Rolle για την f' , στα [χ , , χ3 ] και [χ3 , χ2 ] θα υπάρχουν ξ, Ε χ 1 , χ3 και ξ2 Ε χ3 , χ 2 τέτοια, ώστε f " (ξ , ) = Ο = f " (ξ2 ) .

δ. Η f" είναι παραγωγίσιμη στο [ ξ1 ,ξ2 J και f " (ξ , ) = f " (ξ2 ) , επομένως, σύμφωνα με το θ. Rolle, θα υπάρχει ξ Ε (ξ, ,ξ2 ) c (χ , , χ2 ) τέτοιο, ώστε: f (3 ) (ξ) = Ο .

5. Δίνεται η συνάρτηση f : 1R � IR , για την ο­

ποία ισχύει: f (χ) = rx �dt ' με α > ο . Jo e + α

Δείξτε ότι

α. Η f αντιστρέφεται . β. Η εξίσωση f (χ) = Ο έχει μοναδική ρίζα

στο 1R • Ποια είναι; 2α r2 2α ! 2α 2α γ --dx< --dt- --dt <--.

· e2 + α e' + α e' + α e + α

δ. Οι εικόνες των μιγαδικών αριθμών z, με

z = f (χ) + xi είναι σημεία της γραφικής

παράστασης της Γ' • ε. α = ι ' όταν lz + ψς ι� - ί ι .

Λύση

α. Για κάθε χ Ε IR είναι f ' ( x ) = ( r· �dι)' =�(αφού η συνάρ-k e1 + α e' + α τηση g ( ι ) =� είναι συνεχής στο IR ), αλ­e1 + α

λά �>0 για κάθε x E IR , (αφού α > Ο ), e' + α έτσι f ι ( χ ) > ο ' επομένως η f είναι γνησίως αύ-ξουσα στο IR , άρα και 1 - 1 , οπότε είναι αντι­στρέψιμη .

β. Παρατηρούμε ότι f (Ο) = f' �dt = Ο , δη-.b e + α λαδή το Ο είναι ρίζα της f (χ ) = Ο , που είναι μοναδική , γιατί η f είναι γνησίως αύξουσα στο IR .

γ. Επειδή η f είναι παραγωγίσιμη στο IR θα είναι παραγωγίσιμη και στο [ 1, 2] , οπότε από Θ.Μ.Τ. θα υπάρχει ξ Ε (1, 2) τέτοιο, ώστε f ' (ξ) = f (2) - f (1) = f (2) - f (1) = 2 - 1

!z 2α 1 2α = --dt - --dt e1 + α e1 + α ( 1 ) Είναι: 1 < ξ < 2 <=> e1 < eξ < e2 <=> <=> e + α < eξ + α < e2 + α <=>

1 1 1 <:::? -- > -- > --<:::? e + α eξ + α e2 + α 2α 2α 2α <:::? -- < -- < -- <:::? e2 + α eξ + α e + α 2α f '(ξ) 2α <:::? -- < <--e2 + α e + α 2α !z 2α 1 2α 2α <:::? -- < --dt - --dt <--e2 + α e1 + α e1 + α e + α

λόγω της ( 1 ) . δ. Οι εικόνες των μιγαδικών αριθμών z είναι τα

σημεία Μ(ψ, χ), με ψ = f (χ ) , οπότε Γ' (ψ) = Γ' (f ( x )) = χ , άρα τα σημεία Μ(ψ,χ) ανήκουν στη γραφική παράσταση της Γ' .

ε. Θέτουμε z = κ + λί , οπότε Ι z + 1 1 :::; li - il <=> 1( κ + 1) + λi l :::; Ι κ - (λ + ΨI <=> J(κ + 1)2 + λ2 ::; Jκ2 + (λ + 1)2 <=> <=> (κ + 1)2 + λ2 ::; κ2 + (λ + 1)2 <=> κ :::; λ <=> IR Ε ( z) :::; Im ( z)

<=> f (x ) ::; x <=> f (x ) - x ::; O ( 1 ) . Θεωρούμε τη συνάρτηση g(x) =f ( x) - x , που είναι παραγωγίσιμη στο IR με g'(x) =f(x) -1 (2).

ΕΥΚΛΕΙΔΗΣ Β' λθ ' τ.4/57

Page 60: Ευκλειδης Β 60

Μαθηματικά για την Γ' Λυκείου

( 1 ) <:=> g (x ) s g (O) (αφού (! 2α g (O) = f (0) - 0 = .b e• + α dt = O ) .

Έτσι η g παρουσιάζει μέγιστο στο Ο και είναι παραγωγίσιμη σ' αυτό, οπότε σύμφωνα με το θεώρημα του Fennat θα έχουμε: g' (O) = Ο <::::> f ' (0) - 1 = Ο <::::>

2α <::::> -- = 1 <::::> 2α = α + 1 <::::> α = 1 e0 + α I

6. Δίνεται η συνάρτηση f (χ) = J: e1-1dt •

α.

β.

γ.

δ.

ε.

Να βρείτε το πεδίο ορισμού της.

Να τη μελετήσετε ως προς τα κοίλα και

τα σημεία καμπής.

Να βρείτε την εφαπτομένη της γραφικής της παράστασης στο χ0 = 2 • Δείξτε ότι f (χ) s ex - 2e για χ > ι .

Α ν Ε είναι το εμβαδόν του χωρίου που

περικλείεται από τη γραφική παράστα-

ση της f, τον άξονα χ 'χ, και τις ευθείες χ = 2 και χ = 4 , δείξτε ότι Ε s 2e •

Λύση

Η . 1 ' . α. συναρτηση ι:ι ειναι συνεχης στο e

( -οο, 1) υ ( 1, +οο) . Για να ορίζεται η f, πρέπει τα άκρα της ολοκλήρωσης 2, χ να ανήκουν στο ίδιο διάστημα του πεδίου ορισμού της �-Ι . Επειδή 2 Ε (1, +οο) πρέπει και το e

χ Ε (1, +οο) . Άρα το πεδίο ορισμού της f είναι το Α = (l, +oo) .

I

β. Για κάθε χ Ε Α είναι: f ' ( χ ) = e � , άρα f ' ( 2) = e

γ. Και η εξίσωση εφαπτομένης είναι: ψ - f ( 2) = f ' ( 2) (χ - 2) <::::> ψ = ex - 2e .

δ. Για κάθε χ Ε Α , είναι ( _..!..._ )' _..!..._ ( 1 )' e '�1 f " ( x ) = e •-Ι = e•-Ι -- = 2 < 0 , χ - 1 ( χ - 1) άρα η f είναι κοίλη στο Α και συνεπώς η γρα­φική της παράσταση θα βρίσκεται κάτω από

οποιαδήποτε εφαπτομένη της, γι' αυτό: f (χ ) s ex - 2e για χ > 1 .

I

ε. Για κάθε Χ Ε [2, 4] είναι f (Χ ) = r eϊ=ίdt > 0 και f (χ ) s ex - 2e , έτσι Ε = rιf (x ) l ctx = l f (x ) dx s ί (ex - 2e) dx =

= [ e�' � 2ex Ι = 2e . 7. Δίνεται η συνάρτηση

Α.

Β.

{αe• - χ + ι, x s O f (χ) = με α, β Ε 1R .

x2 1n x + β, χ>Ο Ν α βρείτε τις τιμές των α, β ώστε η f να εί­

ναι παραγωγίσιμη στο 1R . Για α = ι και β = 2 :

α. Να βρείτε την εφαπτομένη της γραφικής

παράστασης της f στο σημείο της

Μ{ O,f (O) ) .

β. Να εξετάσετε αν υπάρχουν aσύμΠτωτες

της γραφικής της παράστασης της συ­

νάρτησης.

γ. Ν α δείξετε ότι f (χ) > � για κάθε χ > Ο . 2

δ. Να υπολογίσετε το εμβαδό του χωρίου

που περικλείεται από τη γραφική παρά­

σταση της f, τον άξονα χ'χ και τις ευ­

θείες χ = -ι και χ = ι . Λύση

Α. Για χ < Ο η f είναι παραγωγίσιμη ως αλγεβρι­κό άθροισμα παραγωγισίμων συναρτήσεων. Για χ > Ο η f είναι παραγωγίσιμη ως άθροισμα παραγωγίσιμων συναρτήσεων. Έτσι: η f είναι παραγωγίσιμη στο 1R , αν εξασφαλίσουμε να είναι παραγωγίσιμη στο Ο, αλλά τότε θα πρέπει να είναι και συνεχής στο Ο

<::::> lill! f (χ) = lim f (χ) = f (ο) <::::> lill! (αe' - χ + ι) = χ-+0 χ-+0+ χ....Ο

= lim (x2 ln x + β) = α · e0 - 0 + 1 χ �ο+

<:=> α · e0 - 0 + 1 = 0 + β [γιατί ( ) ln x (:) ( In x )' lim x2 ln x = lim - = lim -- =

χ -+0+ χ -+0+ :2 χ -+0+ ( :2 )'

ΕΥΚΛΕΙΔΗΣ Β ' λθ ' τ.4/58

Page 61: Ευκλειδης Β 60

Μαθηματικά Ύια την Γ Λυκείου

ι -2 = lim__x_2 = - lim -χ = 0 ] <::::> α + ι = β ( ι ). χ-->0+ χ-->0+ 2

χ3 Η f είναι παραγωγίσιμη στο Ο <::::> lim f (χ) - f (Ο) = lim f (χ) - f

(Ο) Ε JR Χ-->0- Χ - 0 Χ-->0+ Χ - ο . αeχ - χ + ι - (α + ι) <::::> lιm = Χ-->0- Χ

. χ 2 ln χ + β - (α + ι) = lιm Ε JR χ

αeχ - χ - α <::::> lim = lim (χ ln χ) Ε JR χ-->0- χ χ-->0+ (αφού β = α + ι )

( )' ' αeχ - χ - α ln x <::::> lim = lim _( _) Ε JR <::::> (χ )' χ-->0+

( � )'

ι Ι. αeχ - 1 ι· � liD <::::> ιm --= 1m -ι- Ε m. <::::> Χ-->0- ι Χ-->0•

χ2

β = lim f(x) = lim (ex - χ + ι) = Ο + οο + ι = +οο χ 4--«> χ �-ς()

Άρα η γραφική παράσταση της f δεν έχει aσύ­μπτωτη στο +οο . Για κίχθε χ Ε (0, +οο) έχουμε λ = lim

f (χ ) = lim x2 ln χ + 2 = χ->+«> χ χ->+«> χ

= lim (x ln x + �) = +οο( +οο ) + 0 = +οο . Χ->+«> Χ

Άρα η γραφική παράσταση της f δεν έχει aσύ­μπτωτη στο +οο . Επειδή η f είναι συνεχής, ως πα­ραγωγίσιμη, στο JR = ( -οο, +οο) η γραφική της πα-ράσταση δεν έχει ούτε κατακόρυφη aσύμπτωτη οπότε γενικά δεν έχει aσύμπτωτες. γ. Για χ > Ο είναι: f ' (x ) = (x2 ln x + 2)' = 2x ln x + χ2 • .!. = 2x ln x + χ

χ ι f ' (x ) = Ο<::::> x (2 1n χ + 1) = Ο<::::> ln χ = --<::::> 2

_.!_ 1 <::::> x = e 2 = � ·

<::::> Ii� ( αeχ - ι) = li� (-χ ) Ε JR <::::> α - ι = ο<::::> α = ι Ο πίνακας μεταβολών της f για χ > Ο είναι ο πα-χ-->ο χ-->ο

·

οπότε β = 2 . Β. Για α = ι και β = 2 είναι:

f {x ) = {e: - χ + ι, χ � Ο . χ· ln χ + 2, χ>Ο

α. Είναι f {O) = e0 - Ο + ι = 2 . Η f είναι παραγω­γίσιμη στο JR για α = ι και β = 2 , άρα και στο Ο,

f {x ) - f {O) ex χ + ι 2 (:) f ' (Ο) = lim = lim - -= Χ-->0- Χ - ο Χ-->0- Χ

. ( ex - χ + ι)' . ex - ι = lιm = lιm--= 0 . χ ..... ο- ( χ )' χ ..... ο- ι

Η εξίσωση της εφαπτομένης στο Μ είναι: ψ - f (Ο) = f ' (O) (x - 0) <::::> ψ - 2 = 0 <::::> ψ = 2 .

β. Για κάθε χ Ε ( -οο, Ο) έχουμε . f {x ) . eχ - χ + ι (:) λ = lιm --= lιm =

χ

. ( ex - χ + ι)' . ex - ι = lιm . = I1m --=0 Χ-->--α:> (Χ )' Χ-->--α:> ι

με

ρακάτω: χ ο _fe -ω

Γ( χ) - 1 + f(x) � ολ:κό / ελαχ.

Αφού για χ > Ο ισχύει f {x ) � f (�) = (�)\n � + 2 = ±( -Ξ) + 2 =

= --1 + 2 = 4e - 1 > 4e- e = 3e = �

2e 2e 2e 2e 2 '

άρα f (χ) > � , για κάθε χ > Ο . 2

δ. Για χ � ο είναι f (χ ) = ex - χ + 1 > ο . Για χ > Ο είναι f (χ) = χ 2 ln χ + 2 > � > Ο .

2 Άρα f (χ ) > Ο για κάθε χ Ε JR , έτσι:

Ε = fι lf (x )l dx = L/ (x)dx .

Δεν μπορούμε να γράψουμε = L/ (x )dx = l/ (x)dx + tf (x )dx =

= L1 (ex - χ + ι}dχ + ί (x2 ln x + 2)dx ,

γιατί το t ln xdx δεν ορίζεται στο [Ο, ι ] . ΕΥΚΛΕΙΔΗΣ Β' λθ ' τ.4/59

Page 62: Ευκλειδης Β 60

Μαθηματικά για την Γ Λυκείου

Επειδή η f είναι συνεχής στο [-1 , 1) , θα έχουμε: l/ (x )dx = F(1 ) - F(- 1) , όπου F μία παράγουσα της f.

χ2 Για χ < Ο είναι F(x) = J(e' -x+ 1)dx=e' -2+x+c . Για χ > Ο είναι F( χ ) � J( χ' ln χ + 2 )dx � ( �' )' ln xdx + 2χ = χ3 χ3 , =-ln x - J-(In x ) dx + 2χ = 3 3 χ3 1 3 1 =-ln x -- Jx -dx + 2x = 3 3 χ χ3 1 χ3 1 =-lnx -- Jx2dx + 2x = =-ln x --χ3 + 2χ + c . 3 3 3

9 2 !e' -�+ χ + c χ < 0 'Ετσι: F(x ) = 2 1 ,

χ3 χ3 -ln x --+ 2x + c χ > 0 3 9 2 '

Άλλα η F ως αρχική της f πρέπει να ειναι παραγω­γίσιμη, άρα και συνεχής, επομένως: lim_ F( χ) = lim F( χ) � lim_ (e' -� + χ + c1 ) = χ ..-.+0 χ -+0+ Χ-+0 2 ( χ3 χ3 ) = lim -ln x --+ 2x + c2 � 1 + c1 = c2 (γιατί χ -+0+

3 9

lim (� ln χ) = lim � lim ( x2 ln χ) = Q · Ο = Ο ) . χ -+0+

3 χ -+0+

3 χ -+0+

3 !e' -�+ x + c1 , x :s; O Επομένως: F( χ) = χ

χ3 χ3 -ln x --+ 2x + I + c χ > 0 3 9

I '

Άρα Ε = F(1) - F( -1) = =-ln 1 --+ 2 · I + I + c - e- 1 - - - 1 + c 13 Ι3 ( 1 ) 3

9 I 2 I

1 1 Ι Ι 79 1 = - · 0 - - + 2 + 1 + c -- + - + 1 - c =---3 9 1 e 2 1 1 8 e

• Επανάληψη στα Μαθηματιιιό rενιιιής Παιδείας

του Δημήτρη Μυλωνά

«Η περιπέτεια της γνώσης θα μπορούσε να παρομοιαστεί μ ' ένα παιχνίδι : ένα παιχνίδι, στο οποίο ο a­ντίπαλός μας είναι το σύμπαν ή κάτι πίσω από το σύμπαν. Μπορούμε να κερδίσουμε - ή έχουμε κερδίσει -πολλούς πόντους, αλλά είναι αμφίβολο αν μπορούμε να κερδίσουμε την παρτίδα. Εδώ δεν έχουμε καταφέρει να μαντέψουμε (κι ίσως αυτό να είναι η καλύτερη μορφή να εκφράσουμε το μυστήριο) περί τίνος παιχνιδιού πρόκειται . . . »

«Ο αλγόρ ιθμος της μελαγχολίας» Κάρλο Φραμπέτι

Κάθε παιχνίδι έχει τους δικούς του κανόνες. Τα Μαθηματικά θέτους τέτοιους κανόνες στην περιπέτεια της γνώσης. Όποιος ταλαιπωρείται και καταγίνεται με τα προβλήματα των Μαθηματικών μετατρέπεται αυτόμα­τα σε πρωταγωνιστή της περιπέτειας.

ΑΣΚΗΣΕΙΣ ΑΝΑΠΤΥΞΗΣ 1. Μια εταιρεία παράγει χ ποσότητα από ένα

προϊόν, χ > α όπου α θετική παράμετρος. Οι εισπράξεις από την πώληση της ποσότη­τας χ του προϊόντος, δίνονται από τη συ-νάρτηση R(x) = χ 2 - 2χ - α (1), ενώ το κό­στος για την παραγωγή της ποσότητας χ, δίνεται από τη συνάρτηση C(x)=

= χ2 - 5χ + (χ - α) · Ιη(χ - α) (2), χ>α. Το

κέρδος από την πώληση της ποσότητας χ

του παραπάνω προϊόντος είναι, σε χιλιάδες ευρώ, Ρ(χ) = R(x) - C(x) (3), χ>α. α) Να βρείτε την ποσότητα χ0 που πρέπει

να πουληθεί, ώστε το κέρδος να γίνεται μέγιστο καθώς και το μέγιστο αυτό κέρ­δος, το οποίο συμβολίζουμε με Ρ( α).

β) Ένας οικονομολόγος ισχυρίστηκε : « αν η παράμετρος α μεγαλώνει, τότε το μέ­γιστο κέρδος Ρ(α) μεγαλώνει ». Να δικαιολογήσετε τον ισχυρισμό του.

ΕΥΚΛΕΙΔΗΣ Β' λθ' τ.4/60

Page 63: Ευκλειδης Β 60

Μαθηματικά για την Γ' Λυκείου

γ) Αν η παράμετρος α παίρνει τιμές στο σύνολο Ω = {1 ,2, • • • , 20}, να βρείτε την πιθανότητα του ενδεχομένου Α : « το μέγιστο κέρδος να είναι πάνω από 20.000 ευρώ»

Λύση α) Αντικαθιστώντας τις ( 1 ) και (2) στην (3)

προκύπτει η συνάρτηση κέρδους Ρ(χ) = 3χ - α - (χ - α) ·ln(x - α), χ > α. Παραγωγίζοντας έχουμε: Ρ ' (χ) = 2 - ln(x - α) Ρ ' (χ) = Ο <::::> 2 - 1n(x - α) = Ο <::::> ln(x - α) = 2 <::::> χ - α = e2 <::::> χ = α + e2 Ρ ' (χ) > Ο <::::> 2 - ln(x - α) > Ο <::::> ln(x - α) < 2 <::::> χ - α < e 2 <::::> χ < α + e2 Οι ρίζες και το πρόσημο της Ρ ' φαίνονται στον παρακάτω πίνακα :

χ α + e2 +οο α Ρ ' (χ) ι, + ! I -Ρ(χ) ι, � I �

Δηλαδή το κέρδος γίνεται μέγιστο όταν που­ληθεί ποσότητα χ 0 = α + e 2 και το μέγιστο αυτό κέρδος είναι Ρ( α) = 3(α + e 2 ) - α - (α + e2 - α) . ln(α + e2 - α) <::::>

Ρ(α) = 3α + 3 e 2 - α - e2 ·ln e 2 <::::> Ρ(α) = 2α + 3 e2 - 2 e 2 <::::> Ρ( α) = 2α + e2

β) Επειδή η συνάρτηση f(x) = 2χ + e2 έχει f'(x) = 2 > Ο, δηλαδή είναι γνησίως αύξου­σα, αυτό σημαίνει ότι το μέγιστο κέρδος Ρ( α) = 2α + e 2 , καθώς η παράμετρος α με­γαλώνει, θα αυξάνεται και αυτό.

γ) Για να είναι το μέγιστο κέρδος μεγαλύτερο από 20.000 ευρώ αρκεί Ρ( α) > 20 <::::> 2α + e2 > 20 <::::> 2α > 20 - e2

e2 <::::> α > 1 0 - - <::::> α > 6,328 2 ή αλλιώς α = 7, 8, 9, . . . , 1 9, 20. Κατά συνέπεια Α = { 7,8,9, . . . , 1 9,20 }

Οπότε Ρ(Α) = Ν(Α) = .!i = Ο 7 . Ν(Ω) 20 '

2. Δίνεται η συνάρτηση

f(x) = (tι - X)3 + (t2 - χ)3 + ... + (tν - χ)3

χ e R, όπου t1 , t2 , . • • , tν οι παρατηρήσεις \

ενός δείγματος με τυπική απόκλιση s>O και μέση τιμή χ .

ν Σ t�

α) Να αποδείξετε ότι s2 = i=t - (χ}2 (1) . ν

β) Να αποδείξετε ότι

f '(x) = -3ν { χ2 - 2χ . χ + (s2 + (χ)2 }) γ) Να αποδείξετε ότι η f είναι γνησίως φθί­

νουσα στο R.

δ) Να βρείτε το σημείο χ στο οποίο η f έχει το μέγιστο ρυθμό μεταβολής.

Λύση α) Από τον τύπο

Σι. [ ν )2

s2 = .!. :Σι; - i=Ι • προκύπτει ότι ν i=Ι ν

ν Σι;

ή ισοδύναμα s 2 = .i.::L_ -ν

ν Σ ι;

κά s2 =.i.::L_ - (xY . ν

2

β) f '(x) = -3(ι 1 - χ)2 - 3(ι2 - χ)2 - 0 0 0 -3(ιν - Χ)2

και τελι-

= -3(ι� - 2ι1χ + χ 2 ) - 3(ι� - 2ι2χ + χ 2 ) - ο ο · -3(ι� - 2ινχ + χ2 )

3( 2 2 2 ) 6( = - ι1 + ι2 + 0 0 0 + ιν + ι1 + ι2 + ο ο • +tν )χ - 3νχ2

ν = -3νχ2 + 6νχ · χ - 3Σι; (2)

i=l

ν

και επιλύοντας τον τύπο (1) ως προς Σ ι; i = l

και με αντικατάσταση στη (2) προκύπτει ότι f '(x) = -3νχ2 + 6νχ · χ - 3ν (s2 + (χ)2 ) <::::> f '(χ) = -3ν {χ2 - 2x · x + (s2 + (χ)2 ))

ΕΥΚΛΕΙΔΗΣ Β ' λθ ' τ.4/61

Page 64: Ευκλειδης Β 60

Μαθηματικά για την Γ Λυκείου

γ) Η διακρίνουσα του τριωνύμου x2 - 2x · x + {s2 + (x)2 ) είναι Δ = 4(χ)2 - 4(s2 + (χ)2 ) = 4(Χ)2 - 4s2 - 4(χ)2 = -4s2 < Ο οπότε για κάθε πραγματικό αριθμό χ ισχύει χ2 - 2χ . χ + (s2 + (χ)2 ) >0 ή f '(x) = -3ν ( χ2 - 2χ · χ + ( s2 + (χ)2 )) <ο αφού ν>Ο και κατά συνέπεια η f είναι γνη­σίως φθίνουσα στο R.

δ) Ο ρυθμός μεταβολής της f ως συνάρτηση του χ είναι : g(x) = f '(x) = -3ν {x2 - 2x · x + {s2 + (χ)2 )) . Ισχύει g'(x) = -3ν ( 2χ - 2Χ) = -6ν ( χ -Χ) g'(x) = Ο<:::> -6ν ( χ - χ) = Ο <:::> χ = χ g'(x) > Ο <:::> -6ν ( χ - χ) > Ο <:::> χ < χ Οι ρίζες και το πρόσημο της g ' φαίνονται στο ν παρακάτω πίνακα :

χ χ +ω

g "(x) + τ -

g(x) � I � Οπότε η g (άρα ο ρυθμός μεταβολής της t) μεγιστοποιείται όταν χ = χ .

3. Δίνεται η συνάρτηση με τύπο s 3 χ 2 -f(x) = - · x - - · χ - 2x · x + s χ eR όπου 3 2

' '

χ και s η μέση τιμή και η τυπική απόκλιση αντίστοιχα ενός δείγματος με χ > Ο : Α ν η γραφική παράσταση της f δέχεται στο ση­μείο με τετμημένη χ0 = 10 οριζόντια εφα-

πτομένη τότε : α. Ν α υπολογίσετε το συντελεστή μεταβο­

λής του δείγματος και να εξετάσετε αν αυτό είναι ομοιογενές.

β. Να μελετήσετε την f ως προς τη μονοτονία και να βρείτε τα σημεία του πεδίου ορι­σμού της f στα οποία η f παρουσιάζει α­κρότατα, καθώς και το είδος των ακροτά­των.

γ. Με δεδομένο ότι η καμπύλη κατανομής του δείγματος είναι περίπου κανονική να υπολογίσετε το εύρος R των τιμών του δείγματος ως συνάρτηση της μέσης τιμής χ και να βρείτε το ποσοστό των παρατη­ρήσεων του δείγματος που περιέχονται στο διάστημα (0.88 χ ,1.36 χ ).

Λύση α. Για κάθε χ ε R ισχύει f '(x) =

= s · χ 2 - χ · χ - 2χ (1) και αφού η γραφική παράσταση της f δέχεται στο σημείο με τε­τμημένη χ0 = 1 Ο οριζόντια εφαπτομένη έ-χουμε f '(10) = Ο<:::> s · 102 -χ · 1 0- 2χ = Ο<:::> 100s = 12x<=:>� =� οπότε CV = 1 2%. χ 100 Ο συντελεστής μεταβολής είναι μεγαλύτερος από 10% οπότε το δείγμα δεν είναι ομοιογενές.

β. Από το (α) προκύπτει ότι s = 0, 1 2 ·χ και αντικαθιστώντας στην (1) έχουμε f '(x) = 0, 1 2x · x2 - χ · χ - 2χ = . =χ(Ο, 1 2 · χ2 - χ - 2) Άρα, f '(x) = O<=:> x(0, 1 2 · x2 - x - 2) = 0 και επειδή χ > Ο έχουμε 0, 1 2 · χ2 - χ - 2 = 0

' ' ' ' 1 0 ' 5 . απ οπου προκυπτει οτι χ = η χ = - - . 3

f '(x) > O<=:> x(0, 1 2 · x2 - x - 2) > 0 και επει-δή χ > Ο έχουμε Ο, 12 · χ 2 - χ - 2 > Ο

' ' ' ' 5 ' 10 απ οπου προκυπτει οτι χ < - - η χ > . 3

Οι ρίζες και το πρόσημο της f ' φαίνονται στον παρακάτω πίνακα :

χ 5 1 0 "3 f"(x) + - +

f(x) / � / Οπότε η f είναι γνησίως αύξουσα στο (-οο, -� ] και στο [ 10,+ οο) και γνησίως φθί-3

5 νουσα στο [ -- , 1 0] . 3 Η f ' ζ 5 ' ' παρουσια ει στο -3 τοπικο μεγιστο το

f (-�) και στο 1 0 τοπικό ελάχιστο το f( 1 0) . γ. Με δεδομένο ότι η η καμπύλη κατανομής

του δείγματος είναι περίπου κανονική έχου­με ότι το εύρος είναι R :::::: 6 · s = 6 · 0, 1 2χ = 0, 72χ . Επιπλέον λόγω της κανονικότητας της κατανομής • το 68% των παρατηρήσεων βρίσκεται στο

διάστημα (χ -s,x +s) =(χ -0. 12Χ,χ +0. 12χ) = = (0.88χ ' 1 . 12χ) (2)

ΕΥΚΛΕΙΔΗΣ Β ' λθ ' τ.4/62

Page 65: Ευκλειδης Β 60

Μαθηματικά για την Γ Λυκείου

• το 95% των παρατηρήσεων βρίσκεται στο διάστημα (χ -2s,x + 2s) = (χ -0.24χ,χ + 0.24χ) =

= (0.76χ ' 1 .24χ) • το 99,7% των παρατηρήσεων βρίσκεται

στο διάστημα (χ -3s,x + 3s) = (χ -0.36χ,χ + 0.36χ) =

= (0.64χ ' 1 .36χ)

0,64χ 0,76χ Ο,ββχ 1 , 1 2χ 1 ,24χ 1 ,36χ

I r- 6s

%-.J

I ,.._, --- 95% --_...,.ι I '+-•----99,7%.----�·

Στο διάστημα (χ+ s,x + 3s) =

= (1 . 12χ , 1 .36χ) βρίσκεται το 99• 7 - 68 =

2 3 1, 7 =-2

- = 1 5, 85% των παρατηρήσεων.

Άρα συνυπολογίζοντας και τη (2) στο διά­στημα (0.88 χ , 1 .36 χ )= (χ- s, χ + 3s) βρί-σκεται το 68+ 15 ,85 = 83,85% των παρατη­ρήσεων.

4. Ένα χαρακτηριστικό Χ ενός πληθυσμού παίρ­νει τις τιμές 1,2,3,4 με συχνότητες ν 1 , ν 2 , ν 3 , ν 4 αντίστοιχα. Ένα δείγμα μεγέθους

ν έχει μέση τιμή χ = 5 και τυπική απόκλιση

s = .JS . Α ν η συχνότητα ν i της κάθε τιμής

γίνει νί +ν, i = 1,2,3,4, να βρείτε τη νέα μέση

τιμή χ' και τη νέα τυπική απόκλιση s '. Λύση

Ισχύει η σχέση _χ =

_1_· ν�1_+_2_·_ν...::.2_+_3_·_ν�3 _+_4_·_ν.:t.4 ή αλλιώς ν 1 · ν1 + 2 · ν2 + 3 · ν3 + 4 · ν4 = νχ = 5ν (1) όπως

4 επίσης και Σχ;νί = ν (s2 + (χ)2 } = 30ν (2)

ι=! Το καινούργιο δείγμα έχει μέγεθος ν+4ν = 5ν αφού η συχνότητα καθεμιάς από τις τέσσερις παρατηρήσεις μεγαλώνει κατά ν. Έτσι προκύ­πτει ότι:

χ' _ 1(ν1 + ν) + 2(ν2 + ν) + 3(ν3 + ν) + 4(ν4 + ν) , - 5ν η

χ'=

(lν1 + 2ν2 + 3ν3 + 4ν4 ) + (1 + 2 + 3 + 4)ν 5ν

και λόγω της (1) -ι 5ν + 10ν _, 1 5ν χ = <=:> χ = - = 3 (3) 5ν 5ν

Η νέα διακύμανση προκύπτει από τον τύπο :

s '2 = -1 (Σ χ; (νί + ν)J - (χ')2 ή 5ν i=I

s'2 = -1 (Σ (χ;νί + χ;ν)J - (χ')2 ή ισοδύναμα 5ν i=I

s'2 = -1 (Σ χ;νί + Σ χ;νJ - (χ')2 και λόγω 5ν i=I i=I των (2), (3)

ι2 1 [ Σ4 · 2 ] 2 s = - 30ν + ν · χί - 3 ή ισοδύναμα

5ν i=I

ι2 1 [ Σ4 2 ] s = - 30 + χί - 9 και λόγω της (3) 5 i=l

s '2 = ±[30 + (12 + 22 + 32 + 42 )] - 9 <:::>

s'2 = 3 <=:> s' = J3 Σχόλια :

1 ) Στα πλαίσια του σχολικού βιβλίου αντιμε­τωπίσαμε την μεταβολή της μέσης τιμής και της τυπικής απόκλισης στην περίπτrοση που στις παρατηρήσεις ενός δείγματος προστίθε­ται η ίδια σταθερά ή που οι παρατηρήσεις πολλαπλασιάζονται με την ίδια σταθερά. Στην παραπάνω άσκηση φαίνεται πώς επη­ρεάζεται η μέση τιμή και η τυπική απόκλιση στην περίπτωση εκείνη που οι παρατηρήσεις διατηρούνται αμετάβλητες αλλά διαφορο­ποιούνται οι συχνότητές τους.

2) Παρόλο που οι τιμές των παρατηρήσεων στην παραπάνω άσκηση είναι σχετικά λίγες και η μεταβολή στις συχνότητές τους γίνεται με έναν ειδικό τρόπο (δηλαδή όλες αυξάνο­νται τόσο, όσο είναι το μέγεθος του αρχικού δείγματος), εν τούτοις στη διαδικασία επί­λυσης ακολουθείται μία μεθοδολογία που θα μπορούσε να εφαρμοστεί και σε λιγότερο συμβατικές περιπτώσεις.

5. Από τους ψηφοφόρους μιας πόλης, το 45 % είναι άντρες. Το 36% των ανδρών και το 40% των γυναικών ψήφισαν στις τελευταί­ες εκλογές το κόμμα Κ. Εκλέγουμε τυχαία ένα άτομο.

α) Να βρείτε την πιθανότητα των ενδεχομένων: Γ : Ν α είναι γυναίκα. ·

ΕΥΚΛΕΙΔΗΣ Β ' λθ ' τ.4/63

Page 66: Ευκλειδης Β 60

Μαθηματικά για την Γ Λυκείου

Κ : Να ψήφισε το κόμμα Κ.

Ε : Να είναι άντρας ή να ψήφισε το κόμμα Κ. Δ : Ούτε να είναι γυναίκα ούτε να ψήφισε το

κόμμα Κ.

β) Αν από τους ψηφοφόρους της πόλης, οι 259.560 δεν ψήφισαν το κόμμα Κ, να βρείτε το σύνολο των ψηφοφόρων.

Λύση α) Το 45% των ψηφοφόρων είναι άνδρες, άρα το

100 - 45 = 55% είναι γυναίκες. Το ποσοστό των ανδρών που ψήφισαν το κόμ­μα Κ είναι 36% και επί του συνόλου των

' 45 36 1620 ψηφοφορων 100.100

= 10000

= 1 6'2%.

Το ποσοστό των γυναικών που ψήφισαν το κόμμα Κ είναι 40% και επί του συνόλου των

' 55 40 2200 ψηφοφορων - ·- = -- = 22%. 1 00 100 10000

Άρα το κόμμα Κ ψηφίστηκε από το 1 6,2+22=38,2% των ψηφοφόρων. Ισχύει : Ρ(Γ) = 55%. Ρ(Κ) = 38,2%. Α ν Α είναι το ενδεχόμενο να είναι άντρας τότε: Ρ(Ε) = Ρ( Α υ Κ) = Ρ( Α) + Ρ(Κ) - Ρ( Α n Κ) =

45 38, 2 16, 2 67 = - + -- - -- = -100 100 100 100 Ρ(Δ) = Ρ ( (Γ υ Κ)'] = 1 - Ρ(Γ υ Κ)= 1 - Ρ(Γ) - Ρ(Κ) + Ρ(Γ n Κ) = 1 - 55 - 38, 2 + 22 = 28, 8 · = 28 8%. 100 100 100 100 '

β) Αν με Ν(Ω) συμβολίσουμε το πλήθος όλων των ψηφοφόρων, ισχύει Ρ(Κ') = 1 - Ρ( Κ) <=> Ν(Κ') = 1 - 38• 2 <=>

Ν(Ω) 100 259.560 = 61 ,8 <=> Ν(Ω) = 42Ο.ΟΟΟ . Ν(Ω) 100

6. Έστω ο δειγματικός χώρος Ω και δύο ενδε-

χόμενα Α, Β του Ω με Ρ(Α)=Ρ(Β)=.!. . Θεω-2

ρούμε τις παρατηρήσεις : Ρ(Α), Ρ(Β), Ρ( Α n Β ), Ρ(Α υ Β). α) Να υπολογίσετε τη μέση τιμή και τη διάμε­

σό τους. β) Να δείξετε, ότι η διακύμανσή τους είναι:

s2 = _!_ [P(A n B)]2 _ _!_P(A n B) + _!_ 2 2 8 γ) Να δείξετε ότι η πιθανότητα να πραγματο­

ποιηθεί μόνο το ενδεχόμενο Α, είναι ίση με

sJ2 . Λύση

α) Η μέση τιμή των παρατηρήσεων είναι: _ Ρ(Α) + Ρ(Β) + Ρ(Α n B) + Ρ(Α υ Β) Χ = = 4

Ρ( Α) + Ρ(Β) + Ρ( Α n Β) + Ρ( Α) + Ρ(Β) - Ρ( Α n Β)

β)

4 1 1

2Ρ(Α) + 2Ρ(Β) Ρ( Α) + Ρ(Β) 2 + 2 1 = = =-- = -4 2 2 2 Επειδή Α n Β ς;::; Α ς;::; Α υ Β και Α n Β ς;::; Β ς;::; Α υ Β ισχύει ότι Ρ( Α n Β) :=::; Ρ( Α) :=::; Ρ( Α υ Β) και Ρ( Α n Β) :=::; Ρ(Β) :=::; Ρ( Α υ Β) . Άρα από τις τέσσερις παρατηρήσεις Ρ(Α), Ρ(Β), Ρ( Α n Β ), Ρ( Α υ Β), οι δύο μεσαίες είναι οι Ρ(Α), Ρ(Β) και κατά συνέπεια

1 1 - + -δ = Ρ( Α) + Ρ(Β) 1.____1. = .!_ . 2 2 2

2 (Ρ( Α) - χΥ + (Ρ(Β) - χΥ + s = �--���-�� 4

+(P(A n B) - xY + (Ρ(Α υ Β) - χΥ -->.------<------'----� = 4

( 1 1 )2 ( 1 1 )2 ( 1 )2 2 - 2 + 2 - 2 + P(A n B) - 2 +

4

+ (Ρ( Α) + Ρ(Β) - Ρ(Α n Β) -�)2 ---'---------� = 4

P(A n B) - 2) + 2 + 2 - P(A n B)- 2 ( 1 )2 ( 1 1 1 )2 = = 4

(� - P(A n B) Υ + (� - Ρ(Α n B))2 = �---���----�

4

2(� - P(A n B)y (� - P(A n B)y = =

4 2 = .!_(Ρ(Α n Β) ]2 - .!_ Ρ(Α n Β) + .!_

2 2 8 γ) Από την ισότητα

(-!- - Ρ( Α n Β) )2 2 s2= 2 2 <=> (� - P(A n B)) =

2s2 <=> (Ρ( Α) - Ρ( Α n Β) )2 = 2s2 <=> (Ρ( Α - Β) )2 = 2s2 <=> Ρ( Α - Β) = sJ2 .

7. Έστω Ω={1,2,3, . . . ,ν} ένας δειγματικός

ΕΥΚΛΕΙΔΗΣ Β ' λθ ' τ.4/64

Page 67: Ευκλειδης Β 60

Μαθηματικά για την Γ Λυκείου

χώρος που αποτελείται από ισοπίθανα απλά ενδεχόμενα το δείγμα των οποίων έχει μέση τιμή χ = 5 . α) Να βρείτε το ν. β) Θεωρούμε το σύνολο Κ={ λωί + 21/

λ ε Ν* και ω; στοιχείο του Ω}. Να βρεί­

τε την τιμή του λ ώστε το δείγμα τιμών του Κ να είναι ομοιογενές.

γ) Επιλέγουμε ένα στοιχείο ωί του Ω. Να

βρείτε την πιθανότητα το 3 ωί να είναι

και στοιχείο του Κ με δεδομένο ότι το δείγμα τιμών του Κ είναι ομοιογενές

Λύση

α) _ 5 1 + 2 + 3 + . . . + ν 5 χ = � = � ν

ν(ν + 1) = 5ν � ν + 1 = 10 ν = 9 2 Άρα Ω = { 1 ,2,3, . . . ,9 } .

β) Η διακύμανση των στοιχείων του Ω είναι : 2 ( 1 - 5)2 + (2 - 5)2 + (3 - 5)2 + s = ��--�--�--��---

9 +(4 - 5)2 + (5 - 5)2 + (6 - 5)2 + (7 - 5)2 +

9 +(8 - 5)2 + (9 - 5)2

9 2 16+9+4+ 1+0+ 1+4+9+ 16 2 60 �s = � s = --

9 9 . ' κλ 2JlS και η τυπικη τους απο ιση s = --

3-- .

Οι παρατηρήσεις λω; λεΝ και ω; στοιχείο του Ω έχουν μέση τιμή λχ = 5λ και τυπική

. 2λJΙ5 αποκλιση λs = . 3 Οι παρατηρήσεις λω; +2, λεΝ και ω; στοιχείο του Ω έχουν μέση τιμή λχ + 2 1 = 5λ + 2 1 και

. ' κλ λ 2λJΙ5 τυπικη απο ιση s = 3

. Ο συντελεστής μεταβολής του δείγματος τι­μών του Κ είναι :

λs CV = ----λx + 2 1

2λJΙ5 5λ + 2 1

να είναι αυτό ομοιογενές αρκεί : cν s; 10% �

και για

2λJΙ5 s; -1 � 2oλJlS s; 1 5λ + 63 � 3(5λ + 2 1) 10 λ(2οJΙ5 - 1 5) s; 63

λ s; .)!] ::::: 1, 008 και επειδή το λ είναι 20 1 5 - 1 5

θετικός ακέραιος, έχουμε λ = 1 . γ) Ω= { 1 ,2,3, . . . ,9 } και για λ = 1 έχουμε

Κ= { 22,23 ,24, . . . ,30 } . Το σύνολο που αποτε-λείται από τα στοιχεία 3 ω; είναι το Μ={ 3 ,6,9, 1 2, 1 5 , 1 8,2 1 ,24,27,30 } . Οπότε αν Α είναι το ενδεχόμενο το 3 ω; να εί­ναι και στοιχείο του Κ ισχύει Α = { 24,27 } .

Ρ( Α) = Ν(Α) = 3_ . Ν(Ω) 9

8. Κατά τη στατιστική επεξεργασία της βαθ­μολογίας (στην εικοσάβαθμη κλίμακα) των μαθητών της Γ' Λυκείου στις απολυτήριες εξετάσεις στο μάθημα «Μαθηματικά και Στοιχεία Στατιστικής», διαπιστώθηκε ότι μόνο το 0,15 % των μαθητών έγραψε πάνω από 18, ενώ το 2,5 % έγραψε το πολύ 8. Υ­ποθέτουμε ότι η κατανομή της βαθμολογίας είναι κατά προσέγγιση κανονική.

α.Να βρείτε τη διάμεσο, τη μέση βαθμολο­γία και την τυπική απόκλιση της βαθμο­λογίας.

β. Να εξετάσετε, αν το δείγμα είναι ομοιογε­νές.

γ. Εκλέγουμε στην τύχη έναν μαθητή. Να βρείτε την πιθανότητα να έχει γράψει από 10 μέχρι 18.

δ. Αν 31.920 μαθητές έγραψαν μέχρι 14, να βρείτε το πλήθος των μαθητών της Γ Λυ­κείου που έλαβαν μέρος στις απολυτήριες εξετάσεις στο μάθημα «Μαθηματικά και Στοιχεία Στατιστικής».

Λύση α. Λόγω της κανονικότητας της κατανομής

• το 68% των παρατηρήσεων βρίσκεται στο διάστημα (χ - s, χ + s)

• το 95% των παρατηρήσεων βρίσκεται στο διάστημα (χ - 2s, χ + 2s )

• το 99,7% των παρατηρήσεων βρίσκεται στο διάστημα (x -3s,x + 3s )

Άρα λιγότερο από χ -3s και περισσότερο από χ+ 3s αντιστοιχεί στο 100-99,7=0,3% των παρατηρήσεων και κατά συνέπεια πε­ρισσότερο από χ+ 3s αντιστοιχεί στο 0;3 = 0, 1 5% των παρατηρήσεων. Άρα x+ 3s = 1 8 (1)

ΕΥΚΛΕΙΔΗΣ Β ' λθ ' τ.4/65

Page 68: Ευκλειδης Β 60

Μαθηματικά για την Γ Λυκείου

Επίσης λιγότερο από χ -2s και περισσότε­ρο από χ+ 2s αντιστοιχεί στο 100-95=5% των παρατηρήσεων και κατά συνέπεια λιγό-τερο από χ- 2s αντιστοιχεί στο � = 2, 5% 2 των παρατηρήσεων. Άρα χ- 2s =8 (2) Λύνοντας το σύστημα των ( 1 ) και (2) προ­κύπτει ότι s = 2 και χ = 12 . Στην κανονική κατανομή δ = χ οπότε δ=1 2.

β. CV = � = � = .! � 1 6,66% > 10%. Άρα το Ιχl 1 2 6

δείγμα είναι ομοιογενές. γ. Η βαθμολογία από 1 Ο μέχρι 1 8 αντιστοιχεί

στο διάστημα (χ - s, χ + 3s) • το 68% των παρατηρήσεων βρίσκεται στο

διάστημα (χ - s,x + s)

• το 99• 7 - 68 =

3 1• 7 = 15, 85% των παρα-2 2 τηρήσεων βρίσκεται στο διάστημα (x+ s,x + 3s) .

Δηλαδή από 10 μέχρι 1 8 έχει το 68 + 1 5 ,85 = 83,85% των μαθητών. Άρα η πιθανότητα ένας μαθητής να έχει γρά­ψει από 10 μέχρι 1 8 είναι 83,85%.

δ. Μέχρι 14 δηλαδή μέχρι χ+ s έχουν το 68 8 θ ' ' 50+- = 4% των μα ητων που αντιστοιχει 2

σε 3 1 .920 μαθητές. Άρα όλοι οι μαθητές εί-84 ναι: 3 1 .920 : - = 38.000 100

Απαντήσεις Τα Μαθηματικά μας διασκεδάζουν 4ου Τεύχους

100 ευρώ 100 σοκολάτες

Είναι 5 σοκολάτες των 10 Ευρώ, 1 των 3 και 94 των 0,5.

Η Κληρονομιά

Οι 17 καμήλες

Έγινε το μοίρασμα αλλά όχι ανάλογα των αριθμών 2,3,9.

Είχε 50 του έμειναν 51

Δεν έχουν καμία σχέση τα δύο αθροίσματα.

Οι σοκολάτες

Η Αφροδίτη έφαγε τα 8/3 και έδωσε 8 Ευρώ. Άρα όλες οι σοκολάτες κοστολογήθηκαν από την Α­φροδίτη 24 Ευρώ. Δηλαδή 3 Ευρώ η κάθε μια. Έτσι η Σοφία θα πάρει 7 Ευρώ και η Αρετή 1 Ευ­ρώ.

Οι Μαθητές

Είναι μαθητές από το 2° Γυμνάσιο. Αυτό γιατί ο Ερμής από τις απαντήσεις του είναι φανερό ότι λέ­ει Ψέματα άρα είναι μαθητής του 200 Γυμνασίου. Με την δεύτερη απάντησή του επίσης λέει ψέματα άρα και η Αθηνά είναι του 2ου. Η Κόρη

Το όνομά της είναι ΑΝΝΑ. Διότι πάντα προκύπτει ο αριθμός 1089. Από τη Γη στη Σελήνη

Δεν διπλώνει περισσότερο από 7 φορές το σεντόνι διότι αποκτά γρήγορα μεγάλο πάχος. Α ν το πάχος του είναι 1 χιλιοστό στο 38 δίπλωμα το πάχος του θα είναι όσο η απόσταση Γης -Σελήνης. (Γεωμε­τρική πρόοδος) .

Η Πίτα

Αυτά έλεγαν οι σοφιστές.τον 6° π.Χ. αιώνα. Μετά από μερικά μοιράσματα η πίτα θα είναι ένα αόρα­το ψίχουλο.

ΕΥΚΛΕΙΔΗΣ Β ' λθ' τ.4/66

Page 69: Ευκλειδης Β 60

Το Βήμα του Ευκλεiδη

ο I I

Η στήλη αυτή έχει ως στόχο την ανάπτυξη μαθηματικού διαλόγου. Φιλοδοξούμε να συμμετάσχουν όλοι

όσοι έχουν ένα γενικώτερο ενδιαφέρον για τα Μαθηματικά.

Ι. Στο τεύχος 53 του περιοδικού μας (Ιούλιος -Αύγουστος - Σεπτέμβριος 2004) στο κομμάτι που αναφέρεται στη Α! τάξη του Λυκείου είχαμε δη­μοσιεύσει ένα άρθρο με τίτλο: «Για να κατανοή­σουμε καλύτερα τους αριθμούς» (Γιάννης Στρα­τής) . Ήταν μια σειρά θέματα πάνω στις ταυτότη­τες, τις εξισώσεις α ! βαθμού, τις μόνιμες ανισότη­τες (και με περιορισμό), τις ανισώσεις α ! βαθμού. Το άρθρο συμπληρωνόταν με μερικά προβλήματα (συνολικά 5 · δύο στις εξισώσεις, τρία στις ανισό­τητες) .

Στη σελίδα 1 9 είχαμε δημοσιεύσει το παρα­κάτω ζήτημα:

Αφού αποδειχθεί η ταυτότητα: ( α2 + β2 + γ2 )( χ2 + y2 + z2 ) _ (αχ + βy + γz )2 =

= (βχ - αy )2 + ( γy - βz )2 + ( γχ - αz )2 ,

να βρεθεί στο εσωτερικό τριγώνου ΑΒΓ σημείο Ρ έτσι ώστε: «αν χ, y, z είναι οι αποστάσεις του Ρ απ' τις πλευρές του τριγώνου και χ + y + z = c

όπου c κάποιος θετικός αριθμός σταθερός ( κατάλ­ληλος) το άθροισμα χ2 + y2 + z2 να είναι το μι-

κρότερο δυνατό» (το 'χε προτείνει ο συνάδελφος Θανάσης Κυριακόπουλος, μέλος της Συντακτικής Επιτροπής) .

Πρέπει να παραδεχθούμε κατ' αρχάς ότι η λέξη «κατάλληλος» δεν υπήρχε στο κείμενο που είχαμε δημοσιεύσει. Απ' την όλη όμως διαπραγμάτευση του ζητήματος γινόταν φανερό ότι ο c είναι μεν σταθε­ρός, αλλά εννοείται ότι είναι προσδωριστέος.

Ο συνάδελφος Παναγιώτης Οικονομάκος μας απέστειλε με γράμμα του (Γενάρης 2005) την πα­ρακάτω παρατήρηση :

Επιμέλεια:Γιάννης Στρατής - Βαγγέλης Ευσταθίου

«Το πρόβλημα 3 της σελίδας 1 9 (περιόδου Ιουλ. - Αυγ. - Σεπτ. 2004) έχει λανθασμένη υπό­θεση, διότι αν Ρ μεταβλητό σημείο στο εσωτερικό τυχαίου τριγώνου ΑΒΓ και χ, y, z είναι οι αποστά­σεις του από τις πλευρές του, τότε το άθροισμα χ + y + z δεν είναι σταθερό (Υπογράμμιση δική μας) .

Παρακάτω δείχνουμε με δύο τρόπους ότι η πα­ρατήρηση του συναδέλφου δεν ισχύει. Ακριβέστε­ρα υπάρχουν άπειρα σημεία Ρ στο εσωτερικό ενός σκαληνού τριγώνου, που το άθροισμα: χ + y + z των aποστάσεών του απ' τις αντίστοιχες πλευρές του τριγώνου είναι ένας σταθερός αριθμός c, ο οποίος διατρέχει ένα συγκεκριμένο διάστημα.

1 ος τρόπος (Σωτήρης Ε. Λουρίδας)

ΓΕΩΜΕΤΡΙΚΉ ΑΝΤΙΜΕΤΩΠΙΣΗ ΤΟΥ ΘΕ­

ΜΑΤΟΣ:

«Να προσδιοριστεί στο εσωτερικό τριγώνου Δ

ΑΒΓ σημείο Ρ ώστε αν χ, y, z οι αποστάσεις του Ρ από τις πλευρές ΒΓ = α , ΑΓ = β , ΑΒ = γ

αντίστοιχα με χ + y + z = c όπου c κατάλληλη

θετική σταθερή το άθροισμα w = χ2 + y2 + z2

να είναι το ελάχιστο δυνατό.

ΛΥΣΗ

1. Αν το τρίγωνο είναι ισόπλευρο με ΑΒ=ΒΓ=ΓΑ=α , , α.J3 τοτε υποχρεωτικ:α c = -- = υ .

2 Δ

2. Έστω ότι το τρίγωνο ΑΒΓ είναι ισοσκελές με ΑΒ = ΑΓ * ΒΓ ( σχ. 1 ) . Θεωρούμε ΑΒ = ΑΓ = β , ΒΓ = α . Έχουμε το σύστημα

ΕΥΚΛΕΙΔΗΣ Β' λθ' τ.4/67

Page 70: Ευκλειδης Β 60

Το Βήμα του Ευκλείδη

{αχ + βy + βz = 2Ε ή x + y + z = c {αχ + β ( c - χ ) = 2Ε y + z = c - x

(1) (2)

Από τη σχέση ( 1 ) του συστήματος προκύπτει , 2Ε - βc (3) οτι χ = .

α - β , 2Ε - βc 2Ε Θα πρεπει Ο < < υ α = - . α - β α

β , Ο 2Ε - βc 2Ε , 2Ε Α ν α > τοτε < < - η c > - και α - β α α

επειδή α - β > Ο <:::> Ο < 2Ε - βc ή c < 2: , οπό-2Ε 2Ε τε - < c < - . α β

2Ε - βc 2Ε Α ν α < β τότε: < - και επειδή α - β α

α - β < 0 <:::> α (2Ε - βc) > 2Ε ( α - β) ή

2αΕ - αβc > 2αΕ - 2βΕ ή c < 2Ε και βέβαια α

, 2Ε 2Ε 2Ε 2Ε - βc < 0 η c > - με - > -

, 2Ε 2Ε αρα: - < c < - . β α

Β

β α β

Α

Σχ. Ι Γ

(ε)

2Ε - βc Σε κάθε περίπτωση ( α > β ή α < β ) η ----'--α - β αντιπροσωπεύεται από μία θετική σταθερή κ < υ α . Άρα η απόσταση χ είναι σταθερή χ = κ . Αυτό ση­μαίνει ότι το σημείο Ρ ανήκει σε σταθερή ευθεία (ε) που απέχει από την πλευρά ΒΓ απόσταση κ και τέμνει τις ΑΒ, ΑΓ στα σημεία Βι , Γι αντίστοιχα. Αυτό σημαίνει ότι: w = κ2 + y2 + z2 • Άρα αρκεί το άθροισμα Wι = y2 + z2 να είναι το ελάχιστο

δυνατό. Αν ονομάσουμε υ = Γι Γ; όπου Γι Γ; .1. ΑΒι γνωρίζουμε ότι: y + z = υ (Βασική πρόταση για το

Δ Δ ισοσκελές τρίγωνο ΑΒιΓι ) . Το ΑΒιΓι είναι στα-θερό τρίγωνο άρα και το ύψος του υ = Γι Γι' είναι σταθερό. Επομένως το w 1 = y2 + z2 με y + z = υ

(υ σταθερή) γίνεται ελάχιστο όταν y = z = � . Στην 2

περίπτωση αυτή το Ρ συμπίπτει με το μέσο του ευθύγραμμου τμήματος Βι Γι . Προφανώς η ευθεία (ε) δεν υποχρεούται να διέρχεται από το έκκεντρο

Δ του τριγώνου ΑΒΓ ή το σημείο τομής των συμμε-τροδιαμέσων του ίδιου τριγώνου (σημείο του Lemoine).

3. ΕΝΑΣ ΒΑΣΙΚΟΣ ΓΕΩΜΕΤΡΙΚΟΣ

ΤΟΠ ΟΣ -Έστω κυρτή γωνία xOy . Θεωρούμε ότι στο

εσωτερικό της κινείται ένα σημείο Ρ, ώστε κ · z + λ · y = t όπου z = ΡΡι απόσταση του Ρ από την Οχ, y = ΡΡ2 απόσταση του Ρ θα κινείται σε σταθερή ευθεία που τέμνει τις Οχ, Oy (σχ. 2). Πράγματι:

ο

χ

Σχ. 2

λ λ t ' κ · z + · y = t <=:> z +- · y = - οποτε αν κ κ

' λ . t z = ΡΥ = -y <:::> z + z = - ( 1 ) κ κ

Ε ' λ κ t ' πισης: · y + κ · z = t <:::> y + - · z = - οποτε αν λ λ

' κ ' t Υ = pτ = -z <=:> y + y = -λ λ (2)

ΕΥΚΛΕΙΔΗΣ Β ' λθ ' τ.4/68

Page 71: Ευκλειδης Β 60

Το Βήμα του Ευκλείδη

Άρα ( 1 ) <=> Ρι γ = λλ = d2 = _!_ θετική σταθε­κ

ρή, ( 2) <=> p2 γ = κκ' = dι = _!_ θετική σταθερή . λ Επομένως το Τ θα κινείται σε σταθερή ευθεία η ι παράλληλη στην Oy που απέχει από αυτή απόστα­ση dι ενώ το σημείο γ θα κινείται σε σταθερή ευθεία n2 παράλληλη στην Οχ που απέχει από αυτή απόσταση d2 .

'Ε , Υ κ Υ , Ρ θ , χουμε οτι - = - = � αρα το α κινειται z λ z

στην ευθεία ΚΑ. -Μάλιστα ΡΚΤ = ΡΡι Τ και Ρ Λ γ = ΡΡ2 γ .

4. Θα έρθουμε τώρα στο πρόβλημά μας σε Δ

τρίγωνο ΑΒΓ με α > β > γ (α = ΒΓ, β = ΑΓ, γ = ΑΒ) . Από τη σχέση α > β > γ

έχουμε A > B > r και υγ >1>ιJ >υ0 ( υ0 = 2: ,

2Ε 2Ε ) 'Ε , υ β = β , υ γ = Υ . χουμε το συστημα {αχ + βχ + γz = 2Ε χ + y + z = c από όπου προκύπτει x, y, z > Ο

(α - β)y + (α - γ ) z = αc - 2Ε .

Προφανώς αc - 2Ε > Ο ή c > 2Ε . α

Ας πάμε στο σχ.3 . Από το προηγούμενο λήμμα είναι κατανοητό ότι το Ρ θα κινείται σε σταθερή ευθεία (ε) που τέμνει τις Αχ, Ay στα σημεία Κ, Λ αντίστοιχα,

αc - 2Ε αc - 2Ε , με dι = και d2 = . Άμεσα α - β α - γ έχουμε ότι:

d d αc - 2Ε αc - 2Ε ι > 2 <=> > <=> α - β α - γ

�-�-�+�>�-�-·+� <=> <=> (β - γ) ( αc - 2Ε) > Ο

που ισχύει. Από εδώ προκύπτει ότι ΑΛ > ΑΚ .

Α

χ Υ

Σχ. 3 Επειδή ταυτόχρονα έχουμε ότι ΑΓ > ΑΒ όταν

το σημείο Κ είναι μεταξύ των Α, Β το Λ θα είναι μεταξύ των Α, Γ.

Για να είναι το σημείο Κ πλευράς ΑΒ θα πρέπει

μη εξωτερικό της 2Ε dι � υβ = β ή

αc - 2Ε 2Ε , 2Ε Α --- � - η c � - = υβ . ν α - β β β

c = υβ = -2Ε <=> ( α - β ) χ = (β - γ) z <=> χ = -β -_γ · z . β α - β

Αυτό μας λέει ότι το Ρ εκτός της ευθείας (ε) θα κινείται επίσης σε σταθερή ευθεία διερχόμενη από την κορυφή Β.

Από dι = υβ έχουμε κ = Β <=> Ρ = Β . Η περί-πτωση αυτή δε μας κάνει. Τελικά δεχόμαστε ότι

2Ε Δ λ δ , 2Ε 2Ε Τ , ΑΒ c < - . η α η - < c < - οτε κ ε με β α β κ ::F- Α και βέβαια Λ ε ΑΓ με Λ '* Α και Λ '* Γ . Έστω Χ ι = ΚΚι η απόσταση του Κ από την ΒΓ, χ2 = ΛΛι η απόσταση του Λ από την ΒΓ και χ = pp' η απόσταση του Ρ από την ΒΓ. Είναι κα­θαρό ότι αχ ι + βdι = 2Ε και αχ2 + βd2 = 2Ε ή

2Ε - βc 2Ε - γc Χ ι = και χ2 = . α - β α - γ

Προφανώς Χ ι < χ2 (απλές ισοδυναμίες) . Θεω­ρούμε ΚΡ = μ , ΡΛ = ν και ΚΑ = u όπου η θετική σταθερή, στην τυχαία θέση του Ρ εντός του ευθύ­γραμμου τμήματος ΚΑ άρα και εντός του τριγώ­νου ΑΒΓ.

π , , z d2 αc - 2Ε αρατηρουμε οτι: - = -. = tι = -----,---,-μ u u (α - γ) οπό-

ΕΥΚΛΕΙΔΗΣ Β' λθ ' τ.4/69

Page 72: Ευκλειδης Β 60

Το Βήμα του Ευκλείδη

Επίσης Υ

d1 αc - 2Ε - = - = t2 = <=:> Υ = t2 . ν . ν u u (α - β) Από το τραπέζιο

u · x 2 - (χ 2 - χ1 )ν χ = . u

Έτσι παίρνουμε W = X2

+ y2+ z2 = . . . =

έχουμε:

[(χ2 - χ1 )2 + υ( t� + t�)} v2

-2u[ χ2 ( χ2 -x1 )+ u2t�}+ u4 · t�

u2

Το W γίνεται ελάχιστο όταν u [ χ2 (χ2 - x1 ) + u2t� J

ν = που είναι θετική (χ2 - χι )2 + u ( t� + tO ποσότητα.

Για την τιμή αυτή έχουμε

_ - ( _ ) χ2 ( χ2 - x1 ) + u 2t� χ - χ2 χ2 Χ ι 2 . ( χ2 - χ1 ) + u 2t� + u 2t�

Προφανώς χ < χ2 . Για να είναι χ > χ αρκεί

( ) χ2 ( χ2 - χ 1 ) + u 2t� , Χ 2 - Χι > Χ 2 - Χ ι 2 η (χ2 - χ1 ) + u 2t� + u 2t�

( )2 2 2 2 2 ( ) 2 2 χ2 - χ1 + u t1 + u t2 > χ2 χ2 - χ1 + u t1 <=:> 2 2 2 <=:> u t2 > χ2χ1 - χ 1

' 2 d� 2 ' d2 2 ' η u 2 + χ1 > χ2χ 1 η 1 + χ1 > χ1χ2 που ει-κ

ναι αληθής αν d1 � χ-2 (βασική ανισότητα).

'Εστω d1 < χ2 τότε αc - 2Ε < 2Ε - γc ή α - β α - γ

α2c - αγc - 2αΕ + 2γΕ > 3αΕ - αγc - 2βΕ + βγc ή α2c - βγc > 4αΕ - 2βΕ - 2γΕ ή

(α 2 - βγ) c > 2Ε ( 2α - β - γ) ή c > 2Ε ( 2: - β - γ) α - βγ

2Ε 2Ε (2α - β - γ) 2 με - > 2 ( <=:> . . . <=:> (α - β) > 0 β α - βγ που είναι αληθής αφού α > β ) .

Ε , 2Ε 2Ε (2α - β - γ) 2Ε καθότι πομενως: - > c > 2 > -β α - βγ α , , 2Ε (2α - β - γ) 2Ε , , ευκολα εχουμε 2 > - . Θεωρουμε ηδη

α - βγ α

d� + χ� > χ2χ1 <=:> d� > χ1 - ( χ2 - χ 1 ) ή

( αc - 2Ε)2 (2Ε - βc ) (β - γ) ( αc - 2Ε) 2 > 2 <=:> . . . <=:>

(α - β) (α - β) · (α - γ)

2Ε ( α + β - 2γ) <=:> c > 2 2 · α + β - αγ - βγ , , . 2Ε 2Ε (α + β - 2γ) Παρατηρουμε οτι. - > 2 2 . β α + β - αγ - βγ

Επίσης παρατηρούμε ότι: 2Ε (2α - β - γ) 2Ε ( α + β - 2γ) --'--::,......--� > <=:>

α2 - βγ α2 + β2 - αγ - βγ

<=:> Ο < α2 + β2 + γ2 - αβ - αγ - βγ

(που είναι αληθής). 2Ε 2Ε (2α - β - γ) Τελικά όταν - > c > 2 έχουμε β α - βγ

d� + χ� > χ2χ1 που οδηγεί στην χ > χ 1 • Άρα χ1 < χ < χ2 που σημαίνει ότι το σημείο Ρ με

, , u [ χ2 ( χ2 - x 1 ) + u2t� J ΡΡ = χ οταν ν =

2 είναι (χ2 - χ1 ) + u 2t� + u 2t�

εσωτερικό σημείο του ευθύγραμμου τμήματος ΚΑ άρα και του τριγώνου ΑΒΓ , ώστε χ2 + y2 + z2 να είναι το ελάχιστο δυνατό.

Υπενθύμιση : Δ

Έστω τρίγωνο ΑΒΓ με ΒΓ = α , ΑΓ = β , ΑΒ = γ και θετικοί αριθμοί χ, y, z ώστε

Δ αχ + βy + γz = 2Ε . Τότε εντός του τριγώνου ΑΒΓ υπάρχει σημείο Ρ που απέχει από τις πλευρές α, β, γ αποστάσεις χ, y, z αντίστοιχα.

Ανοιχτό πρόβλημα:

Να βρεθεί στο εσωτερικό τριγώνου ΑΒΓ ση­μείο Ρ με αποστάσεις από τις α, β, γ τις χ, y, z α­ντίστοιχα έτσι ώστε κ · χ + λ · y + μ · z = c όπου κ, λ, μ, c κατάλληλες θετικές σταθερές ώστε το ά­θροισμα χ 2 + y2 + z2 είναι το ελάχιστο δυνατό.

Σημείωση :

Στο επόμενο τεύχος θα δημοσιεύσουμε την αντιμετώπιση του ίδιου θέματος από τον συνά­δελφο Γιάννη Στρατή.

ΕΥΚΛΕΙΔΗΣ Β ' λθ ' τ.4ΠΟ

Page 73: Ευκλειδης Β 60

υκλεiδης .. προτεινει ιιι ιιι ιιι Ευκλείδη

και . .. . Δι6φαντο Επιμέλεια: Α. Κυριακόπουλος, Γ. Στρατής, Γ. Τριάντος, Ν. Αντωνόπουλος

ΑΣΚΗΣΕΙΣ ΓΙΑ Λ ΥΣΗ

70. Δίνεται τετράπλευρο ΑΒΓ Δ. Να αποδείξετε

ότι αν το ευθύγραμμο τμήμα που ενώνει τα

μέσα δύο απέναντι πλευρών του διέρχεται

από το σημείο τομής των διαγωνίων του,

τότε το ΑΒΓ Δ είναι τραπέζιο.

( Επροτάθη από το συνάδελφο Βασίλη Καραχά­

λιο, Τρίπολη)

Λύση : (Από το συνάδελφο Γιάννη Σταματογιάννη,

Δροσιά Απικής)

Δ z Γ

Έστω ότι το τετράπλευρο δεν είναι τραπέζιο και ΑΕ'Ρ //ΓΔ . Τότε έχουμε: ΑΕ'

= ΑΟ = ΟΕ' ( 1 ) και Ε

'Ρ = ΟΕ' (2) . ΖΓ ΟΓ ΟΖ ΔΖ ΟΖ

Α , ( 1 ) (2) , , ΑΕ' Ε'Ρ πο τις , προκυπτει οτι: -- = - και ΖΓ ΔΖ. εξαιτίας της υπόθεσης ΔΖ = ΖΓ , παίρνουμε ΑΕ' = Ε'Ρ . Από την τελευταία ισότητα και την ΑΕ = ΕΒ , συνεπάγεται ότι ΕΕ' //ΒΡ , το οποίο είναι άτοπο. Άρα το τετράπλευρο ΑΒΓ Δ είναι τραπέζιο.

Λύσεις έσrειλαν επίσης: Ο καθηγητής Θόδωρος

Μπόλης, οι μαθητές Νάκος Βασίλης - 3ο Ε. Λ. Η­

ρακλείου Απικής και Μαραβίτσας Νίκος - 6ο Ε. Λ.

Περισrερίου, οι συνάδελφοι Ηλίας Κων/νος - Αλι­

βέρι, Τσαπακίδης Γιώργος - Αγρίνιο, Κούρτης Χρυ­

σόσrομος - Λάρισα, Καραβότας Δημ. - Κ. Αχαία,

Αποσrολόπουλος Γιώργος - Μεσολόγγι, Πετρολέ-

κας Στέλιος - Κορυδαλλός, Αθήνα, ο κύριος Μπό­

ρης Ροδόλφος - Δάφνη Απικής και ο κύριος Αν­

δρής Ιωάννης, μηχανικός - Αθήνα.

7 1 . Στις πλευρές ΑΒ και Γ Δ τετραπλεύρου

ΑΒΓ Δ θεωρούμε σημεία Ε, Ζ ώστε να ισχύ­

ουν: ΕΒ = λ · ΑΕ και ΔΖ = λ · ΖΓ , λ ε JR . Να

αποδείξετε ότι αν η ΕΖ και οι διαγώνιες συ­

ντρέχουν τότε το τετράπλευρο ΑΒΓ Δ είναι

τραπέζιο.

( Επροτάθη από το συνάδελφο Βασίλη Καραχά­

λιο, Τρίπολη)

Λύση : (Από το συνάδελφο Γιάννη Σταματογιάννη,

Δροσιά Απικής)

Γ

Έστω ότι το τετράπλευρο δεν είναι τραπέζιο και ΑΕ'Ρ /I ΔΓ . Τότε έχουμε: ΑΕ'

= Ε'Ο = ΑΟ

( 1 ) και Ε'Ρ = ΟΡ = ΟΕ' (2) .

ΖΓ ΟΖ ΟΓ ΔΖ ΟΔ ΟΖ Από τις ( 1 ), (2) προκύπτει ότι:

ΑΕ' = Ε

'Ρ => ΑΕ' = _!!__ => Ε'Ρ = λ · ΑΕ' (3) ΖΓ ΔΖ ΖΓ λ · ΖΓ

Ισχύει: ΕΒ = λ · ΑΕ <::> ΕΒ = λ (4). Από τις (3), ΑΕ

, ΕΒ Ε'Ρ (4) παιρνουμε: - = - = λ <::> ΕΕ' // ΡΒ , το ο­ΑΕ ΑΕ' ποίο είναι άτοπο. Άρα το τετράπλευρο ΑΒΓ Δ είναι τραπέζιο.

Λύσεις έσrειλαν επίσης: Ο καθηγητής Θόδωρος

Μπόλης, οι μαθητές Νάκος Βασίλης - 3ο Ε. Λ. Η-

ΕΥΚΛΕΙΔΗΣ Β' λθ' τ.4Πl

Page 74: Ευκλειδης Β 60

Ο Ευκλείδης προτείνει. .. Ευκλείδη ... και Διόφαντο

ρακλείου Αττικής και Μαραβίτσας Νίκος - 6ο Ε. Λ.

Περιστερίου, οι συνάδελφοι Ηλίας Κων/νος - Αλι­

βέρι, Τσαπακίδης Γιώργος - Αγρίνιο, Κούρτης Χρυ­

σόστομος - Λάρισα, Αποστολόπουλος Γιώργος -

Μεσολόγγι, Πετρολέκας Στέλιος - Κορυδαλλος,

Α θήνα, ο κύριος Μπόρης Ροδόλφος - Δάφνη Αττι­

κής και ο κύριος Ανδρής Ιωάννης, μηχανικός - Α­

θήνα.

73. Να βρείτε τις ακέραιες λύσεις της εξίσωσης: (x2 - y2 )4 + 8 (x2 - y2 )3 + 28 (x2 - y2 )2 + +48 (χ2 - y2 ) + 1 1 3 = z2

(Επροτάθη από τον χημικό Δ ημήτριο Καρβελά,

Πεύκη)

Λύση : (Από τον καθηγητή Θόδωρο Μπόλη)

Μπορούμε να υποθέσουμε ότι τα χ, y, z είναι μη αρνητικά και στις λύσεις που θα βρούμε να α­φήσουμε ελευθέρια προσήμου. Η εξίσωση γράφε­ται ισοδύναμα: (χ2 - y2 )4 + 4 (χ2 - y2y · 2 + 6 (χ2 - y2 ) 2 .

·22 + 4 ( χ2 - y2 ) · 23 + 24 + 4 ( χ2 - y2 )2 + + 16 (χ2 - y2 ) + 16 + 4 + 77 = z2

( χ2 - y2 + 2 )4 + 4 ( χ2 - / + 2 )2 + 4 + 77 = z2

z2 - [ (χ 2 - y2 + 2 γ + 22 τ = 77

[ z - (χ 2 - y2 + 2 )2 - 2 J [z + (x2 - y2 + 2)2 + 2] = 77

οπότε ή z + (x2 - y2 + 2)2 + 2 = 77 και

z - (x2 - y2 + 2)2 - 2 = 1

ή z + (x2 - y2 + 2)2 + 2 = 1 1 και

z - (x2 - y2 + 2)2 - 2 = 7

Εύκολα πλέον βρίσκουμε ότι στην πρώτη περί­πτωση έχουμε τις λύσεις z = 39 , χ = 2 , y = Ο και z = 39 , χ = 1 , y = 3 ενώ στην δεύτερη περίπτωση, δεν υπάρχουν λύσεις. Συνολικά, αφήνοντας ελευ­θερία προσήμου, υπάρχουν 12 λύσεις εκ των ο­ποίων 2 είναι μη αρνητικές και μόνο μία θετική .

Λύσεις έστειλαν επίσης οι συνάδελφοι Καρα­

γκιόζης Ν. - Σέρρες, Καραβότας Δημ. Κάτω Αχαία

και Τσαπακίδης Γιώργος - Αγρίνιο.

76. Αν για τους θετικούς ακέραιους αριθμούς χ, y ισχύει: χ 2 + y2 - χ = πολ · 2xy , να αποδεί-

ξετε ότι ο χ είναι τετράγωνο ακεραίου.

(Επροτάθη από τον συνάδελφο Α ντώνη /ωαννί­

δη, Λάρισα)

lη Λύση : (από τον ίδιο) Είναι:

χ 2 + y2 - χ = πολ · 2xy <::::> χ 2 + y2 - χ = 2κχy , κ ε Ζ

<::::> χ 2 _ χ = 2κχy _ y2 <::::> y2 _ 2κχy = χ _ χ 2 <=> (y - κχ )2 = Κ2Χ 2 + Χ - Χ2 <::::>

(κχ - yγ = χ [(κ2 - 1)χ + 1] ( 1 )

Θα αποδείξουμε ότι (χ, ( κ2 - 1 ) χ + 1) = 1 .

Πράγματι , έστω (χ , ( κ2 - 1 ) χ + 1} = δ . Τότε δ/χ

και δ ! (κ2 - 1) χ + 1 , οπότε δ ! χ (κ2 - 1) και δ Ι ( κ2 - 1) χ + Ι . Άρα δ/Ι , πράγμα που σημαίνει ότι

δ = l , οπότε (χ , (κ2 - l) x + l } = I (2). Από την ισό­

τητα (2) έχουμε: χ ( χ , ( κ2 - 1 ) χ + 1} = χ =>

( χ2 , χ [(κ2 - l) x + t]) = χ� (χ2 , (κχ - y)2 ) = χ

οπότε (*) χ = (χ, κχ - y )2 , δηλαδή ο αριθμός χ είναι τετράγωνο ακεραίου.

(*) Με τη βοήθεια της κανονικής μορφής των ακεραίων ή και διαφορετικά, μπορούμε να αποδεί­ξουμε ότι αν α, β ε Ζ με α :;t Ο ή β :;t Ο , τότε (α2 , β2 ) = (α, β)2

2η Λύση : (Από τον καθηγητή Θόδωρο Μπόλη)

Η εξίσωση χ 2 + y2 - χ = 2xyz είναι ισοδύναμη με την (2χγ + (2y)2 = 4x (1 + 2yz) η οποία γρά­φεται (2x - l - 2yz)2 + (2y)2 = (1 + 2yz)2 δηλαδή οι αριθμοί 2χ - 1 - 2yz , 2y και 1 + 2yz σχηματί­ζουν Πυθαγόρεια τριάδα. Από τον Ευκλείδη ξέ-

ΕΥΚΛΕΙΔΗΣ Β' λθ ' τ.4Π2

Page 75: Ευκλειδης Β 60

Ο Ευκλείδης προτείνει ... Ευκλείδη •.. και Διόφαντο

ρουμε ότι: 2χ - 1 - 2yz = t ( r2 - s2 ) , y = trs , t, r, s ε Ν *, (r, s) = 1 ,

r + s Ξ 1mod .2 απ' όπου συνάγεται ότι t = 1 , χ = r2 , y = rs και 1 + 2rsz = r2 + s2 , οπότε ο αριθ-μός χ είναι τετράγωνο ακεραίου. Κατόπιν ο καθη-γητής αναφέρει ότι από την εξίσωση 1 + 2rsz = r2 + s2 η οποία γράφεται ( s - rz γ - ( z2 - 1 ) r2 = 1 και είναι τύπου Pell-

Fermat προσδιορίζονται τα r, s. Λύση έστειλε επίσης ο κύριος Μπόρης Ροδόλ­

φος - Δάφνη Αττικής.

79. Δίνεται κυρτό τετράπλευρο ΑΒΓΔ εγγε­

γραμμένο σε κύκλο με ακτίνα 1. Αν ισχύει:

ΑΒ · ΒΓ · Γ Δ · ΔΑ � 4 , να αποδείξετε ότι το

τετράπλευρο είναι τετράγωνο.

(Επροτάθη από το συνάδελφο Σωτήρη Σκοτί­

δα, Καρδίτσα)

Λύση : (Από τον μηχανικό κύριο Α νδρή Ιωάννη ­

Αθήνα)

Αν θέσουμε ΑΒ · ΓΔ = κ και ΒΓ · ΔΑ = λ , τότε κ, λ > Ο και από το θεώρημα του Πτολεμαίου παίρνουμε: κ + λ = ΑΓ · ΒΔ � 2 · 2 διότι κάθε χορ­δή του κύκλου είναι το πολύ ίση με τη διάμετρό του.

Άρα κ + λ � 4 ( 1 ) . Η δοσμένη σχέση γράφεται (2 ) κλ � 4 (2) . Επίσης ισχύει κ + λ � 2.JιJ: � 2 · 2 , δη-

λαδή κ + λ � 4 (3) . Από τις ( 1 ) και (3) προκύπτει ότι κ + λ = 4 .

Τότε όμως: 4 � 2.JιJ: � .JιJ: < 2 � κλ � 4 η οποία σε συνδυασμό με την (2) δίνει κλ = 4 . Άρα κ + λ = κλ = 4 , οπότε κ = λ = 2 . Με κ = λ = 2 έχουμε ΑΓ · ΒΔ = 4 � ΑΓ = ΒΔ = 2 , οπότε οι ΑΓ, ΒΔ είναι διαγώνιες του κύκλου, επομένως το τε­τράπλευρο ΑΒΓ Δ είναι ορθογώνιο. Εύκολα πλέον από τις ισότητες κ = 2 , λ = 2 βρίσκουμε ότι κα-

θεμιά από τις πλευρές του ορθογωνίου είναι ίση με J2 , οπότε το τετράπλευρο είναι τετράγωνο.

Λύσεις έστειλαν επίσης: Ο καθηγητής Θόδωρος

Μπόλης, ο μαθητής Νάκος Βασίλης - 3ο Ε. Λ. Η­

ρακλείου Αττικής οι συνάδελφοι Κούρτης Χρυσό­

στομος - Λάρισα, Αποστολόπουλος Γιώργος -

Μεσολόγγι, Τσαπακίδης Γιώργος Αγρίνιο,

Πετρολέκας Στέλιος - Κορυδαλλος, Αθήνα και ο

κύριος Μπόρης Ροδόλφος - Δάφνη Αττικής.

ΑΣΚΗΣΕΙΣ ΓΙΑ Λ ΥΣΗ

80. Να αποδείξετε ότι για κάθε δοθέντα ακέραιο αριθμό, α η διοφαvτική εξίσωση (α2 - 1 ) (β2 - 1) = γ2 - 1 έχει τουλάχιστον δύο διακεκριμένες ακέραιες λύσεις ως προς β και γ.

(Προτείνεται από τον Ακαδημαϊκό κύριο Νι­

κόλαο Αρτεμιάδη)

81. Δίνεται το εμβαδόν Ε ενός τριγώνου καθώς και το μέγεθος φ μιας εκ των γωνιών του.

82.

Να βρείτε τα μήκη των πλευρών α και β ώ­στε το μήκος της πλευράς γ που βρίσκεται απέναντι από τη γωνία φ να είναι το ελάχι­στο δυνατό.

(Προτείνεται από τον ΑκαδημαϊΚό κύριο Νι­

κόλαο Αρτεμιάδη ).

Να αποδείξετε ότι το Ρ ( χ ) = 2χ8 + 3χ 6-5χ3 - 4χ + 6 πραγματικές ρίζες.

πολυώνυμο δεν έχει

(Προτείνεται από τον καθηγητή Θόδωρο

Μπόλη).

83. Να λύσετε την εξίσωση

.J2 (x3 +� + 5χ +-ξ-) = 25 + 5χ2 + 2_ + χ χ χ χ

(Προτείνεται από τον καθηγητή Θόδωρο

Μπόλη).

84. Θεωρούμε ένα τρίγωνο ΑΒΓ και στις προε­κτάσεις των πλευρών ΒΓ (προς το Γ), ΓΑ (προς το Α) και ΑΒ (προς το Β) παίρνουμε

ΕΥΚΛΕΙΔΗΣ Β ' λθ ' τ.4Π3

Page 76: Ευκλειδης Β 60

Ο Ευκλείδης προτείνει ... Ευκλείδη ..• και Διόφαντο

αντιστοίχως τα σημεία Δ, Ε και Ζ έτσι, ώ­στε: ΓΔ = ΑΕ = ΒΖ . Να αποδείξετε ότι, αν το τρίγωνο ΔΕΖ είναι ισόπλευρο, τότε και το τρίγωνο ΑΒΓ είναι ισόπλευρο.

(Προτείνεται από τον συνάδελφο Κυριακό­

πουλο Αντώνη).

85. Να βρείτε την ελάχιστη τιμή της παράστα-

σης Α = (� +� +�) ( α + 1) (β + 1) ( γ + 1) α β γ

για τις διάφορες θετικές τιμές των αριθμών α, β, γ.

(Προτείνεται από το συνάδελφο Γιώργο Απο­

σrολόπουλο, Μεσολόγγι)

86. Δίνεται τρίγωνο ΑΒΓ και η διχοτόμος του ΒΔ. Α ν Μ είναι το μέσο της πλευράς ΑΒ και ΜΔ .l ΒΔ , να αποδείξετε ότι ΑΒ = 3ΒΓ .

Ν α αποδείξετε ότι: α) Για κάθε χ Ε IR , ισχύει: f (χ ) > χ - 1 .

β) Η συνάρτηση f είναι γνησίως αύξουσα στο IR .

γ) f (O) = O

δ) Η συνάρτηση f είναι συνεχής στο IR .

ε) Η συνάρτηση f αντιστρέφεται και να βρείτε την Γ' .

στ) Η εξίσωση f (χ 2 - 1) = f (-χ) έχει μία

τουλάχιστον ρίζα στο διάστημα (0, 1 ) .

ζ) Η συνάρτηση f είναι παραγωγίσιμη στο IR .

(Προτείνεται από το συνάδελφο Σαμπά Θεό­

δωρο, Πάτρα)

(Προτείνεται από το συνάδελφο Μπάμπη 89. Έστω μια συνάρτηση f : IR � IR , η οποία Στεργίου, Χαλκίδα) είναι παραγωγίσιμη στο 1R . Αν η συνάρτηση

87. Δίνεται η συνάρτηση f : [0, +οο) � IR για την οποία ισχύει: f (χ )ef(x ) = χ για κάθε χ � Ο .

Ν α αποδείξετε ότι: α) Για κάθε χ � Ο , ισχύει f (χ ) � Ο .

β) Η συνάρτηση f είναι γνησίως αύξουσα στο [Ο, +οο) .

γ) Η συνάρτηση f αντιστρέφεται και να βρείτε την f -1 •

δ) Η συνάρτηση f είναι συνεχής στο [Ο, +οο) .

ε) 1im f (x ) = +oo χ -Η«>

στ) Η συνάρτηση f είναι παραγωγίσιμη στο (Ο, +οο) .

(Προτείνεται από το συνάδελφο Σαμπά Θεό­

δωρο, Πάτρα)

88. Δίνεται συνάρτηση f : IR � IR για την οποία ισχύει f ( χ ) - e-f(x ) = χ - 1 για κάθε χ Ε IR και f ( IR) = IR .

f είναι συνεχής και γνησίως μονότονη και επιπλέον ισχύει: lim f (χ ) = 1 , 1 Ε IR , να α-

χ -+tοο

ποδείξετε ότι υπάρχει ένα, το πολύ, χ0 Ε IR , ώστε f ' (x0 ) = f (x0 ) .

(Προτείνεται από το συνάδελφο Κυριακόπου­

λο Θανάση, Αθήνα)

90. Έστω μια συνάρτηση f : IR � IR , η οποία είναι παραγωγίσιμη στο IR , χωρίς να είναι σταθερή σε κάποιο διάστημα, με f (α) = Ο και f ( χ0 ) > Ο για κάποιο χ0 > α .

Αν ισχύουν: f " ( x ) = p (x ) f (x ) , όπου ρ (χ) συνάρτηση συνεχής, μη σταθερή στο IR ώ­στε ρ (χ ) > Ο για κάθε χ Ε IR και f ' (κ)f (m) = f ' (m)f (κ ) για κάποιους κ, m με α < κ < m , να αποδείξετε ξ Ε (κ,m) με την

�ρ (ξ) = J 1nf (m) - 1n f (κ )J . m - κ

ότι υπάρχει ιδιότητα

(Προτείνεται από τον κύριο Ροδόλφο Μπόρη,

Δάφνη Αθήνα).

ΕΥΚΛΕΙΔΗΣ Β ' λθ ' τ.4Π4

Page 77: Ευκλειδης Β 60

Ο Ευκλείδης προτείνεL •. Ευκλείδη ... και Διόφαντο

91. Αν για τους πραγματικούς αριθμούς χ, y, z ' ' 1 1 1 1 ισχυουν οι σχεσεις -- + --+ -- =

χ + 1 y + 1 z + 1 χ Υ z και 3 + 3 + 3 = Ο , να α-

(χ + 1) (y + l) {z + 1) ποδείξετε ότι: χ 3 + y3 + z3 = 6 .

(Προτείνεται από τον μηχανικό Ανδρή Ιωάν­

νη, Αθήνα).

92. Αν για τους πραγματικούς αριθμούς α, β, γ ισχύουν αβγ :;e Ο και α + β + γ = αβγ , να α-ποδείξετε ότι για οποιοδήποτε ακέραιο κ, κ :;e Ο , ισχύει: ακ + βκ + γκ :;e Ο .

(Προτείνεται από τους συναδέλφους Τάκη

Δρούτσα και Νίκο Πανουσάκη, Αθήνα).

93. Να βρείτε διψήφιο ακέραιο αριθμό, ο οποίος να είναι ίσος με το άθροισμα του κύβου του ψηφίου των δεκάδων του και του τετραγώ­νου του ψηφίου των μονάδων του.

(Προτείνεται από τους συναδέλφους Τάκη

Δρούτσα και Νίκο Πανουσάκη, Αθήνα).

94. Αν για του πραγματικούς αριθμούς α, β, γ ισχύουν αβγ :;e Ο και β2 + γ2 - α2 γ2 + α2 _ β2 α2 + β2 - γ2 _,______:. __ + + = 1 2βγ 2αγ 2αβ να αποδείξετε ότι δύο από τα κλάσματα εί­ναι ίσα με + 1 και το illo είναι ίσο με - 1 . (Προτείνεται από το συνάδελφο Νίκο Αντω­

νόπουλο, 1λιον, Αθήνα)

95. Δίνεται η εξίσωση 3 ( χ2 + y2 ) + 2 { xy - 33 1) = 2 .

Να βρείτε τις ακέραιες λύσεις της.

(Προτείνεται από το συνάδελφο Γιώργο Τρι­

άντο, Αθήνα)

96. Σε κύκλο με κέντρο Ο και ακτίνα R θεω­ρούμε μια διάμετρο ΑΒ και σημείο Ρ στο εσωτερικό του. Αν Γ, Δ είναι τα σημεία το­μής των ΑΡ, ΒΡ με τον κύκλο και D1 , D2 οι δυνάμεις του σημείου Ο ως προς τους περι-

γεγραμμένους κύκλους C1 , C2 των τριγώ­νων ΡΒΓ και Ρ ΑΔ αντίστοιχα, να αποδείξετε ότι: ID1 - D2 1 � 2R · OH , όπου Η το δεύτερο κοινό σημείο των κύκλων C1 , C2 . (Προτείνεται από το συνάδελφο Γιώργος Τρι­

άντο, Αθήνα).

97. Έστω οι ν το πλήθος θετικοί ακέραιοι κ, , κ2 . . . κv . Να αποδείξετε ότι υπάρχουν δείκτες i και j με 1 � i � j � ν τέτοιοι, ώστε το άθροισμα κi+Ι + Κ;+2 . . . κj να είναι πολλα-πλάσιο του ν.

(Προτείνεται από το συνάδελφο Σωτήρη Σκο­

τίδα, Αθήνα)

98. Έστω S το σύνολο των τριγώνων των οποί­ων οι κορυφές έχουν ακέραιες συντεταγμέ­νες και είναι όμοια με δοσμένο τρίγωνο. Αν ΑΒΓ είναι το τρίγωνο του S με το ελάχιστο εμβαδόν, να αποδείξετε ότι το κέντρο του περιγεγραμμένου του κύκλου δεν έχει ακέ­ραιες συντεταγμένες. (Προτείνεται από το συνάδελφο Νίκο Αντω­

νόπουλο, 1λιον, Αθήνα).

Σ υ ν ά δ ε λ φ ο ι •Αν νοιώθετε την ανάγκη για μια ευρύτερη

ενημέρωση γύρω απο θέματα που αφορούν στη δουλειό του δασκόλου των μαθηματικών • Αν θέλετε δείγματα της δουλειός των

συναδέλφων μαθηματικών απ' όλη την Ελλόδα και απο την Κύπρο

• Αν θέλετε δείγματα και υλικό απο Μαθηματκοός Διαγωνισμοός απ' όλο τον Ι(όσμο

•Αν θέλετε να γίνετε κοινωνοί του παγκόσμ ιου προβληματισμού γύρω απο τα Αναλυτικό Προγρόμματα Μαθηματικών, γύρω απο τη Διδασκαλία των Μαθηματικών και τις νέες προτεινόμενες Μεθόδους

• Αν θέλετε επιλεγμένες «σπαζοκεφαλιές» και νοητικές προκλήσεις για τα παιδιό σας, τους μαθητές σας ή και τους φίλους σας,

• Αν θέλετε να χρησιμοποιήσετε μαθηματικό παιγνίδια για να κόνετε το μόθημα σας πιο ελκυστικό και αποτελεσματικό

• Αν θέλετε να αναθέσετε στους μαθητές σας μικρές ερευνητικές μαθηματικές εργασlες

•Αν θέλετε να γνωρίσετε παγκόσμια διόσημους επιστήμονες και τις σκέψεις τους γύρω απο τα Μαθηματικό και τη

-

Μαθηματική Εκπαίδευση,

ΟΜ ΑvrΑ ΚΑΙ ΜΜ ΠΟΛΛΑ ΠΑ ΔΥΟ ΤΕΥΧΗ ΤΟΥ Cp ΠΟΥ ΚΥΚΛΟΦΟΡΟΥΝ ΚΑΙ ΣτΟ 3ο ΠΟΥ ΘΑ ΚΥΚΛΟΦΟΡΗΣΕΙ ΤΟ ΦΘΙΝΟΠΩΡΟ

ΕΥΚΛΕΙΔΗΣ Β ' λθ ' τ.4Π5

Page 78: Ευκλειδης Β 60

Τ Α 18ΑΘ 11 J8AT Ι Ι( Α Α .Σ

.ΣI( εJJAZDYII Επιμέλεια: Παναγιώτης Χριστόπουλος

Τα μαθηματικά αν και είναι επιστήμη που απαιτεί αυστηρή διατύπωση, έχουν τη μαγεία να αποσπούν το

ενδιαφέρον όλων των ανθρώπων. Επινοήσεις σε προβλήματα ή ασκήσεις με κατάλληλο τρόπο διατυπωμένα

εξάπτουν το πνεύμα, διεγείρουν τη φαντασία και κεντρίζουν την περιέργεια. Πρώτοι οι Αρχαίοι Έλληνες

όπως ο Διόφαντος, ο Ζήνωνας κ.ά. μας δίδαξαν αυτά τα μαθηματικά.

Στη στήλη αυτή θα παρουσιάζουμε θέματα τα οποία δεν απαιτούν ιδιαίτερες μαθηματικές γνώσεις αλλά

μας διασΚεδάζουν με την εκφώνησή τους ή τη λύση τους και είναι μια ευχάριστη και συναρπαστική ασχολία

100 ευρώ 100 σοκολάτες

Μπορείτε να αγοράσετε με 100 Ευρώ 100 σο­κολάτες διαφορετικών μεγεθών αν η τιμή τους εί­ναι 10 ευρώ, 3 ευρώ και 0,5 ευρώ;

Η Κληρονομιά

Πέντε αδέλφια θέλουν να μοιράσουν τετρά­γωνο οικόπεδο με πλευρά 100 μέτρα.

Σύμφωνα με τη διαθήκη ο μεγαλύτερος θα πά­ρει τμήμα σχήματος τετραγώνου στη μια γωνία του οικοπέδου ίσο με το ένα τέταρτο. Το υπόλοιπο θα μοιραστεί στους άλλους 4,μπορείτε εσείς να βάλετε τους φράχτες γιατί οι ίδιοι δεν κατάφεραν να κάνουν τη διανομή ;

Οι 17 καμήλες

Ένας Αιγύπτιος θέλησε να μοιράσει τις 17 κα­μήλες του στους 3 γιούς του 2,3 και 9 ετών ανάλο­γα με τις ηλικίες τους χωρίς να το πετύχει. Τότε ένας σοφός φίλος του τον δάνεισε μια από τις κα­μήλες του και αφού έγινε η μοιρασιά με τις 1 8 πλέον καμήλες την ξαναπήρε.

Πώς έγινε το μοίρασμα; Είναι σωστό; Είχε 50 του. έμειναν 51

Ένας μαθητής είχε 50 ευρώ δίνει τα 25 για ένα βιβλίο του μένουν 25

« 5 για τετράδια « 20 « 1 5 για το θέατρο « 5 « 4 για ένα χυμό « 1 « 1 για τσίχλες « ο

Σύνολο 50 Σύνολο 5 1 ! ! ! Δώστε την απάντησή σας πώς έγινε αυτό;

Οι σοκολάτες

Η Αφροδίτη μοιράστηκε με τις φίλες της Σο­φία και Αρετή 8 σοκολάτες, τις 5 από αυτές είχε αγοράσει η Σοφία και τις 3 η Αρετή . Αφού τις έ­φαγαν η Αφροδίτη τους έδωσε 8 Ευρώ, πόσα πρέ­πει να πάρει η Σοφία και πόσα η Αρετή ;

Οι ολυμπιακοί κύκλοι

Να τοποθετήσετε τους αριθμούς 1 ,2,3 ,4, 5,6,7,8,9 σε καθεμιά από τις εννέα περιοχές (ένας αριθμός σε κάθε περιοχή) που ορίζουν οι πέντε ολυμπιακοί κύκλοι, ώστε το άθροισμα των αριθ­μών που είναι στο εσωτερικό κάθε κύκλου να είναι το ίδιο.

ΕΥΚΛΕΙΔΗΣ Β ' λθ ' τ.4Π6

Page 79: Ευκλειδης Β 60

Τα Μαθηματικά μας διασκεδάζουν

Οι Μαθητές

Οι μαθητές του 1 ου Γυμνασίου λένε πάντα την αλήθεια ενώ του2ου Γυμνασίου πάντα ψέματα Συναντάμε στο δρόμο μαθητές από αυτά τα σχολεία την Αθηνά και τον Ερμή . Ρωτάμε τον Ερμή : «Είστε μαθητές του Ι ου Γυμνασίου;)) μας απαντά «ναι)) . «Είστε μαθητές του ίδιου σχολείου;)) μας απαντά «όχι)) .

Από ποιο σχολείο είναι η Αθηνά και από ποιο ο Ερμής;

Η Κόρη

Οι μαθητές ρώτησαν το Μαθηματικό τους ποιο όνομα θα δώσει στη νεογέννητη κόρη του και αυτός απάντησε: Πάρετε έναν αριθμό 3ψήφιο με

διαφορεπκά ψηφία μονάδων και εκατοντάδων.

Δημιουργούμε τη θεπκή διαφορά που προκύπτει

μεταξύ του αριθμού μας και του αριθμού που

προκύπτει αν αλλάξουμε μεταξύ τους τα ακραία

ψηφία. Το ίδιο κάνουμε στα ψηφία της διαφοράς

και προσθέτουμε τον νέο αριθμό στην διαφορά.

Πολλαπλασιάζουμε το άθροισμα με 5, τα ψηφία

του γινομένου δίνουν το όνομα αν ανπκαταστα­

θούν με γράμματα ως εξής: τοl με � το 2 με Ψ,

το3 με Ω, το 4 με Ν, το 5 με Α, το6 με Ζ, το 7 με

Ρ, το 8 με Γ, το 9 με Λ, το Ο με Σ. Όλοι οι μαθη­

τές βρήκαν το όνομα ποιο είναι και πώς i:yινε αυ­

τό;

Από την αρχαιότητα είχαν διαπιστώσει ότι υ­πάρχουν τέλειοι αριθμοί φίλοι αριθμοί ευτυχι­

σμέ\'ΟΙ ή δυστυχΙσμέ\'ΟΙ αριθμοί, τρίγωνοι, τετρά­γωνοι, κ.ά.

Τέλειος αριθμός λέγεται ο αριθμός που το ά­θροισμα των γνήσιων διαιρετών του (εκτός του ι­δίου) είναι ο ίδιος ο αριθμός. Τέλειοι είναι οι 6,28, 496, 8 1 28, . . . . Βέβαια είναι λίγοι γνωστοί και κάτι

ακόμα είναι όλοι άρτιοι, τρίγωνοι και το άθροισμά των αντιστρόφων του συνόλου των διαιρετών τους είναι πάντα δύο.

Φίλοι είναι οι αριθμοί που ο καθένας είναι το άθροισμα των γνήσιων διαιρετών του άλλου. π. χ, 220, 284.

Ευτυχισμένος Παίρνουμε το άθροισμα των τετραγώνων των ψηφίων του στον αριθμό που προκύπτει παίρνουμε πάλι τον άθροισμα των τε­τραγώνων των ψηφίων του, ξανά και ξανά μέχρι να βρούμε μονοψήφιο. Αν ο μονοψήφιος είναι ο αριθμός 1 τότε ο αριθμός μας είναι ευτυχισμένος, για κάθε άλλο ψηφίο δυστυχισμένος.

Μαγικοί

Πολλαπλασιάστε τον αριθμό 1 23456789 με τα πολλαπλάσια του 9 .

Σβήστε ένα από τα ψηφία και πολλαπλασιάστε με τα πολλαπλάσια του 9.

Βρείτε την τετραγωνική του ρίζα. Πολλαπλασιάστε αριθμούς που τα ψηφία τους

είναι μόνο μονάδες με τον εαυτό τους. Πολλαπλασιάστε τον 37 με τα πολλαπλάσια

του 3 . Πολλαπλασιάστε τον 143 με τα πολλαπλάσια

του 7. Πολλαπλασιάστε τον 142857 με τους 1 ,2,3, . . . ,

10.

Από τη Γη στη Σελήνη

Έχετε διπλώσει ένα σεντόνι περισσότερες από 7 φορές;

Αν διπλώνατε ένα σεντόνι 35 φορές τι ύψος θα είχε;

Η Πίτα

Κάποιος αποφάσισε τη μια μέρα να φάει τη μισή πίτα, την άλλη μέρα από αυτή που έμεινε πά­λι τη μισή, την 3η μέρα το ίδιο, σκέφτηκε ότι έτσι η πίτα του δεν θα τελειώσει ποτέ, σωστά;

ΕΥΚΛΕΙΔΗΣ Β' λθ ' τ.4Π7

Page 80: Ευκλειδης Β 60

2 4 3 5 1 8 7 9 9 6 5 4 7 1 9 6

3 5 6 8 4 2 8 7 5 7 4 3 8 9 1 2 6 3 2 1

ΕΥΚΟΛΟ

7 8 1 6

5 9 3 4 6 2 4 1 4 7 2 9 8 3 5 7 9 1 6 3 1 5 7 8 2 4 9 5 3 6 8 2

ΜΕΤΡ ΙΟ

7 3 6 4 9 1 2 7 4 5 8 2 2 6 1 8 3 9 4 5 8 7 5 6

1 8 9 3 5 2 7 1 6 4 3 9

ΔΥΣΚΟΛΟ

Τα Μαθηματικά μας διασκεδάζουν

Απαντιίσεις Τα Μαθηματικά μας διασκεδάζουν 3ου Τεύχους

8 7 1 3 6 5 1 2 8 2 3 4 4 9 2 5 1 3 9 8 6

6 4 7 7 5 9

3 9 4 2 7 1 5 8 7 8 5 6

6 1 2 4 2 5 9 6 3 7 3 8 1 4 9

8 9 5 6 5 3 3 1 9 9 4 7 2 7 8 1 3 2 7 6 4 4 8 6

5 2 1

6 9 2 4 3 7 8 5 7 1 9 6

1 2 5 3 4 8

2 5 8 6

3 9 1 3 9 4 7 8 4 2 6 1 5 7

2 1 8 4 6 7 3 5 1 6

4 9 5 2 9 3 7 8

Η Ι ΣΟΤΗΤΑ lη λύση 3 + 8 - 2 19 ;f 1 29, 2η λύση 3 4 8 -

219 = 1 29, 3η λύση 3 + 8 - 2 19 < 129 . ΤΑ 9 Σ Η Μ ΕΙΑ

Η Ο Ι ΚΟ Γ Ε Ν Ε Ι Α Δύο αδέλφια (αδελφός και αδελφή) με τον γιο

του ο ένας και την κόρη του ο άλλος. Ο Ι Π ΠΕΣ Ψήνει τις δύο πίτες 1 η και 2η από την μια μεριά

(2 λεπτά) στη συνέχεια μια από αυτές π. χ. την 1 η από την άλλη πλευρά μαζί με την 3η (2 λεπτά) . Τέ­λος την 3η και την 2η από την άλλη πλευρά (2 λε­πτά) . Σύνολο 6 λεπτά.

Η ΠΑΛ Ι ΡΡΟ Ι Α ΚΛ Ι ΤΟ KΛ J>A B I Με την παλίρροια το καράβι θα ανυψωθεί και η

σκάλα θα είναι βυθισμένη το ίδιο. Ο ΕΞΥΠΝΟΣ Μ ΑΘΗΤΗΣ Ο Μιχάλης βάζει αριθμούς ώστε το άθροισμα

των ψηφίων με τους αντίστοιχους αριθμούς που δίνει η Βάσω να είναι 9. Δηλαδή 2+7, 5+4, 4+5 , 1+8 .

Έτσι έχει 9999+9999=20000-2 από τον αριθμό 7543 αφαιρεί δύο μονάδες και τις θέτει ως ψηφίο δεκάδων χιλιάδων.(το ίδιο γίνεται και με αριθμούς με διαφορετικό αριθμό ψηφίων).

Η ΓΥΝΑ Ι ΚΑ Πήγαινε Πεζή . ΤΑ ΤΕΤΡΆΓΩΝΑ

Οι Απαντήσεις για «τα Μαθηματικά μας διασκε­

δάζουν» του 4°11 Τεύχους είναι στην σελίδα 66.

ΕΥΚΛΕΙΔΗΣ Β' λθ ' τ.4Π8

Page 81: Ευκλειδης Β 60

ΑΝΑΚΟΙΝΩΣΗ Η Ελληνική Μαθηματική Εταιρεία στην ετήσια Γενική της Συνέλευση που πραγματοποιήθηκε την

Κυριακή 12 Μαρτίου 2006 αποφάσισε την αύξηση της ετήσιας συνδρομής των μελών από το έτος 2006, από 15€ σε 20€.

Σημειώνουμε ότι η συνδρομή είχε την ίδια τιμή τα τελευταία οκτώ χρόνια και η αύξηση αυτή ήταν επιβεβλημένη μετά από τη σημαντική αύξηση των τελών αποστολής των περιοδικών στα μέλη από τα Ελληνικά Ταχυδρομεία, όπως διαπιστώσαμε στο τέλος Ιανουαρίου.

Επειδή η επιβάρυνση της αποστολής των περιοδικών καθώς και η συνεχής αύξηση του κόστους έκδοσης των περιοδικών είναι μεγάλη παρακαλούμε όπως εξοφλείτε τη συνδρομή σας έγκαιρα στην Ε.Μ.Ε.

Για ενημέρωσή σας, επάνω στην ετικέτα με το όνομα σας αναγράφεται το συνολικό ποσό το οποίο οφείλετε στην Ε .Μ.Ε . Οι τρόποι που μπορείτε να πληρώσετε τη συνδρομή σας είναι:

Στα γραφεία της Ε.Μ.Ε. Στα γραφεία των παραρτημάτων της Ε .Μ.Ε . • Με ταχυδρομική επιταγή σε διαταγή, ΕΛΛΗΝΙΚΗ ΜΑΘΗΜΑτΙΚΗ ΕΤΑΙΡΕΙΑ, ΤΑΧ. ΓΡΑΦΕΙΟ

ΑΘΗΝΑ 54, Τ. Θ. 30044

Είτε με κατάθεση του αντιτίμου της συνδρομής στους παρακάτω λογαριασμούς: � Τράπεζα ΕΘΝΙΚΗ, λογαριασμός όψεως 080/48002300 � Τράπεζα ALPHA, λογαριασμός όψεως 10 100 200 20 19 988

ΠΡΟΣΟΧΗ: Στην περίπτωση που η πληρωμή θα γίνει σε τράπεζα πρέπει να αποστείλετε στο Fax της ΕΜΕ (2 10-364 1 025) την απόδειξη κατάθεσης συμπληρωμένη με τα πλήρη στοιχεία του καταθέτη (ονοματεπώνυμο, διεύθυνση, τηλέφωνο) ώστε να είναι εφικτή η ορθή αποστολή των περιοδικών.

Από το Δ.Σ. της Ε.Μ.Ε .

ΕΝΗΜΕΡΩΣΗ ΤΟΥ ΑΡΧΕΙΟΥ ΤΗΣ Ε.Μ.Ε. Αγαπητοί Συνάδελφοι, Η ΕΜΕ για την καλύτερη επικοινωνία και συνεργασία με τα μέλη της, στο πλαίσιο αναβάθμισης και

εμπλουτισμού της ιστοσελίδας της αλλά και την πληρέστερη ενη μέρωσή σας σε θέματα που αφορούν τόσο στις δραστηριότητες του Σωματείου μας, αλλά και σε γενικότερα θέματα αποφάσισε να ανανεώσει και να συμπληρώσει το αρχείο των μελών της.

Για το λόγο αυτό σας παρακαλούμε, αφού συμπληρώσετε με ιδιαίτερη προσοχή και με ΚΕΦΑΛΑΙΑ γράμματα τη φόρμα που ακολουθεί, να την αποστείλετε στην ΕΜΕ το συντομότερο δυνατό με fax στο 21 0-364 1 025 ή με e-mail στο [email protected] ή ταχυδρομικά. Εάν μέχρι τέλος Ιουνίου δεν έχουμε λάβει το σχετικό έντυπο θα θεωρήσουμε ότι τα στοιχεία που έχουμε στο αρχείο μας είναι σωστά και θα εξακολουθήσουμε να σας αποστέλλουμε τα περιοδικά «Ευκλείδης Α' και Β ' ». Διευκρινίζεται ότι στη θέση «ΕΡΓΑΣΙΑ}} πρέπει να συμπλη ρωθούν τα πλήρη στοιχεία της υπηρεσίας της έπιχείρησης, του οργανισμού κλπ. στην οποία εργάζεστε. Επίσης στη θέση «ΠΕΡΙΟΔΙΚΟ}} εάν στην ίδια οικογένεια υπάρχουν 2 άτομα που είναι μέλη της Ε.ΜΕ. έχετε τη δυνατότητα να επιλέξετε δύο από τα τρία περιοδικά (μικρός Ευκλείδης, Ευκλείδης Α ' , Ευκλείδης Β ' από τέσσερα τεύχη το χρόνο) ή ένα από τα τρία επιστημονικά περιοδικά (Ευκλείδης Γ, Μαθη ματική Επιθεώρηση , από δύο τεύχη το χρόνο, Δελτίο από ένα τεύχος το χρόνο) .

Είμαστε βέβαιοι για τη θετική ανταπόκρισή σας στην προσπάθεια της Εταιρείας να ενημερώσει το αρχείο και να καλύψει τις δικές σας απαιτήσεις, προς όφελος όλων, σας ευχαριστούμε εκ των προτέρων.

ΚΩΔΙΚΟΣ ΜΕΛΟΥΣ [ . . . . . . . . . . . . . . • . ] ΟΝΟΜΑ : [ . . . . . . . . . . . . . . . . . . . . . . . . . . . . . . . . . . . . . . . . . . . . . . . . . . . . . . . . . . . . . . . . . . . . . . . . . ] ΕΠΩΝΥΜΟ: [ . . . . . . . . . . . . . . . . . . . . . . . . . . . . . . . . . . . . . . . . . . . . . . . . . . . . . . . . . . . . . . . . . . . . . . . . . . . . . . . . . ] ΟΝΟΜΑ ΠΑΤΡΟΣ: [ . . . . . . . . . . . . . . . . . . . . . . . . . . . . . . . . . . . . . . . . . . . . . . . . . . . . . . . . . . . . . . . . . . . . . . . . . . . . . . . . ] ΗΜ/ΝΙΑ ΓΕΝΝΗΣΗΣ : . . . . . / . . . . / . . . . . . . ΟΔΟΣ : [ . . . . . . . . . . . . . . . . . . . . . . . . . . . . . . . . . . . . . . . . . . . . . . . . . . . . . . . . . . . . . . . . . . . . . . . . . . . . . ΑΡ.: . . . . . . . . . . . . . . ] ΠΟΛΗ I ΠΕΡΙΟΧΗ : [ . . . . . . . . . . . . . . . . . . . . . . . . . . . . . . . . . . . . . . . . . . . . . . . . . . . . . . . . . . . . . . . . . . . . . . . . . . . . . . . . . . . . . . . . . . . . . . . ] ΤΑΧ. ΚΩΔΙΚΟΣ: [ . . . . . . . . . . . . . . . ] ΤΗΛΕΦΩΝΟ: [ . . . . . . . . . . . . . . . . . . . . . . . . . . . . . . . . . . . . . . . . . . . . . . . . . . . . . . . . . . . . . . . . . . . . . . . . . . . . . . . . . . . . . . . . . . . . . . . . . . . ΚΙΝ.: . . . . . . . . . . . . . . . . . . . . . . . . . . . . . . . , . . . . . . . . . . . . . . . . . . . . . . . . . . . . . . ] FAX:. [ . . . . . . . . . . . . . . . . . . . . . . . . . . . . . . . . . . . . . . . . . . . . . . . . . . . . . . . . . . . . . . . . . . ] E-MAIL : [ . . . . . . . . . . . . . . . . . . . . . . . . . . . . . . . . . . . . . . . . . . . . . . . . . . . . . . . . . . . . . . ] ΣΠΟΥΔΕΣ: Α' ΠΤΥΧΙΟ [ . . . . . . . . . . . . . . . . . . . . . . . . . . . . . . . . . . . . . . . . . . . . . . . . . . . . . . . . . . . . . . . . . . . . . . . . . . . . . . . . . . . . . . . . . . . . . . . . . . . . . . . . . . . . . . . . . . . . . . . . . . . ]

ΜΕΤΑΠΤΥΧΙΑΚΑ [ . . . . . . . . . . . . . . . . . . . . . . . . . . . . . . . . . . . . . . . . . . . . . . . . . . . . . . . . . . . . . . . . . . . . . . . . . . . . . . . . . . . . . . . . . . . . . . . . . . . . . . . . . . . . . . . . . . . . . . . . . . ] ΔΙΔΑΚΤΟΡΙΚΟ [ . . . . . . . . . . . . . . . . . . . . . . . . . . . . . . . . . . . . . . . . . . . . . . . . . . . . . . . . . . . . . . . . . . . . . . . . . . . . . . . . . . . . . . . . . . . . . . . . . . . . . . . . . . . . . . . . . . . . . . . . . . ]

ΕΡΓΑΣΙΑ [ . . . . . . . . . . . . . . . . . . . . . . . . . . . . . . . . . . . . . . . . . . . . . . . . . . . . . . . . . . . . . . . . . . . . . . . . . . . . . . . . . . . . . . . . . . . . . . . . . . . . . . . . . . . . . . . . . . . . . . . . . . . . . . . . . . . . . . . . . . . . . . . . . . . . . . . . . . . . . . . . . . . . . . . . . . . . . . . . . . . . . . ] ΠΕΡΙΟΔΙΚΟ: Μικρό Ευκλείδη Ο ΕΥΚΛΕΙΔΗ Γ Ο

ΕΥΚΛΕΙΔΗ Α ' Ο ΜΑΘΗΜΑτJΚΗ ΕΠΙΘΕΩΡΗΣΗ. Ο ΕΥΚΛΕΙΔΗ Β ' Ο ΔΕΛτJΟ Ο

Page 82: Ευκλειδης Β 60

Ελληνική Μαθηματική ΕταιQεία ΠΑΝΕΛΛΗΝΙΟ ΣΥΝΕΔΡΙΟ

ΜΑΘΗΜΑΤΙΚΗΣ ΠΑΙΔΕΙΑΣ

Με διεθνείς συμμετοχές Πάτρα 24 - 25 - 26 Νοεμβρίου 2006

ΤΑ •ΑeΝ.ΑΤΙΙιΑ 11% DΟΑΙΤΙ%.0% %ΤΟ %ΊrΧΡΟΝΟ 1(0%.0, DΡΟ8ΙιΤΑ%8Ι% %ΤΝΝ Ι(ΡΙΤΙΙ(Ν %1(8ΨΝ .

ΤΝΝ 8DΙΧ8ΙΡΝ.ΑΤΟΑΟrΙΑ ΙιΑΙ ΤΝΝ ΑΙ%tJΝΤΙΙιΝ Τα μαθηματικά είναι ένα από τα πιο σημαντικά πολιτιστικά συστατικά κάθε σύγχρονης κοινωνίας. Η

επιρροή τους πάνω σε όλα τα άλλα πολιτιστικά στοιχεία ε ίναι τόσο θεμελιώδης και διάχυτη που ε ίναι φανερό ότι ο σύγχρονος τρόπος ζωής μας δεν θα υπήρχε χωρίς τα μαθηματικά.

Σκοπός του Συνεδρίου είναι να αποτελέσει το βήμα παρουσίασης και διαλόγου σχετικά με ερευνητικές μελέτες και εργασίες, προτάσεις, αναλύσεις και θεωρητικά πλαίσια που σχετίζονται με τα μαθηματικά ως κουλτούρα στο σύγχρονο κόσμο και την ανάπτυξη της κριτικής σκέψης, της επιχειρηματολογίας και της αισθητικής μέσα από τη διδασκαλία των μαθηματικών σε όλες τις βαθμίδες της εκπαίδευσης και τη χρήση των μαθηματικών στις άλλες επιστήμες, την τέχνη και την τεχνολογία.

Θεματικές Ενότητες του Συνεδρίου

1 . Ο πολιτισμός των Μαθηματικών και στις τρε ις βαθμίδες της εκπαίδευσης 2. Τα Μαθηματικά ως στοιχείο επικοινωνίας μεταξύ των ανθρώπων και των πολιτισμών 3. Φύλο και Μαθηματικά 4. Τα Μαθηματικά στην Τέχνη και στην Τεχνολογία 5 . Επιχειρηματολογία και μαθηματική απόδειξη στην πορεία του χρόνου και η διδακτική αξιοποίηση τους

Οδηγίες για τη Σύνταξη των Εργασιών

Οι εργασίες οι οποίες θα υποβληθούν μέχρι τις 4 Σεπτεμβρίου 2006 θα κριθούν με βάση τα πλήρη κείμενα από την επιστη μονική επιτροπή. Θα πρέπει να ε ίναι πρωτότυπες και να εντάσσονται στους σκοπούς του συνεδρίου,

Οι εργασίες μπορεί να παρουσιάζουν έρευνα πεδίου ή να αναπτύσσουν ένα θεωρητικό θέμα που · άπτεται της Μαθηματικής Παιδείας (όπως φιλοσοφικές - γνωστικές θεωρήσεις) .

Στο πρώτο είδος εργασιών πρέπει να αναφέρεται το θεωρητικό πλαίσιο που ο (η) συγγραφέας στήριξε τον σχεδιασμό της έρευνας πεδίου . Επίσης πρέπει να παρουσιάζεται μια σύντομη επισκόπηση της �φλιαγραφίας στο θέμα υπό εξέταση. Η μεθοδολο'{ία ανάλυσης των δεδομένων (που αναλυτικά θα

παρατίθετω.) μπορεί να είναι ποιοτική ή ποσοτική. , , Τα συμπεράσματα (και στα δύο είδη εργασιών) θα ε μπε�ιέχουν �ολιασμο για

_τη σημασια το�ς και τη

συμβολή τους στους θεματικούς άξονες του συνεδριου, οι οποιοι θα πρε�ει να αναφερο�αι ευκρινως; τα πλήρη κείμενα των εργασιών που θα εγκριθούν θα περιληφθουν στα πρακτικα του συνεδριου, τα

οποία θα καταβληθεί προσπάθεια να εκδοθούν πριν το συνέδριο.

ΣΗΜΑΝΠΚΗ ΗΜΕΡΟΜΗΝΙΑ 4 Σεπτεμβρίου 2006

Τα πλήρη κείμενα των εργασιών (μέχρι 10 σελίδες) θα πρέπει να υποβληθούν μέχρι ττις 4 Σεπτεμβρίου 2006 στην παρακάτω διεύθυνση :

Ε.Μ.Ε. (για το 23ο Συνέδριο της ΠΑ'fΡΑΣ) Πανεπιστημίου 34 - 106 79 Αθήνα

Τηλ. : 210 3616 532 - 3617 784 Fax: 210 3641 025 e-mail : [email protected]

www.hms.gr

Ιστοσελίδα Συνεδρίου

Παράρτημα Ε.Μ.Ε. Αχαίας Καπποδιστρίου 52

26222 Πάτρα Τηλ./Faχ: 2610-422273

e-mail: [email protected]

http:/ /www .emepatras.gr/emec23/ email: [email protected], [email protected]

ΟΡΓΑΝΩΣΗ: Ελληνική Μαθηματική Εταιρεία, Παράρτημα Ε.Μ.Ε. Αχαίας Περιφερειακό Ταμείο Ανάπtυξης Δυτικής Ελλάδας, Τμήμα Μαθηματικών Πανεπιστημίου Πατρών

Page 83: Ευκλειδης Β 60

από τις εκδόσεις <<Εν Δυνάμει>>

Για την Γ' Λυκείου • Φυσική ΙΚστεύβυνσης (ταλαντώσεις - Κύματα) - Γ. θ. Ντούβαλης

• Φυσική Κατεύβυνσπς (Μηχανική του στερεού σώματος) - Γ. θ. Ντούβαλης

• Φυσική Κατεύβυνσης (Κρούσεις - Φαινόμενο Doppler) - Γ. θ. Ντούβαλης

• Ασκήσεις Βιολοyίαc; Γενικής Παιδείας - Κ. Ρ. Παπαζήσης

• Μα8πματικά Κατεύβυνσης (Παράγωγοι) - Γ. & Π. Λοuκόποuλος • Μα8πματικά Κατεύβυνσπς (Ολοκληρώματα) - Γ. & Π. Λοuκόποuλος

• Έκφρασπ -Έκθε:ση - Α. Καλλή - Γ. Σοuλτάνης

• Ανάπτυξη εφαρμογών σε προyραμμστιστικό ιιεριβά)...λον - Κ. Ν. Ιορδανόποuλος • Αρχές Οικονομικής Θεωρίαc; - Κ. Γαροφαλάκης

Για την Α' Λυκείου • Φυσική - Κ. Ρ. Παπαζήσης

• 'Άλγεβρα - Γ. & Π. Λοuκόποuλος

Ειι Δuιιόμ&ι Ε Κ Δ Ο Σ Ε Ι Σ

ΧΡΥΣΙΠΠΟΥ 1 & ΟΥΛΟΦ ΠΑΛΜΕ - ΖΩΓΡΑΦΟΥ Τηλ. : 2 1 0 74 88 030, fax: 2 1 0 74 83 03 1

:z -:ΞΙ Ξ

Page 84: Ευκλειδης Β 60

• � .

ΕΛΛΗΝΙΚΉ ΜΑΘΗΜΑΤΙΚΉ ΕΤΑΙΡΕΙΑ Πανεπιστημίου (Ελευθερίου Βενιζέλου) 34 - 1 06 79 ΑΘΗΝΑ

Τηλ. 3 6 1 6532 - 3 6 1 7784 - Fax: 3 64 1 025 www.hms.gr - e-mail : [email protected]

Παίγνια και Λήψη Αποφάσεων Σελίδες 3 1 8

ISBN: 960-7341 -27-9. Σχήμα 1 9Χ23 Λ.Τ. 28 €

Το βιβλίο πραγματεύεται με ενιαίο τρόπο το αντικείμενο της Θεωρίας Παιγνίων και της κλασικής Θεωρίας Βελτιστοποίησης υπό το πρίσμα των εφαρμογών των θεωριών και των τεχνικών τους στα οικονομικά, τις πολιτικές επιστήμες και τη διοίκηση επιχειρήσεων. Αποφεύγοντας τη σχολαστικότητα, αλλά χωρίς να υστερεί σε μαθηματική αυστηρότητα, το βιβλίο απευθύνεται κυρίως σε προπτυχιακούς αλλά και μεταπτυχιακούς φοιτητές των οικονομικών, θετικών και τεχνολογικών σχολών, καθώς και σε όσους ενδιαφέρονταrεΥγένει για τη Θεωρία Παιγνίων και τη Θεωρία Αποφάσεων. Ο μεγάλος α9�θμός των επεξεργασμένων παραδειγμάτων και των προς επίλυση ασ.κησεων που καλύπτουν ένα ευρύτατο φάσμα εφαρμογών, εισάγουν βαθμιαία τον αναγνώστη στη μεθοδολογία και τη λογική της θεωρίας, και τον διδάσκουν πώς να αξιολογεί και να εφαρμόζει κατάλληλα τις σχετικές τεχνικές

ΗΡΩΝΟΣ ΑΛΕΞΑΝΔΡΕΩΣ

Ν Ε ΕΣ ΕΚΔΟΣΕ Ι Σ

ΕΛΙ\ΗΝΙΚΗ ΜΑΘΗΜΑηΚΗ ΕΤΑΙΡΕΙΑ

1/ιιί ,.,, ιιιιι

lfψf ΙrιΙφtίtιcωr Χ. Δ. Αλιnρόντn ι<αι S. Κ. Chal<rabarti

Η Ελληνική Μαθηματική Εταιρεία με μεγάλη ικανοποίηση και υπερηφάνεια παραδίδει το πολύ σημαντικό αυτό έργο όχι μόνο στους Μαθηματικούς αλλά και σε όλους αυτούς που θέλουν να μελετήσουν σε βάθος το Ηρωνικό έργο.

ΗΡΩΝΟΣ ΑΛΕΞΑΝΔΡΕΩΣ

ΟΝΟΜΑΤΑ ΓΕΩΜΕΤΡΙΚΏΝ

ΟΡΩΝ ΓΕΩΜΕΤΡΙΚΆ

Σχήμα 1 7Χ24 Σελίδες 5 1 6

ISBN: 960-734 1 - 1 6-3 Λ.Τ. 2 0 €

Ο Αλεξανδρινός «μηχανικός» Ηρών εκτός από τις περίφημες πραγματείες του που αναφέρονται σε θέματα κατασκευής και λειτουργίας διαφόρων μηχανών, μας έδωσε και αξιόλογα μαθηματικά κείμενα

ΗΡΩΝΟΣ ΑΛΕΞΑΝΔΡΕΩΣ

ΜΕΤΡ/ΚΑ - ΔΙΟΠΤΡΑ

Σχήμα 1 7Χ24 Σελίδες 328

ISBN: 960-7341 -22-8 Λ.Τ. 20 €

ΗΡΩΝΟΣ ΑΛΕΞΑΝΔΡΕΩΣ

ΣΤΕΡΕΟΜΕΤΡΙΚΆ

Σχήμα 1 7Χ24 Σελίδες 4 1 4

ISBN: 960-734 1 -26-0 Λ.Τ. 2 0 €

Συνάδελφοι Επισκεφθείτε την ιστοσελίδα της Ε.Μ.Ε. (www.hms.gr ) όπου υπάρχουν όλα τα τεύχη του Ευκλέιδη Γ ' και της Μαθηματικής Επιθεώρησης με τα ?fεριεχόμενά τους καθώς και όλες οι εκδόσεις της Ελληνικς Μαθηματικής Εταιρείας (Ε.Μ.Ε.).